Acute 222 final

Pataasin ang iyong marka sa homework at exams ngayon gamit ang Quizwiz!

A primigravida has just delivered a healthy infant girl. The nurse is about to administer erythromycin ointment in the infant's eyes when the mother asks, "What is that medicine for?" How should the nurse respond?

"Erythromycin is prophylactically given to prevent a gonorrheal infection."

During a prenatal examination, a woman reports having two cats at home. The nurse informs her that she should not be cleaning the litter box while she is pregnant. The client questions the nurse as to why. What is the nurse's most appropriate response?

"Your cats could be carrying toxoplasmosis. This is a zoonotic parasite that can infect you and have severe effects on your unborn child."

Which assessments will the nurse make to monitor a patient's cerebellar function (select all that apply)?

*Observe arm swing with gait. *Perform the finger-to-nose test.

PPH may be sudden and result in rapid blood loss. The nurse must be alert to the symptoms of hemorrhage and hypovolemic shock and be prepared to act quickly to minimize blood loss. Astute assessment of the client's circulatory status can be performed with noninvasive monitoring. Match the type of noninvasive assessment that the nurse would perform with the appropriate clinical manifestation or body system. a.Palpation b.Auscultation c.Inspection d.Observation e.Measurement 1. Pulse oximetry 2. Heart sounds 3. Arterial pulses 4. Skin color, temperature, and turgor 5. Presence or absence of anxiety

1. ANS: E DIF: Cognitive Level: Apply REF: p. 808 TOP: Nursing Process: Assessment MSC: Client Needs: Physiologic Integrity NOT: To perform a complete noninvasive assessment of the circulatory status in postpartum clients who are bleeding, the nurse must perform the following: palpation (rate, quality, equality) of arterial pulses; auscultation of heart sounds or murmurs and breath sounds; inspection of skin color, temperature, and turgor; level of consciousness; capillary refill, neck veins, and mucous membranes; observation of either the presence or absence of anxiety, apprehension, restlessness, and disorientation; and measurement of blood pressure, pulse oximetry, and urinary output. 2. ANS: B DIF: Cognitive Level: Apply REF: p. 808 TOP: Nursing Process: Assessment MSC: Client Needs: Physiologic Integrity NOT: To perform a complete noninvasive assessment of the circulatory status in postpartum clients who are bleeding, the nurse must perform the following: palpation (rate, quality, equality) of arterial pulses; auscultation of heart sounds or murmurs and breath sounds; inspection of skin color, temperature, and turgor; level of consciousness; capillary refill, neck veins, and mucous membranes; observation of either the presence or absence of anxiety, apprehension, restlessness, and disorientation; and measurement of blood pressure, pulse oximetry, and urinary output. 3. ANS: A DIF: Cognitive Level: Apply REF: p. 808 TOP: Nursing Process: Assessment MSC: Client Needs: Physiologic Integrity NOT: To perform a complete noninvasive assessment of the circulatory status in postpartum clients who are bleeding, the nurse must perform the following: palpation (rate, quality, equality) of arterial pulses; auscultation of heart sounds or murmurs and breath sounds; inspection of skin color, temperature, and turgor; level of consciousness; capillary refill, neck veins, and mucous membranes; observation of either the presence or absence of anxiety, apprehension, restlessness, and disorientation; and measurement of blood pressure, pulse oximetry, and urinary output. 4. ANS: C DIF: Cognitive Level: Apply REF: p. 808 TOP: Nursing Process: Assessment MSC: Client Needs: Physiologic Integrity NOT: To perform a complete noninvasive assessment of the circulatory status in postpartum clients who are bleeding, the nurse must perform the following: palpation (rate, quality, equality) of arterial pulses; auscultation of heart sounds or murmurs and breath sounds; inspection of skin color, temperature, and turgor; level of consciousness; capillary refill, neck veins, and mucous membranes; observation of either the presence or absence of anxiety, apprehension, restlessness, and disorientation; and measurement of blood pressure, pulse oximetry, and urinary output. 5. ANS: D DIF: Cognitive Level: Apply REF: p. 808 TOP: Nursing Process: Assessment MSC: Client Needs: Physiologic Integrity NOT: To perform a complete noninvasive assessment of the circulatory status in postpartum clients who are bleeding, the nurse must perform the following: palpation (rate, quality, equality) of arterial pulses; auscultation of heart sounds or murmurs and breath sounds; inspection of skin color, temperature, and turgor; level of consciousness; capillary refill, neck veins, and mucous membranes; observation of either the presence or absence of anxiety, apprehension, restlessness, and disorientation; and measurement of blood pressure, pulse oximetry, and urinary output.

b

A newborn was admitted to the neonatal intensive care unit (NICU) after being delivered at 29 weeks of gestation to a 28-year-old multiparous, married, Caucasian woman whose pregnancy was uncomplicated until the premature rupture of membranes and preterm birth. The newborn's parents arrive for their first visit after the birth. The parents walk toward the bedside but remain approximately 5 feet away from the bed. What is the nurse's most appropriate action? a. Wait quietly at the newborn's bedside until the parents come closer. b. Go to the parents, introduce him or herself, and gently encourage them to meet their infant. Explain the equipment first, and then focus on the newborn. c. Leave the parents at the bedside while they are visiting so that they have some privacy. d. Tell the parents only about the newborn's physical condition and caution them to avoid touching their baby.

c

A nurse practicing in the perinatal setting should promote kangaroo care regardless of an infant's gestational age. Which statement regarding this intervention is most appropriate? a. Kangaroo care was adopted from classical British nursing traditions. b. This intervention helps infants with motor and CNS impairments. c. Kangaroo care helps infants interact directly with their parents and enhances their temperature regulation. d. This intervention gets infants ready for breastfeeding.

a

A pregnant woman was admitted for induction of labor at 43 weeks of gestation with sure dates. A nonstress test (NST) in the obstetrician's office revealed a nonreactive tracing. On artificial rupture of membranes, thick meconium-stained fluid was noted. What should the nurse caring for the infant after birth anticipate? a. Meconium aspiration, hypoglycemia, and dry, cracked skin b. Excessive vernix caseosa covering the skin, lethargy, and RDS c. Golden yellow to green-stained skin and nails, absence of scalp hair, and an increased amount of subcutaneous fat d. Hyperglycemia, hyperthermia, and an alert, wide-eyed appearance

b

A premature infant never seems to sleep longer than an hour at a time. Each time a light is turned on, an incubator closes, or people talk near her crib, she wakes up and inconsolably cries until held. What is the correct nursing diagnosis beginning with "ineffective coping, related to"? a. Severe immaturity b. Environmental stress c. Physiologic distress d. Behavioral responses

a

A premature infant with respiratory distress syndrome (RDS) receives artificial surfactant. How does the nurse explain surfactant therapy to the parents? a. "Surfactant improves the ability of your baby's lungs to exchange oxygen and carbon dioxide." b. "The drug keeps your baby from requiring too much sedation." c. "Surfactant is used to reduce episodes of periodic apnea." d. "Your baby needs this medication to fight a possible respiratory tract infection."

Nursing activities that promote parent-infant attachment are many and varied. One activity that should not be overlooked is the management of the environment. While providing routine mother-baby care, the nurse should ensure that: A. An environment that fosters as much privacy as possible should be created B. The father should be encouraged to go home at night to prepare for mother-baby discharge C. The baby is able to return to the nursery at night so that the new mother can sleep D. Routine times for care are established to reassure the parents

A. An environment that fosters as much privacy as possible should be created

When assisting the patient in labor and delivery, the nurse knows that operative vaginal births may include: SELECT ALL THAT APPLY (partial credit will not be given) A. C-Section delivery B. Augmentation delivery C. Forceps delivery D. Vacuum extraction delivery

A. C-Section delivery B. Augmentation delivery C. Forceps delivery D. Vacuum extraction delivery

A prolapsed cord occurs when cord lies below the presenting part of the fetus and is an obstetrical emergency. When assisting a patient with a prolapsed cord the nurse knows that the following interventions are essential: SELECT ALL THAT APPLY ( partial credit will not be given). A. Call for assistance, do not leave woman alone B. Administer oxygen via a nonrebreather mask at 8 to 10 L/min at least until birth is accomplished C. With a sterile gloved hand, insert two fingers into the vagina to the cervix and lift the presenting part off of the cord to avoid compression D. Encourage the patient to push while the MD is in the room because immediate delivery is important E. Place patient in wheelchair to prepare for immediate delivery via C Section

A. Call for assistance, do not leave woman alone B. Administer oxygen via a nonrebreather mask at 8 to 10 L/min at least until birth is accomplished C. With a sterile gloved hand, insert two fingers into the vagina to the cervix and lift the presenting part off of the cord to avoid compression

The maternity nurse assessing a postpartum patient knows that the causes of postpartum hemorrhage include: SELECT ALL THAT APPLY (partial credit will not be given). A. Decreased uterine tone B. Excessive vomiting in the PP client C. Retained placental tissue D. Clotting

A. Decreased uterine tone C. Retained placental tissue D. Clotting

Because a full bladder prevents the uterus from contracting normally, nurses intervene to help the woman empty her bladder spontaneously as soon as possible. If all else fails, the last thing the nurse might try is: A. Inserting a sterile catheter B. Pouring water from a squeeze bottle over the woman's perineum C. Placing oil of peppermint in a bedpan under the woman D. Asking the physician to prescribe analgesics

A. Inserting a sterile catheter

A woman gave birth to a healthy infant boy 7 days ago. What type of lochia does the nurse expect to find when assessing this woman? A. Lochia serosa B. Lochia sangra C. Lochia rubra D. Lochia alba

A. Lochia serosa

The labor and delivery nurse is assisting a newborn post delivery and assigning an APGAR score. The nurse knows that the APGAR scoring includes: SELECT ALL THAT APPLY( partial credit will not be given). A. Muscle tone B. Skin Color C. Respiratory rate D. Heart rate E. Weight

A. Muscle tone B. Skin Color C. Respiratory rate D. Heart rate

Medications used to manage postpartum hemorrhage (PPH) include SELECT ALL THAT APPLY ( partial credit will not be given). A. Oxytocin B. Cytotec C. Methergine D. Terbutaline

A. Oxytocin B. Cytotec C. Methergine

With regard to the adaptation of other family members (mainly siblings and grandparents) to the newborn, nurses should be aware that: A. Participation in preparation classes helps both siblings and grandparents B. Since 1990, the number of grandparents providing permanent care to their grandchildren has been declining C. In the United States, paternal and maternal grandparents consider themselves as equally as important as the newborn's parents D. Sibling rivalry cannot be dismissed as overblown psychobabble; negative feelings and behaviors can take a long time to blow over

A. Participation in preparation classes helps both siblings and grandparents

A primary nursing responsibility when caring for a woman experiencing an obstetric hemorrhage associated with uterine atony is to: A. Perform fundal massage B. Prepare the woman for surgical intervention C. Catheterize the bladder D. Establish venous access

A. Perform fundal massage

Prostaglandin gel has been ordered for a pregnant woman at 43 weeks of gestation. The nurse recognizes that this medication is administered to: A. Ripen the cervix in preparation for labor induction B. Stimulate the amniotic membranes to rupture C. Enhance uteroplacental perfusion in an aging placenta D. Increase amniotic fluid volume

A. Ripen the cervix in preparation for labor induction

The laboratory results for a postpartum woman are as follows: blood type, A; Rh status, positive; rubella titer, 1:8 (EIA 0.8); hematocrit, 30%. How would the nurse best interpret these data? A. Rubella vaccine should be given B. A blood transfusion is necessary C. Rh immune globulin is necessary within 72 hours of birth D. A Kleihauer-Betke test should be performed

A. Rubella vaccine should be given

A first-time father is changing the diaper of his 1-day-old daughter. He asks the nurse, "What is this black, sticky stuff in her diaper?" The nurse's best response is: A. That's meconium, which is your baby's first stool. It's normal." B. "Oh, don't worry about that. It's okay." C. "That's transitional stool." D. "That means your baby is bleeding internally."

A. That's meconium, which is your baby's first stool. It's normal."

Early this morning an infant boy was circumcised using the PlastiBell method. The nurse tells the mother that she and the infant can be discharged after: A. The infant voids B. The bleeding stops completely C. The PlastiBell rim falls off D. Yellow exudate forms over the glans

A. The infant voids

The perinatal nurse assisting with establishing lactation is aware that acute mastitis can be minimized by: A. Using proper breastfeeding techniques B. Washing the nipples and breasts with mild soap and water once a day C. Wearing a supportive bra 24 hours a day D. Wearing a nipple shield for the first few days of breastfeeding

A. Using proper breastfeeding techniques

A perinatal nurse is caring for a woman in the immediate postbirth period. Assessment reveals that the woman is experiencing profuse bleeding and a boggy uterus. The most likely etiology for the bleeding is: A. Uterine atony B. Uterine inversion C. Vaginal hematoma D. Vaginal laceration

A. Uterine atony

During delivery a patient experiences dystocia. As the nurses assist to help with the delivery of the baby by applying abdominal pressure, the nurse knows that fetal causes of dystocia may inlcude: SELECT ALL THAT APPLY (partial credit will not be given) A. malpresentation B. Hyptotonic uterine activity C. CPD D. malposition E. Hypertonic uterine activity

A. malpresentation C. CPD D. malposition

A 63-year-old patient who began experiencing right arm and leg weakness is admitted to the emergency department. In which order will the nurse implement these actions included in the stroke protocol? (Put a comma and a space between each answer choice [A, B, C, D].) a. Obtain computed tomography (CT) scan without contrast. b. Infuse tissue plasminogen activator (tPA). c. Administer oxygen to keep O2 saturation >95%. d. Use National Institute of Health Stroke Scale to assess patient.

ANS: C, D, A, B The initial actions should be those that help with airway, breathing, and circulation. Baseline neurologic assessments should be done next. A CT scan will be needed to rule out hemorrhagic stroke before tPA can be administered.

In which order will the nurse perform the following actions when caring for a patient with possible C5 spinal cord trauma who is admitted to the emergency department? (Put a comma and a space between each answer choice [A, B, C, D, E].) a. Infuse normal saline at 150 mL/hr. b. Monitor cardiac rhythm and blood pressure. c. Administer O2 using a nonrebreather mask. d. Immobilize the patient's head, neck, and spine. e. Transfer the patient to radiology for spinal computed tomography (CT).

ANS: D, C, B, A, E The first action should be to prevent further injury by stabilizing the patient's spinal cord if the patient does not have penetrating trauma. Maintenance of oxygenation by administration of 100% O2 is the second priority. Because neurogenic shock is a possible complication, monitoring of heart rhythm and BP are indicated followed by infusing normal saline for volume replacement. A CT scan to determine the extent and level of injury is needed once initial assessment and stabilization are accomplished.

The health care provider orders the following interventions for a 67-kg patient who has septic shock with a blood pressure of 70/42 mm Hg and O2 saturation of 90% on room air. In which order will the nurse implement the actions? (Put a comma and a space between each answer choice [A, B, C, D, E].) a. Give vancomycin 1 g IV. b. Obtain blood and urine cultures c. Start norepinephrine 0.5 mcg/min. d. Infuse normal saline 2000 mL over 30 minutes. e. Titrate oxygen administration to keep O2 saturation above 95%.

ANS: E, D, C, B, A The initial action for this hypotensive and hypoxemic patient should be to improve the O2 saturation, followed by infusion of IV fluids and vasopressors to improve perfusion. Cultures should be obtained before giving antibiotics.

The nurse estimates the extent of a burn using the rule of nines for a patient who has been admitted with deep partial-thickness burns of the anterior trunk and the entire left arm. What percentage of the patient's total body surface area (TBSA) has been injured?

ANS: 27% When using the rule of nines, the anterior trunk is considered to cover 18% of the patient's body and each arm is 9%.

A patient's vital signs are pulse 87, respirations 24, and BP of 128/64 mm Hg and cardiac output is 4.7 L/min. The patient's stroke volume is _____ mL. (Round to the nearest whole number.)

ANS: 54 Stroke volume = cardiac output/heart rate

An 80-kg patient with burns over 30% of total body surface area (TBSA) is admitted to the burn unit. Using the Parkland formula of 4 mL/kg/%TBSA, what is the IV infusion rate (mL/hour) for lactated Ringer's solution that the nurse will administer during the first 8 hours?

ANS: 600 mL The Parkland formula states that patients should receive 4 mL/kg/%TBSA burned during the first 24 hours. Half of the total volume is given in the first 8 hours and then the last half is given over 16 hours: 4 ´ 80 ´ 30 = 9600 mL total volume; 9600/2 = 4800 mL in the first 8 hours; 4800 mL/8 hr = 600 mL/hr.

1. A patient in the oliguric phase after an acute kidney injury has had a 250-mL urine output and an emesis of 100 mL in the past 24 hours. What is the patient's fluid restriction for the next 24 hours?

ANS: 950 mL The general rule for calculating fluid restrictions is to add all fluid losses for the previous 24 hours, plus 600 mL for insensible losses: (250 + 100 + 600 = 950 mL). DIF: Cognitive Level: Understand (comprehension) REF: 1073 TOP: Nursing Process: Implementation MSC: NCLEX: Physiological Integrity

The nurse is caring for a patient who has an intraortic balloon pump (IABP) following a massive heart attack. When assessing the patient, the nurse notices blood backing up into the IABP catheter. In which order should the nurse take the following actions? (Put a comma and a space between each answer choice [A, B, C, D].) a. Ensure that the IABP console has turned off. b. Assess the patient's vital signs and orientation. c. Obtain supplies for insertion of a new IABP catheter. d. Notify the health care provider of the IABP malfunction.

ANS: A, B, D, C Blood in the IABP catheter indicates a possible tear in the balloon. The console will shut off automatically to prevent complications such as air embolism. Next, the nurse will assess the patient and communicate with the health care provider about the patient's assessment and the IABP problem. Finally, supplies for insertion of a new IABP catheter may be needed, based on the patient assessment and the decision of the health care provider.

1. In which order will the nurse take the following actions when caring for a patient who develops watery diarrhea and a fever after prolonged omeprazole (Prilosec) therapy? (Put a comma and a space between each answer choice [A, B, C, D].) a. Contact the health care provider. b. Assess blood pressure and heart rate. c. Give the PRN acetaminophen (Tylenol). d. Place the patient on contact precautions.

ANS: D, B, A, C Proton pump inhibitors including omeprazole (Prilosec) may increase the risk of Clostridium difficile-associated colitis. Because the patient's history and symptoms are consistent with C. difficile infection, the initial action should be initiation of infection control measures to protect other patients. Assessment of blood pressure and pulse is needed to determine whether the patient has symptoms of hypovolemia and/or shock. The health care provider should be notified so that actions such as obtaining stool specimens and antibiotic therapy can be started. Tylenol may be administered, but is the lowest priority of the actions.

In which order will the nurse take these actions when doing a dressing change for a partial-thickness burn wound on a patient's chest? (Put a comma and a space between each answer choice [A, B, C, D, E].) a. Apply sterile gauze dressing. b. Document wound appearance. c. Apply silver sulfadiazine cream. d. Administer IV fentanyl (Sublimaze). e. Clean wound with saline-soaked gauze.

ANS: D, E, C, A, B Because partial-thickness burns are very painful, the nurse's first action should be to administer pain medications. The wound will then be cleaned, antibacterial cream applied, and covered with a new sterile dressing. The last action should be to document the appearance of the wound

When assisting with oral intubation of a patient who is having respiratory distress, in which order will the nurse take these actions? (Put a comma and a space between each answer choice [A, B, C, D, E].) a. Obtain a portable chest-x-ray. b. Position the patient in the supine position. c. Inflate the cuff of the endotracheal tube after insertion. d. Attach an end-tidal CO2 detector to the endotracheal tube. e. Oxygenate the patient with a bag-valve-mask device for several minutes.

ANS: E, B, C, D, A The patient is pre-oxygenated with a bag-valve-mask system for 3 to 5 minutes before intubation and then placed in a supine position. Following the intubation, the cuff on the endotracheal tube is inflated to occlude and protect the airway. Tube placement is assessed first with an end-tidal CO2 sensor, then with a chest x-ray.

Which condition is considered a medical emergency that requires immediate treatment? a.Inversion of the uterus b.Hypotonic uterus c.ITP d.Uterine atony

ANS: A Inversion of the uterus is likely to lead to hypovolemic shock and therefore is considered a medical emergency. Although hypotonic uterus, ITP, and uterine atony are serious conditions, they are not necessarily medical emergencies that require immediate treatment.

What is the most common reason for late postpartum hemorrhage (PPH)? a.Subinvolution of the uterus b.Defective vascularity of the decidua c.Cervical lacerations d.Coagulation disorders

ANS: A Late PPH may be the result of subinvolution of the uterus. Recognized causes of subinvolution include retained placental fragments and pelvic infection. Although defective vascularity, cervical lacerations, and coagulation disorders of the decidua may also cause PPH, late PPH typically results from subinvolution of the uterus, pelvic infection, or retained placental fragments.

A perinatal nurse is caring for a woman in the immediate postbirth period. Assessment reveals that the client is experiencing profuse bleeding. What is the most likely cause for this bleeding? a.Uterine atony b.Uterine inversion c.Vaginal hematoma d.Vaginal laceration

ANS: A Uterine atony is significant hypotonia of the uterus and is the leading cause of postpartum hemorrhage. Uterine inversion may lead to hemorrhage; however, it is not the most likely source of this client's bleeding. Further, if the woman were experiencing a uterine inversion, it would be evidenced by the presence of a large, red, rounded mass protruding from the introitus. A vaginal hematoma may be associated with hemorrhage. However, the most likely clinical finding for vaginal hematoma is pain, not the presence of profuse bleeding. A vaginal laceration should be suspected if vaginal bleeding continues in the presence of a firm, contracted uterine fundus.

51. Which menu choice by the patient with diverticulosis is best for preventing diverticulitis? a. Navy bean soup and vegetable salad b. Whole grain pasta with tomato sauce c. Baked potato with low-fat sour cream d. Roast beef sandwich on whole wheat bread

ANS: A A diet high in fiber and low in fats and red meat is recommended to prevent diverticulitis. Although all of the choices have some fiber, the bean soup and salad will be the highest in fiber and the lowest in fat.

34. A nurse is caring for a hospitalized patient with a urinary catheter. Which nursing action best prevents the patient from acquiring an infection? a. Maintaining a closed urinary drainage system b. Inserting the catheter using strict clean technique c. Disconnecting and replacing the catheter drainage bag once per shift d. Fully inflating the catheter's balloon according to the manufacturer's recommendation

ANS: A A key intervention to prevent infection is maintaining a closed urinary drainage system. A catheter should be inserted in the hospital setting using sterile technique. Inflating the balloon fully prevents dislodgment and trauma, not infection. Disconnecting the drainage bag from the catheter creates a break in the system and an open portal of entry and increases risk of infection.

26. The nurse admitting a patient with acute diverticulitis explains that the initial plan of care is to a. administer IV fluids. b. give stool softeners and enemas. c. order a diet high in fiber and fluids. d. prepare the patient for colonoscopy.

ANS: A A patient with acute diverticulitis will be NPO and given parenteral fluids. A diet high in fiber and fluids will be implemented before discharge. Bulk-forming laxatives, rather than stool softeners, are usually given, and these will be implemented later in the hospitalization. The patient with acute diverticulitis will not have enemas or a colonoscopy because of the risk for perforation and peritonitis.

A patient arrives in the emergency department with facial and chest burns caused by a house fire. Which action should the nurse take first? a. Auscultate the patient's lung sounds. b. Determine the extent and depth of the burns. c. Infuse the ordered lactated Ringer's solution. d. Administer the ordered hydromorphone (Dilaudid).

ANS: A A patient with facial and chest burns is at risk for inhalation injury, and assessment of airway and breathing is the priority. The other actions will be completed after airway management is assured

30. A 50-year-old female patient calls the clinic to report a new onset of severe diarrhea. The nurse anticipates that the patient will need to a. collect a stool specimen. b. prepare for colonoscopy. c. schedule a barium enema. d. have blood cultures drawn.

ANS: A Acute diarrhea is usually caused by an infectious process, and stool specimens are obtained for culture and examined for parasites or white blood cells. There is no indication that the patient needs a colonoscopy, blood cultures, or a barium enema.

25. A 34-year-old female patient with a new ileostomy asks how much drainage to expect. The nurse explains that after the bowel adjusts to the ileostomy, the usual drainage will be about _____ cups. a. 2 b. 3 c. 4 d. 5

ANS: A After the proximal small bowel adapts to reabsorb more fluid, the average amount of ileostomy drainage is about 500 mL daily. One cup is about 240 mL.

5. A patient complains of gas pains and abdominal distention two days after a small bowel resection. Which nursing action is best to take? a. Encourage the patient to ambulate. b. Instill a mineral oil retention enema. c. Administer the ordered IV morphine sulfate. d. Offer the ordered promethazine (Phenergan) suppository.

ANS: A Ambulation will improve peristalsis and help the patient eliminate flatus and reduce gas pain. A mineral oil retention enema is helpful for constipation with hard stool. A return-flow enema might be used to relieve persistent gas pains. Morphine will further reduce peristalsis. Promethazine (Phenergan) is used as an antiemetic rather than to decrease gas pains or distention.

Which patient is most appropriate for the burn unit charge nurse to assign to a registered nurse (RN) who has floated from the hospital medical unit? a. A 34-year-old patient who has a weight loss of 15% from admission and requires enteral feedings. b. A 67-year-old patient who has blebs under an autograft on the thigh and has an order for bleb aspiration c. A 46-year-old patient who has just come back to the unit after having a cultured epithelial autograft to the chest d. A 65-year-old patient who has twice-daily burn debridements and dressing changes to partial-thickness facial burns

ANS: A An RN from a medical unit would be familiar with malnutrition and with administration and evaluation of response to enteral feedings. The other patients require burn assessment and care that is more appropriate for staff who regularly care for burned patients

11. A patient will need vascular access for hemodialysis. Which statement by the nurse accurately describes an advantage of a fistula over a graft? a. A fistula is much less likely to clot. b. A fistula increases patient mobility. c. A fistula can accommodate larger needles. d. A fistula can be used sooner after surgery.

ANS: A Arteriovenous (AV) fistulas are much less likely to clot than grafts, although it takes longer for them to mature to the point where they can be used for dialysis. The choice of an AV fistula or a graft does not have an impact on needle size or patient mobility. DIF: Cognitive Level: Understand (comprehension) REF: 1088 TOP: Nursing Process: Implementation MSC: NCLEX: Physiological Integrity

4. A patient requests the nurse's help to the bedside commode and becomes frustrated when unable to void in front of the nurse. How should the nurse interpret the patient's inability to void? a. The patient can be anxious, making it difficult for abdominal and perineal muscles to relax enough to void. b. The patient does not recognize the physiological signals that indicate a need to void. c. The patient is lonely, and calling the nurse in under false pretenses is a way to get attention. d. The patient is not drinking enough fluids to produce adequate urine output.

ANS: A Attempting to void in the presence of another can cause anxiety and tension in the muscles that make voiding difficult. Anxiety can impact bladder emptying due to inadequate relaxation of the pelvic floor muscles and urinary sphincter. The nurse should give the patient privacy and adequate time if appropriate. No evidence suggests that an underlying physiological (does not recognize signals or not drinking enough fluids) or psychological (lonely) condition exists.

35. A nurse is providing care to a patient with an indwelling catheter. Which practice indicates the nurse is following guidelines for avoiding catheter-associated urinary tract infection (CAUTI)? a. Drapes the urinary drainage tubing with no dependent loops b. Washes the drainage tube toward the meatus with soap and water c. Places the urinary drainage bag gently on the floor below the patient d. Allows the spigot to touch the receptacle when emptying the drainage bag

ANS: A Avoid dependent loops in urinary drainage tubing. Prevent the urinary drainage bag from touching or dragging on the floor. When emptying the urinary drainage bag, use a separate measuring receptacle for each patient. Do not let the drainage spigot touch the receptacle. Using a clean washcloth, soap, and water, with your dominant hand wipe in a circular motion along the length of the catheter for about 10 cm (4 inches), starting at the meatus and moving away.

A patient with septic shock has a BP of 70/46 mm Hg, pulse of 136 beats/min, respirations of 32 breaths/min, temperature of 104°F, and blood glucose of 246 mg/dL. Which intervention ordered by the health care provider should the nurse implement first? a. Give normal saline IV at 500 mL/hr. b. Give acetaminophen (Tylenol) 650 mg rectally. c. Start insulin drip to maintain blood glucose at 110 to 150 mg/dL. d. Start norepinephrine to keep systolic blood pressure above 90 mm Hg.

ANS: A Because of the decreased preload associated with septic shock, fluid resuscitation is the initial therapy. The other actions also are appropriate, and should be initiated quickly as well.

The nurse is caring for a patient who has septic shock. Which assessment finding is most important for the nurse to report to the health care provider? a. Skin cool and clammy c. Blood pressure of 92/56 mm Hg b. Heart rate of 118 beats/min d. O2 saturation of 93% on room air

ANS: A Because patients in the early stage of septic shock have warm and dry skin, the patient's cool and clammy skin indicates that shock is progressing. The other information will also be reported, but does not indicate deterioration of the patient's status.

8. Which nursing action will be included in the plan of care for a 27-year-old male patient with bowel irregularity and a new diagnosis of irritable bowel syndrome (IBS)? a. Encourage the patient to express concerns and ask questions about IBS. b. Suggest that the patient increase the intake of milk and other dairy products. c. Educate the patient about the use of alosetron (Lotronex) to reduce symptoms. d. Teach the patient to avoid using nonsteroidal antiinflammatory drugs (NSAIDs).

ANS: A Because psychologic and emotional factors can affect the symptoms for IBS, encouraging the patient to discuss emotions and ask questions is an important intervention. Alosetron has serious side effects, and is used only for female patients who have not responded to other therapies. Although yogurt may be beneficial, milk is avoided because lactose intolerance can contribute to symptoms in some patients. NSAIDs can be used by patients with IBS.

A female patient who had a stroke 24 hours ago has expressive aphasia. The nurse identifies the nursing diagnosis of impaired verbal communication. An appropriate nursing intervention to help the patient communicate is to a. ask questions that the patient can answer with "yes" or "no." b. develop a list of words that the patient can read and practice reciting. c. have the patient practice her facial and tongue exercises with a mirror. d. prevent embarrassing the patient by answering for her if she does not respond.

ANS: A Communication will be facilitated and less frustrating to the patient when questions that require a "yes" or "no" response are used. When the language areas of the brain are injured, the patient might not be able to read or recite words, which will frustrate the patient without improving communication. Expressive aphasia is caused by damage to the language areas of the brain, not by the areas that control the motor aspects of speech. The nurse should allow time for the patient to respond.

A patient has just been admitted with a 40% total body surface area (TBSA) burn injury. To maintain adequate nutrition, the nurse should plan to take which action? a. Insert a feeding tube and initiate enteral feedings. b. Infuse total parenteral nutrition via a central catheter. c. Encourage an oral intake of at least 5000 kcal per day. d. Administer multiple vitamins and minerals in the IV solution.

ANS: A Enteral feedings can usually be initiated during the emergent phase at low rates and increased over 24 to 48 hours to the goal rate. During the emergent phase, the patient will be unable to eat enough calories to meet nutritional needs and may have a paralytic ileus that prevents adequate nutrient absorption. Vitamins and minerals may be administered during the emergent phase, but these will not assist in meeting the patient's caloric needs. Parenteral nutrition increases the infection risk, does not help preserve gastrointestinal function, and is not routinely used in burn patients

The following interventions are ordered by the health care provider for a patient who has respiratory distress and syncope after eating strawberries. Which will the nurse complete first? a. Give epinephrine. b. Administer diphenhydramine. c. Start continuous ECG monitoring. d. Draw blood for complete blood count (CBC)

ANS: A Epinephrine rapidly causes peripheral vasoconstriction, dilates the bronchi, and blocks the effects of histamine and reverses the vasodilation, bronchoconstriction, and histamine release that cause the symptoms of anaphylaxis. The other interventions are also appropriate but would not be the first ones completed.

22. A patient has severe flank pain. The urinalysis reveals presence of calcium phosphate crystals. The nurse will anticipate an order for which diagnostic test? a. Intravenous pyelogram b. Mid-stream urinalysis c. Bladder scan d. Cystoscopy

ANS: A Flank pain and calcium phosphate crystals are associated with renal calculi. An intravenous pyelogram allows the provider to observe pathological problems such as obstruction of the ureter. A mid-stream urinalysis is performed for a routine urinalysis or if an infection is suspected; a urinalysis was already performed, a mid-stream would not be obtained again. A cystoscopy is used to detect bladder tumors and obstruction of the bladder outlet and urethra. A bladder scan measures the amount of urine in the bladder.

A 78-kg patient with septic shock has a pulse rate of 120 beats/min with low central venous pressure and pulmonary artery wedge pressure. Urine output has been 30 mL/hr for the past 3 hours. Which order by the health care provider should the nurse question? a. Administer furosemide (Lasix) 40 mg IV. b. Increase normal saline infusion to 250 mL/hr. c. Give hydrocortisone (Solu-Cortef) 100 mg IV. d. Titrate norepinephrine to keep systolic blood pressure (BP) above 90 mm Hg.

ANS: A Furosemide will lower the filling pressures and renal perfusion further for the patient with septic shock. Patients in septic shock require large amounts of fluid replacement. If the patient remains hypotensive after initial volume resuscitation with minimally 30 mL/kg, vasopressors such as norepinephrine may be added. IV corticosteroids may be considered for patients in septic shock who cannot maintain an adequate BP with vasopressor therapy despite fluid resuscitation.

A patient with carotid atherosclerosis asks the nurse to describe a carotid endarterectomy. Which response by the nurse is accurate? a. "The obstructing plaque is surgically removed from an artery in the neck." b. "The diseased portion of the artery in the brain is replaced with a synthetic graft." c. "A wire is threaded through an artery in the leg to the clots in the carotid artery and the clots are removed." d. "A catheter with a deflated balloon is positioned at the narrow area, and the balloon is inflated to flatten the plaque."

ANS: A In a carotid endarterectomy, the carotid artery is incised and the plaque is removed. The response beginning, "The diseased portion of the artery in the brain is replaced" describes an arterial graft procedure. The answer beginning, "A catheter with a deflated balloon is positioned at the narrow area" describes an angioplasty. The final response beginning, "A wire is threaded through the artery" describes the mechanical embolus removal in cerebral ischemia (MERCI) procedure.

32. A nurse is caring for a patient with a continent urinary reservoir. Which action will the nurse take? a. Teach the patient how to self-cath the pouch. b. Teach the patient how to perform Kegel exercises. c. Teach the patient how to change the collection pouch. d. Teach the patient how to void using the Valsalva technique.

ANS: A In a continent urinary reservoir, the ileocecal valve creates a one-way valve in the pouch through which a catheter is inserted through the stoma to empty the urine from the pouch. Patients must be willing and able to catheterize the pouch 4 to 6 times a day for the rest of their lives. The second type of continent urinary diversion is called an orthotopic neobladder, which uses an ileal pouch to replace the bladder. Anatomically, the pouch is in the same position as the bladder was before removal, allowing a patient to void through the urethra using a Valsalva technique. In a ureterostomy or ileal conduit the patient has no sensation or control over the continuous flow of urine through the ileal conduit, requiring the effluent (drainage) to be collected in a pouch. Kegel exercises are ineffective for a patient with a continent urinary reservoir.

15. A 51-year-old male patient has a new diagnosis of Crohn's disease after having frequent diarrhea and a weight loss of 10 pounds (4.5 kg) over 2 months. The nurse will plan to teach about a. medication use. b. fluid restriction. c. enteral nutrition. d. activity restrictions.

ANS: A Medications are used to induce and maintain remission in patients with inflammatory bowel disease (IBD). Decreased activity level is indicated only if the patient has severe fatigue and weakness. Fluids are needed to prevent dehydration. There is no advantage to enteral feedings.

A patient who is receiving positive pressure ventilation is scheduled for a spontaneous breathing trial (SBT). Which finding by the nurse is most important to discuss with the health care provider before starting the SBT? a. New ST segment elevation is noted on the cardiac monitor. b. Enteral feedings are being given through an orogastric tube. c. Scattered rhonchi are heard when auscultating breath sounds. d. HYDROmorphone (Dilaudid) is being used to treat postoperative pain.

ANS: A Myocardial ischemia is a contraindication for ventilator weaning. The ST segment elevation is an indication that weaning should be postponed until further investigation and/or treatment for myocardial ischemia can be done. The other information will also be shared with the health care provider, but ventilator weaning can proceed when opioids are used for pain management, abnormal lung sounds are present, or enteral feedings are being used.

3. A patient is experiencing oliguria. Which action should the nurse perform first? a. Assess for bladder distention. b. Request an order for diuretics. c. Increase the patient's intravenous fluid rate. d. Encourage the patient to drink caffeinated beverages.

ANS: A Oliguria is diminished urinary output in relation to fluid intake. The nurse first should gather all assessment data to determine the potential cause of oliguria. It could be that the patient does not have adequate intake, or it could be that the bladder sphincter is not functioning and the patient is retaining water. Increasing fluids is effective if the patient does not have adequate intake or if dehydration occurs. Caffeine can work as a diuretic but is not helpful if an underlying pathology is present. An order for diuretics can be obtained if the patient was retaining water, but this should not be the first action.

14. A patient asks about treatment for stress urinary incontinence. Which is the nurse's bestresponse? a. Perform pelvic floor exercises. b. Avoid voiding frequently. c. Drink cranberry juice. d. Wear an adult diaper.

ANS: A Poor muscle tone leads to an inability to control urine flow. The nurse should recommend pelvic muscle strengthening exercises such as Kegel exercises; this solution best addresses the patient's problem. Evidence has shown that patients with urgency, stress, and mixed urinary incontinence can eventually achieve continence when treated with pelvic floor muscle training. Drinking cranberry juice is a preventative measure for urinary tract infection. The nurse should not encourage the patient to reduce voiding; residual urine in the bladder increases the risk of infection. Wearing an adult diaper could be considered if attempts to correct the root of the problem fail.

8. A patient has fallen several times in the past week when attempting to get to the bathroom. The patient gets up 3 or 4 times a night to urinate. Which recommendation by the nurse is most appropriate in correcting this urinary problem? a. Limit fluid and caffeine intake before bed. b. Leave the bathroom light on to illuminate a pathway. c. Practice Kegel exercises to strengthen bladder muscles. d. Clear the path to the bathroom of all obstacles before bedtime.

ANS: A Reducing fluids, especially caffeine and alcohol, before bedtime can reduce nocturia. To prevent nocturia, suggest that the patient avoid drinking fluids 2 hours before bedtime. Clearing a path to the bathroom, illuminating the path, or shortening the distance to the bathroom may reduce falls but will not correct the urination problem. Kegel exercises are useful if a patient is experiencing stress incontinence.

45. A 51-year-old woman with Crohn's disease who is taking infliximab (Remicade) calls the nurse in the outpatient clinic about new symptoms. Which symptom is most important to communicate to the health care provider? a. Fever b. Nausea c. Joint pain d. Headache

ANS: A Since infliximab suppresses the immune response, rapid treatment of infection is essential. The other patient complaints are common side effects of the medication, but they do not indicate any potentially life-threatening complications.

7. Sodium polystyrene sulfonate (Kayexalate) is ordered for a patient with hyperkalemia. Before administering the medication, the nurse should assess the a. bowel sounds. b. blood glucose. c. blood urea nitrogen (BUN). d. level of consciousness (LOC).

ANS: A Sodium polystyrene sulfonate (Kayexalate) should not be given to a patient with a paralytic ileus (as indicated by absent bowel sounds) because bowel necrosis can occur. The BUN and creatinine, blood glucose, and LOC would not affect the nurse's decision to give the medication. DIF: Cognitive Level: Apply (application) REF: 1080 TOP: Nursing Process: Assessment MSC: NCLEX: Physiological Integrity

The charge nurse observes the following actions being taken by a new nurse on the burn unit. Which action by the new nurse would require an intervention by the charge nurse? a. The new nurse uses clean latex gloves when applying antibacterial cream to a burn wound. b. The new nurse obtains burn cultures when the patient has a temperature of 95.2° F (35.1° C). c. The new nurse administers PRN fentanyl (Sublimaze) IV to a patient 5 minutes before a dressing change. d. The new nurse calls the health care provider for a possible insulin order when a nondiabetic patient's serum glucose is elevated.

ANS: A Sterile gloves should be worn when applying medications or dressings to a burn. Hypothermia is an indicator of possible sepsis, and cultures are appropriate. Nondiabetic patients may require insulin because stress and high calorie intake may lead to temporary hyperglycemia. Fentanyl peaks 5 minutes after IV administration, and should be used just before and during dressing changes for pain management

Which assessment finding obtained by the nurse when caring for a patient with a right radial arterial line indicates a need for the nurse to take immediate action? a. The right hand is cooler than the left hand. b. The mean arterial pressure (MAP) is 77 mm Hg. c. The system is delivering 3 mL of flush solution per hour. d. The flush bag and tubing were last changed 3 days previously.

ANS: A The change in temperature of the left hand suggests that blood flow to the left hand is impaired. The flush system needs to be changed every 96 hours. A mean arterial pressure (MAP) of 75 mm Hg is normal. Flush systems for hemodynamic monitoring are set up to deliver 3 to 6 mL/hour of flush solution.

During change-of-shift report, the nurse is told that a patient has been admitted with dehydration and hypotension after having vomiting and diarrhea for 4 days. Which finding is most important for the nurse to report to the health care provider? a. New onset of confusion c. Heart rate 112 beats/min b. Decreased bowel sounds d. Pale, cool, and dry extremities

ANS: A The changes in mental status are indicative that the patient is in the progressive stage of shock and that rapid intervention is needed to prevent further deterioration. The other information is consistent with compensatory shock.

A patient with circumferential burns of both legs develops a decrease in dorsalis pedis pulse strength and numbness in the toes. Which action should the nurse take? a. Notify the health care provider. b. Monitor the pulses every 2 hours. c. Elevate both legs above heart level with pillows. d. Encourage the patient to flex and extend the toes on both feet.

ANS: A The decrease in pulse in a patient with circumferential burns indicates decreased circulation to the legs and the need for an escharotomy. Monitoring the pulses is not an adequate response to the decrease in circulation. Elevating the legs or increasing toe movement will not improve the patient's circulation

Which data collected by the nurse caring for a patient who has cardiogenic shock indicate that the patient may be developing multiple organ dysfunction syndrome (MODS)? a. The patient's serum creatinine level is elevated. b. The patient complains of intermittent chest pressure. c. The patient's extremities are cool and pulses are weak. d. The patient has bilateral crackles throughout lung fields.

ANS: A The elevated serum creatinine level indicates that the patient has renal failure as well as heart failure. The crackles, chest pressure, and cool extremities are all symptoms consistent with the patient's diagnosis of cardiogenic shock.

A patient with respiratory failure has arterial pressure-based cardiac output (APCO) monitoring and is receiving mechanical ventilation with peak end-expiratory pressure (PEEP) of 12 cm H2O. Which information indicates that a change in the ventilator settings may be required? a. The arterial pressure is 90/46. b. The heart rate is 58 beats/minute. c. The stroke volume is increased. d. The stroke volume variation is 12%.

ANS: A The hypotension suggests that the high intrathoracic pressure caused by the PEEP may be decreasing venous return and (potentially) cardiac output. The other assessment data would not be a direct result of PEEP and mechanical ventilation.

A patient who is orally intubated and receiving mechanical ventilation is anxious and is "fighting" the ventilator. Which action should the nurse take next? a. Verbally coach the patient to breathe with the ventilator. b. Sedate the patient with the ordered PRN lorazepam (Ativan). c. Manually ventilate the patient with a bag-valve-mask device. d. Increase the rate for the ordered propofol (Diprivan) infusion.

ANS: A The initial response by the nurse should be to try to decrease the patient's anxiety by coaching the patient about how to coordinate respirations with the ventilator. The other actions may also be helpful if the verbal coaching is ineffective in reducing the patient's anxiety.

A 40-year-old patient has a ruptured cerebral aneurysm and subarachnoid hemorrhage. Which intervention will be included in the care plan? a. Apply intermittent pneumatic compression stockings. b. Assist to dangle on edge of bed and assess for dizziness. c. Encourage patient to cough and deep breathe every 4 hours. d. Insert an oropharyngeal airway to prevent airway obstruction.

ANS: A The patient with a subarachnoid hemorrhage usually has minimal activity to prevent cerebral vasospasm or further bleeding and is at risk for venous thromboembolism (VTE). Activities such as coughing and sitting up that might increase intracranial pressure (ICP) or decrease cerebral blood flow are avoided. Because there is no indication that the patient is unconscious, an oropharyngeal airway is inappropriate.

For a patient who had a right hemisphere stroke the nurse establishes a nursing diagnosis of a. risk for injury related to denial of deficits and impulsiveness. b. impaired physical mobility related to right-sided hemiplegia. c. impaired verbal communication related to speech-language deficits. d. ineffective coping related to depression and distress about disability

ANS: A The patient with right-sided brain damage typically denies any deficits and has poor impulse control, leading to risk for injury when the patient attempts activities such as transferring from a bed to a chair. Right-sided brain damage causes left hemiplegia. Left-sided brain damage typically causes language deficits. Left-sided brain damage is associated with depression and distress about the disability.

24. A 72-yr-old patient with a history of benign prostatic hyperplasia (BPH) is admitted with acute urinary retention and elevated blood urea nitrogen (BUN) and creatinine levels. Which prescribed therapy should the nurse implement first? a. Insert urethral catheter. b. Obtain renal ultrasound. c. Draw a complete blood count. d. Infuse normal saline at 50 mL/hour.

ANS: A The patient's elevation in BUN is most likely associated with hydronephrosis caused by the acute urinary retention, so the insertion of a retention catheter is the first action to prevent ongoing postrenal failure for this patient. The other actions also are appropriate but should be implemented after the retention catheter. DIF: Cognitive Level: Analyze (analysis) REF: 1071 OBJ: Special Questions: Prioritization TOP: Nursing Process: Implementation MSC: NCLEX: Physiological Integrity

The nurse caring for a patient admitted with burns over 30% of the body surface assesses that urine output has dramatically increased. Which action by the nurse would best ensure adequate kidney function? a. Continue to monitor the urine output. b. Monitor for increased white blood cells (WBCs). c. Assess that blisters and edema have subsided. d. Prepare the patient for discharge from the burn unit.

ANS: A The patient's urine output indicates that the patient is entering the acute phase of the burn injury and moving on from the emergent stage. At the end of the emergent phase, capillary permeability normalizes and the patient begins to diurese large amounts of urine with a low specific gravity. Although this may occur at about 48 hours, it may be longer in some patients. Blisters and edema begin to resolve, but this process requires more time. White blood cells may increase or decrease, based on the patient's immune status and any infectious processes. The WBC count does not indicate kidney function. The patient will likely remain in the burn unit during the acute stage of burn injury

12. When caring for a patient with a left arm arteriovenous fistula, which action will the nurse include in the plan of care to maintain the patency of the fistula? a. Auscultate for a bruit at the fistula site. b. Assess the quality of the left radial pulse. c. Compare blood pressures in the left and right arms. d. Irrigate the fistula site with saline every 8 to 12 hours.

ANS: A The presence of a thrill and bruit indicates adequate blood flow through the fistula. Pulse rate and quality are not good indicators of fistula patency. Blood pressures should never be obtained on the arm with a fistula. Irrigation of the fistula might damage the fistula, and typically only dialysis staff would access the fistula. DIF: Cognitive Level: Understand (comprehension) REF: 1087 TOP: Nursing Process: Planning MSC: NCLEX: Physiological Integrity

11. A nurse is planning care for a group of patients. Which task will the nurse assign to the nursing assistive personnel? a. Obtaining a midstream urine specimen b. Interpreting a bladder scan result c. Inserting a straight catheter d. Irrigating a catheter

ANS: A The skill of collecting midstream (clean-voided) urine specimens can be delegated to nursing assistive personnel. The nurse must first determine the timing and frequency of the bladder scan measurement and interprets the measurements obtained. Inserting a straight or an indwelling catheter cannot be delegated. Catheter irrigation or instillation cannot be delegated to nursing assistive personnel.

48. A female patient is awaiting surgery for acute peritonitis. Which action will the nurse include in the plan of care? a. Position patient with the knees flexed. b. Avoid use of opioids or sedative drugs. c. Offer frequent small sips of clear liquids. d. Assist patient to breathe deeply and cough.

ANS: A There is less peritoneal irritation with the knees flexed, which will help decrease pain. Opioids and sedatives are typically given to control pain and anxiety. Preoperative patients with peritonitis are given IV fluids for hydration. Deep breathing and coughing will increase the patient's discomfort.

Which patient should the nurse assess first? a. A patient with smoke inhalation who has wheezes and altered mental status b. A patient with full-thickness leg burns who has a dressing change scheduled c. A patient with abdominal burns who is complaining of level 8 (0 to 10 scale) pain d. A patient with 40% total body surface area (TBSA) burns who is receiving IV fluids at 500 mL/hour

ANS: A This patient has evidence of lower airway injury and hypoxemia and should be assessed immediately to determine the need for oxygen or intubation. The other patients should also be assessed as rapidly as possible, but they do not have evidence of life-threatening complications

37. To reduce patient discomfort during a closed intermittent catheter irrigation, what should the nurse do? a. Use room temperature irrigation solution. b. Administer the solution as quickly as possible. c. Allow the solution to sit in the bladder for at least 1 hour. d. Raise the bag of the irrigation solution at least 12 inches above the bladder.

ANS: A To reduce discomfort use room temperature solution. Using cold solutions and instilling solutions too quickly can cause discomfort. During an irrigation, the solution does not sit in the bladder; it is allowed to drain. A container is not raised about the bladder 12 inches when performing a closed intermittent catheter irrigation.

33. The nurse is preparing to apply an external catheter. Which action will the nurse take? a. Allow 1 to 2 inches of space between the tip of the penis and the end of the catheter. b. Spiral wrap the penile shaft using adhesive tape to secure the catheter. c. Twist the catheter before applying drainage tubing to the end of the catheter. d. Shave the pubic area before applying the catheter.

ANS: A When applying an external catheter, allow 2.5 to 5 cm (1 to 2 inches) of space between the tip of the penis and the end of the catheter. Spiral wrap the penile shaft with supplied elastic adhesive. The strip should not overlap. The elastic strip should be snug but not tight. NOTE: Never use adhesive tape. Connect drainage tubing to the end of the condom catheter. Be sure the condom is not twisted. Connect the catheter to a large-volume drainage bag or leg. Clip hair at the base of the penile shaft, as necessary. Do not shave the pubic area.

An employee spills industrial acids on both arms and legs at work. What is the priority action that the occupational health nurse at the facility should take? a. Remove nonadherent clothing and watch. b. Apply an alkaline solution to the affected area. c. Place cool compresses on the area of exposure. d. Cover the affected area with dry, sterile dressings.

ANS: A With chemical burns, the initial action is to remove the chemical from contact with the skin as quickly as possible. Remove nonadherent clothing, shoes, watches, jewelry, glasses, or contact lenses (if face was exposed). Flush chemical from wound and surrounding area with copious amounts of saline solution or water. Covering the affected area or placing cool compresses on the area will leave the chemical in contact with the skin. Application of an alkaline solution is not recommended

After receiving change-of-shift report on the following four patients, which patient should the nurse see first? a. A 60-year-old patient with right-sided weakness who has an infusion of tPA prescribed b. A 50-year-old patient who has atrial fibrillation and a new order for warfarin (Coumadin) c. A 40-year-old patient who experienced a transient ischemic attack yesterday who has a dose of aspirin due d. A 30-year-old patient with a subarachnoid hemorrhage 2 days ago who has nimodipine (Nimotop) scheduled

ANS: A tPA needs to be infused within the first few hours after stroke symptoms start in order to be effective in minimizing brain injury. The other medications should also be given as quickly as possible, but timing of the medications is not as critical.

Which preventive actions by the nurse will help limit the development of systemic inflammatory response syndrome (SIRS) in patients admitted to the hospital (select all that apply)? a. Ambulate postoperative patients as soon as possible after surgery. b. Use aseptic technique when manipulating invasive lines or devices. c. Remove indwelling urinary catheters as soon as possible after surgery. d. Administer prescribed antibiotics within 1 hour for patients with possible sepsis. e. Advocate for parenteral nutrition for patients who cannot take in adequate calories.

ANS: A, B, C, D Because sepsis is the most frequent etiology for SIRS, measures to avoid infection such as removing indwelling urinary catheters as soon as possible, use of aseptic technique, and early ambulation should be included in the plan of care. Adequate nutrition is important in preventing SIRS. Enteral, rather than parenteral, nutrition is preferred when patients are unable to take oral feedings because enteral nutrition helps maintain the integrity of the intestine, thus decreasing infection risk. Antibiotics should be given within 1 hour after being prescribed to decrease the risk of sepsis progressing to SIRS.

Which medications are used to manage PPH? (Select all that apply.) a.Oxytocin b.Methergine c.Terbutaline d.Hemabate e.Magnesium sulfate

ANS: A, B, D Oxytocin, Methergine, and Hemabate are medications used to manage PPH. Terbutaline and magnesium sulfate are tocolytic medications that are used to relax the uterus, which would cause or worsen PPH.

A patient with suspected neurogenic shock after a diving accident has arrived in the emergency department. A cervical collar is in place. Which actions should the nurse take (select all that apply)? a. Prepare to administer atropine IV. b. Obtain baseline body temperature. c. Infuse large volumes of lactated Ringer's solution. d. Provide high-flow O2 (100%) by nonrebreather mask. e. Prepare for emergent intubation and mechanical ventilation.

ANS: A, B, D, E All of the actions are appropriate except to give large volumes of lactated Ringer's solution. The patient with neurogenic shock usually has a normal blood volume, and it is important not to volume overload the patient. In addition, lactated Ringer's solution is used cautiously in all shock situations because an ischemic liver cannot convert lactate to bicarbonate.

1. Which information will be included when the nurse is teaching self-management to a patient who is receiving peritoneal dialysis (select all that apply)? a. Avoid commercial salt substitutes. b. Restrict fluid intake to 1000 mL daily. c. Take phosphate binders with each meal. d. Choose high-protein foods for most meals. e. Have several servings of dairy products daily.

ANS: A, C, D Patients who are receiving peritoneal dialysis should have a high-protein diet. Phosphate binders are taken with meals to help control serum phosphate and calcium levels. Commercial salt substitutes are high in potassium and should be avoided. Fluid intake is not limited unless weight and blood pressure are not controlled. Dairy products are high in phosphate and usually are limited. DIF: Cognitive Level: Apply (application) REF: 1087 TOP: Nursing Process: Planning MSC: NCLEX: Physiological Integrity

Lacerations of the cervix, vagina, or perineum are also causes of PPH. Which factors influence the causes and incidence of obstetric lacerations of the lower genital tract? (Select all that apply.) a.Operative and precipitate births b.Adherent retained placenta c.Abnormal presentation of the fetus d.Congenital abnormalities of the maternal soft tissue e.Previous scarring from infection

ANS: A, C, D, E Abnormal adherence of the placenta occurs for unknown reasons. Attempts to remove the placenta in the usual manner can be unsuccessful, and lacerations or a perforation of the uterine wall may result. However, attempts to remove the placenta do not influence lower genital tract lacerations. Lacerations of the perineum are the most common of all lower genital tract injuries and often occur with both precipitate and operative births and are classified as first-, second-, third-, and fourth-degree lacerations. An abnormal presentation or position of the fetus, the relative size of the presenting part, and the birth canal may contribute to lacerations of the lower genital tract. Congenital abnormalities, previous scarring from infection or injury, and a contracted pelvis may also influence injury to the lower genital tract, followed by hemorrhage.

56. Which information will the nurse include when teaching a patient how to avoid chronic constipation (select all that apply)? a. Many over-the-counter (OTC) medications can cause constipation. b. Stimulant and saline laxatives can be used regularly. c. Bulk-forming laxatives are an excellent source of fiber. d. Walking or cycling frequently will help bowel motility. e. A good time for a bowel movement may be after breakfast.

ANS: A, C, D, E Stimulant and saline laxatives should be used infrequently. Use of bulk-forming laxatives, regular early morning timing of defecation, regular exercise, and avoiding many OTC medications will help the patient avoid constipation.

Which client is at greatest risk for early PPH? a.Primiparous woman (G 2, P 1-0-0-1) being prepared for an emergency cesarean birth for fetal distress b.Woman with severe preeclampsia on magnesium sulfate whose labor is being induced c.Multiparous woman (G 3, P 2-0-0-2) with an 8-hour labor d.Primigravida in spontaneous labor with preterm twins

ANS: B Magnesium sulfate administration during labor poses a risk for PPH. Magnesium acts as a smooth muscle relaxant, thereby contributing to uterine relaxation and atony. A primiparous woman being prepared for an emergency cesarean birth for fetal distress, a multiparous woman with an 8-hour labor, and a primigravida in spontaneous labor with preterm twins do not indicate risk factors or causes of early PPH

What is the primary nursing responsibility when caring for a client who is experiencing an obstetric hemorrhage associated with uterine atony? a.Establishing venous access b.Performing fundal massage c.Preparing the woman for surgical intervention d.Catheterizing the bladder

ANS: B The initial management of excessive postpartum bleeding is a firm massage of the uterine fundus. Although establishing venous access may be a necessary intervention, fundal massage is the initial intervention. The woman may need surgical intervention to treat her postpartum hemorrhage, but the initial nursing intervention is to assess the uterus. After uterine massage, the nurse may want to catheterize the client to eliminate any bladder distention that may be preventing the uterus from properly contracting.

What would a steady trickle of bright red blood from the vagina in the presence of a firm fundus suggest to the nurse? a.Uterine atony b.Lacerations of the genital tract c.Perineal hematoma d.Infection of the uterus

ANS: B Undetected lacerations will bleed slowly and continuously. Bleeding from lacerations is uncontrolled by uterine contraction. The fundus is not firm in the presence of uterine atony. A hematoma would develop internally. Swelling and discoloration would be noticeable; however, bright bleeding would not be. With an infection of the uterus, an odor to the lochia and systemic symptoms such as fever and malaise would be present.

46. A 33-year-old male patient with a gunshot wound to the abdomen undergoes surgery, and a colostomy is formed as shown in the accompanying figure. Which information will be included in patient teaching? a. Stool will be expelled from both stomas. b. This type of colostomy is usually temporary. c. Soft, formed stool can be expected as drainage. d. Irrigations can regulate drainage from the stomas.

ANS: B A loop, or double-barrel stoma, is usually temporary. Stool will be expelled from the proximal stoma only. The stool from the transverse colon will be liquid and regulation through irrigations will not be possible.

Following surgery for an abdominal aortic aneurysm, a patient's central venous pressure (CVP) monitor indicates low pressures. Which action is a priority for the nurse to take? a. Administer IV diuretic medications. b. Increase the IV fluid infusion per protocol. c. Document the CVP and continue to monitor. d. Elevate the head of the patient's bed to 45 degrees.

ANS: B A low CVP indicates hypovolemia and a need for an increase in the infusion rate. Diuretic administration will contribute to hypovolemia and elevation of the head may decrease cerebral perfusion. Documentation and continued monitoring is an inadequate response to the low CVP.

39. The nurse anticipates a suprapubic catheter for which patient? a. A patient with recent prostatectomy b. A patient with a urethral stricture c. A patient with an appendectomy d. A patient with menopause

ANS: B A patient with a urethral stricture is most likely to have a suprapubic catheter. Suprapubic catheters are placed when there is blockage of the urethra (e.g., enlarged prostate, urethral stricture, after urological surgery). A patient with a recent prostatectomy indicates the enlarged prostate was removed and would not need a suprapubic catheter; however, continuous bladder irrigation may be needed. Appendectomies and menopause do not require a suprapubic catheter.

Norepinephrine has been prescribed for a patient who was admitted with dehydration and hypotension. Which patient data indicate that the nurse should consult with the health care provider before starting the norepinephrine? a. The patient is receiving low dose dopamine. b. The patient's central venous pressure is 3 mm Hg. c. The patient is in sinus tachycardia at 120 beats/min. d. The patient has had no urine output since being admitted.

ANS: B Adequate fluid administration is essential before giving vasopressors to patients with hypovolemic shock. The patient's low central venous pressure indicates a need for more volume replacement. The other patient data are not contraindications to norepinephrine administration.

53. The nurse is admitting a 67-year-old patient with new-onset steatorrhea. Which question is most important for the nurse to ask? a. "How much milk do you usually drink?" b. "Have you noticed a recent weight loss?" c. "What time of day do your bowels move?" d. "Do you eat meat or other animal products?"

ANS: B Although all of the questions provide useful information, it is most important to determine if the patient has an imbalance in nutrition because of the steatorrhea.

9. A patient being admitted with an acute exacerbation of ulcerative colitis reports crampy abdominal pain and passing 15 or more bloody stools a day. The nurse will plan to a. administer IV metoclopramide (Reglan). b. discontinue the patient's oral food intake. c. administer cobalamin (vitamin B12) injections. d. teach the patient about total colectomy surgery.

ANS: B An initial therapy for an acute exacerbation of inflammatory bowel disease (IBD) is to rest the bowel by making the patient NPO. Metoclopramide increases peristalsis and will worsen symptoms. Cobalamin (vitamin B12) is absorbed in the ileum, which is not affected by ulcerative colitis. Although total colectomy is needed for some patients, there is no indication that this patient is a candidate.

19. Before administration of captopril to a patient with stage 2 chronic kidney disease (CKD), the nurse will check the patient's a. glucose. b. potassium. c. creatinine. d. phosphate.

ANS: B Angiotensin-converting enzyme (ACE) inhibitors are frequently used in patients with CKD because they delay the progression of the CKD, but they cause potassium retention. Therefore careful monitoring of potassium levels is needed in patients who are at risk for hyperkalemia. The other laboratory values would also be monitored in patients with CKD but would not affect whether the captopril was given or not. DIF: Cognitive Level: Apply (application) REF: 1075 TOP: Nursing Process: Assessment MSC: NCLEX: Physiological Integrity

After change-of-shift report in the progressive care unit, who should the nurse care for first? a. Patient who had an inferior myocardial infarction 2 days ago and has crackles in the lung bases b. Patient with suspected urosepsis who has new orders for urine and blood cultures and antibiotics c. Patient who had a T5 spinal cord injury 1 week ago and currently has a heart rate of 54 beats/minute d. Patient admitted with anaphylaxis 3 hours ago who now has clear lung sounds and a blood pressure of 108/58 mm Hg

ANS: B Antibiotics should be given within the first hour for patients who have sepsis or suspected sepsis in order to prevent progression to systemic inflammatory response syndrome and septic shock. The data on the other patients indicate that they are more stable. Crackles heard only at the lung bases do not require immediate intervention in a patient who has had a myocardial infarction. Mild bradycardia does not usually require atropine in patients who have a spinal cord injury. The findings for the patient admitted with anaphylaxis indicate resolution of bronchospasm and hypotension.

Which finding is the best indicator that the fluid resuscitation for a 90-kg patient with hypovolemic shock has been effective? a. Hemoglobin is within normal limits. b. Urine output is 65 mL over the past hour. c. Central venous pressure (CVP) is normal. d. Mean arterial pressure (MAP) is 72 mm Hg.

ANS: B Assessment of end organ perfusion, such as an adequate urine output, is the best indicator that fluid resuscitation has been successful. Urine output should be equal to or more than 0.5 mL/kg/hr. The hemoglobin level, CVP, and MAP are useful in determining the effects of fluid administration, but they are not as useful as data indicating good organ perfusion.

18. The nurse preparing for the annual physical exam of a 50-year-old man will plan to teach the patient about a. endoscopy. b. colonoscopy. c. computerized tomography screening. d. carcinoembryonic antigen (CEA) testing.

ANS: B At age 50, individuals with an average risk for colorectal cancer (CRC) should begin screening for CRC. Colonoscopy is the gold standard for CRC screening. The other diagnostic tests are not recommended as part of a routine annual physical exam at age 50.

10. Which nursing action will the nurse include in the plan of care for a 35-year-old male patient admitted with an exacerbation of inflammatory bowel disease (IBD)? a. Restrict oral fluid intake. b. Monitor stools for blood. c. Ambulate four times daily. d. Increase dietary fiber intake.

ANS: B Because anemia or hemorrhage may occur with IBD, stools should be assessed for the presence of blood. The other actions would not be appropriate for the patient with IBD. Because dietary fiber may increase gastrointestinal (GI) motility and exacerbate the diarrhea, severe fatigue is common with IBD exacerbations, and dehydration may occur.

27. A 42-yr-old patient admitted with acute kidney injury due to dehydration has oliguria, anemia, and hyperkalemia. Which prescribed action should the nurse take first? a. Insert a urinary retention catheter. b. Place the patient on a cardiac monitor. c. Administer epoetin alfa (Epogen, Procrit). d. Give sodium polystyrene sulfonate (Kayexalate).

ANS: B Because hyperkalemia can cause fatal cardiac dysrhythmias, the initial action should be to monitor the cardiac rhythm. Kayexalate and Epogen will take time to correct the hyperkalemia and anemia. The catheter allows monitoring of the urine output but does not correct the cause of the renal failure. DIF: Cognitive Level: Analyze (analysis) REF: 1073 OBJ: Special Questions: Prioritization TOP: Nursing Process: Implementation MSC: NCLEX: Physiological Integrity

35. A patient in the emergency department has just been diagnosed with peritonitis caused by a ruptured diverticulum. Which prescribed intervention will the nurse implement first? a. Insert a urinary catheter to drainage. b. Infuse metronidazole (Flagyl) 500 mg IV. c. Send the patient for a computerized tomography scan. d. Place a nasogastric (NG) tube to intermittent low suction.

ANS: B Because peritonitis can be fatal if treatment is delayed, the initial action should be to start antibiotic therapy (after any ordered cultures are obtained). The other actions can be done after antibiotic therapy is initiated.

Which intervention will the nurse include in the plan of care for a patient who has cardiogenic shock? a. Check temperature every 2 hours. b. Monitor breath sounds frequently. c. Maintain patient in supine position. d. Assess skin for flushing and itching.

ANS: B Because pulmonary congestion and dyspnea are characteristics of cardiogenic shock, the nurse should assess the breath sounds frequently. The head of the bed is usually elevated to decrease dyspnea in patients with cardiogenic shock. Elevated temperature and flushing or itching of the skin are not typical of cardiogenic shock.

Which finding about a patient who is receiving vasopressin to treat septic shock indicates an immediate need for the nurse to report the finding to the health care provider? a. The patient's urine output is 18 mL/hr. b. The patient is complaining of chest pain. c. The patient's peripheral pulses are weak. d. The patient's heart rate is 110 beats/minute.

ANS: B Because vasopressin is a potent vasoconstrictor, it may decrease coronary artery perfusion. The other information is consistent with the patient's diagnosis, and should be reported to the health care provider but does not indicate an immediate need for a change in therapy.

41. Which information obtained by the nurse interviewing a 30-year-old male patient is most important to communicate to the health care provider? a. The patient has a history of constipation. b. The patient has noticed blood in the stools. c. The patient had an appendectomy at age 27. d. The patient smokes a pack/day of cigarettes.

ANS: B Blood in the stools is a possible clinical manifestation of colorectal cancer and requires further assessment by the health care provider. The other patient information will also be communicated to the health care provider, but does not indicate an urgent need for further testing or intervention.

20. A 74-year-old patient preparing to undergo a colon resection for cancer of the colon asks about the elevated carcinoembryonic antigen (CEA) test result. The nurse explains that the test is used to a. identify any metastasis of the cancer. b. monitor the tumor status after surgery. c. confirm the diagnosis of a specific type of cancer. d. determine the need for postoperative chemotherapy.

ANS: B CEA is used to monitor for cancer recurrence after surgery. CEA levels do not help to determine whether there is metastasis of the cancer. Confirmation of the diagnosis is made on the basis of biopsy. Chemotherapy use is based on factors other than CEA.

6. Which information will the nurse monitor in order to determine the effectiveness of prescribed calcium carbonate (Caltrate) for a patient with chronic kidney disease (CKD)? a. Blood pressure b. Phosphate level c. Neurologic status d. Creatinine clearance

ANS: B Calcium carbonate is prescribed to bind phosphorus and prevent mineral and bone disease in patients with CKD. The other data will not be helpful in evaluating the effectiveness of calcium carbonate. DIF: Cognitive Level: Apply (application) REF: 1081 TOP: Nursing Process: Evaluation MSC: NCLEX: Physiological Integrity

Which nursing action is a priority for a patient who has suffered a burn injury while working on an electrical power line? a. Obtain the blood pressure. b. Stabilize the cervical spine. c. Assess for the contact points. d. Check alertness and orientation.

ANS: B Cervical spine injuries are commonly associated with electrical burns. Therefore stabilization of the cervical spine takes precedence after airway management. The other actions are also included in the emergent care after electrical burns, but the most important action is to avoid spinal cord injury

30. A female patient with chronic kidney disease (CKD) is receiving peritoneal dialysis with 2-L inflows. Which information should the nurse report promptly to the health care provider? a. The patient has an outflow volume of 1800 mL. b. The patient's peritoneal effluent appears cloudy. c. The patient's abdomen appears bloated after the inflow. d. The patient has abdominal pain during the inflow phase.

ANS: B Cloudy-appearing peritoneal effluent is a sign of peritonitis and should be reported immediately so that treatment with antibiotics can be started. The other problems can be addressed through nursing interventions such as slowing the inflow and repositioning the patient. DIF: Cognitive Level: Apply (application) REF: 1087 TOP: Nursing Process: Assessment MSC: NCLEX: Physiological Integrity

A 68-year-old patient has been in the intensive care unit for 4 days and has a nursing diagnosis of disturbed sensory perception related to sleep deprivation. Which action should the nurse include in the plan of care? a. Administer prescribed sedatives or opioids at bedtime to promote sleep. b. Cluster nursing activities so that the patient has uninterrupted rest periods. c. Silence the alarms on the cardiac monitors to allow 30- to 40-minute naps. d. Eliminate assessments between 0100 and 0600 to allow uninterrupted sleep.

ANS: B Clustering nursing activities and providing uninterrupted rest periods will minimize sleep-cycle disruption. Sedative and opioid medications tend to decrease the amount of rapid eye movement (REM) sleep and can contribute to sleep disturbance and disturbed sensory perception. Silencing the alarms on the cardiac monitors would be unsafe in a critically ill patient, as would discontinuing assessments during the night.

32. The nurse is assessing a 31-year-old female patient with abdominal pain. Th nurse,who notes that there is ecchymosis around the area of umbilicus, will document this finding as a. Cullen sign. b. Rovsing sign. c. McBurney sign. d. Grey-Turner's signt.

ANS: B Cullen sign is ecchymosis around the umbilicus. Rovsing sign occurs when palpation of the left lower quadrant causes pain in the right lower quadrant. Deep tenderness at McBurney's point (halfway between the umbilicus and the right iliac crest), known as McBurney's sign, is a sign of acute appendicitis.

Several weeks after a stroke, a 50-year-old male patient has impaired awareness of bladder fullness, resulting in urinary incontinence. Which nursing intervention will be best to include in the initial plan for an effective bladder training program? a. Limit fluid intake to 1200 mL daily to reduce urine volume. b. Assist the patient onto the bedside commode every 2 hours. c. Perform intermittent catheterization after each voiding to check for residual urine. d. Use an external "condom" catheter to protect the skin and prevent embarrassment.

ANS: B Developing a regular voiding schedule will prevent incontinence and may increase patient awareness of a full bladder. A 1200 mL fluid restriction may lead to dehydration. Intermittent catheterization and use of a condom catheter are appropriate in the acute phase of stroke, but should not be considered solutions for long-term management because of the risks for urinary tract infection (UTI) and skin breakdown.

28. A patient has arrived for a scheduled hemodialysis session. Which nursing action is most appropriate for the registered nurse (RN) to delegate to a dialysis technician? a. Teach the patient about fluid restrictions. b. Check blood pressure before starting dialysis. c. Assess for causes of an increase in predialysis weight. d. Determine the ultrafiltration rate for the hemodialysis.

ANS: B Dialysis technicians are educated in monitoring for blood pressure. Assessment, adjustment of the appropriate ultrafiltration rate, and patient teaching require the education and scope of practice of an RN. DIF: Cognitive Level: Apply (application) REF: 1089 OBJ: Special Questions: Delegation TOP: Nursing Process: Planning MSC: NCLEX: Safe and Effective Care Environment

15. Which information in a patient's history indicates to the nurse that the patient is not an appropriate candidate for kidney transplantation? a. The patient has type 1 diabetes. b. The patient has metastatic lung cancer. c. The patient has a history of chronic hepatitis C infection. d. The patient is infected with human immunodeficiency virus.

ANS: B Disseminated malignancies are a contraindication to transplantation. The conditions of the other patients are not contraindications for kidney transplant. DIF: Cognitive Level: Understand (comprehension) REF: 1092 TOP: Nursing Process: Assessment MSC: NCLEX: Physiological Integrity

6. The nurse, upon reviewing the history, discovers the patient has dysuria. Which assessment finding is consistent with dysuria? a. Blood in the urine b. Burning upon urination c. Immediate, strong desire to void d. Awakes from sleep due to urge to void

ANS: B Dysuria is burning or pain with urination. Hematuria is blood in the urine. Urgency is an immediate and strong desire to void that is not easily deferred. Nocturia is awakening form sleep due to urge to void.

The central venous oxygen saturation (ScvO2) is decreasing in a patient who has severe pancreatitis. To determine the possible cause of the decreased ScvO2, the nurse assesses the patient's a. lipase. b. temperature. c. urinary output. d. body mass index.

ANS: B Elevated temperature increases metabolic demands and oxygen use by tissues, resulting in a drop in oxygen saturation of central venous blood. Information about the patient's body mass index, urinary output, and lipase will not help in determining the cause of the patient's drop in ScvO2.

14. After a total proctocolectomy and permanent ileostomy, the patient tells the nurse, "I cannot manage all these changes. I don't want to look at the stoma." What is the best action by the nurse? a. Reassure the patient that ileostomy care will become easier. b. Ask the patient about the concerns with stoma management. c. Develop a detailed written list of ostomy care tasks for the patient. d. Postpone any teaching until the patient adjusts to the ileostomy.

ANS: B Encouraging the patient to share concerns assists in helping the patient adjust to the body changes. Acknowledgment of the patient's feelings and concerns is important rather than offering false reassurance. Because the patient indicates that the feelings about the ostomy are the reason for the difficulty with the many changes, development of a detailed ostomy care plan will not improve the patient's ability to manage the ostomy. Although detailed ostomy teaching may be postponed, the nurse should offer teaching about some aspects of living with an ostomy.

33. A 54-year-old critically ill patient with sepsis is frequently incontinent of watery stools. What action by the nurse will prevent complications associated with ongoing incontinence? a. Apply incontinence briefs. b. Use a fecal management system c. Insert a rectal tube with a drainage bag. d. Assist the patient to a commode frequently.

ANS: B Fecal management systems are designed to contain loose stools and can be in place for as long as 4 weeks without causing damage to the rectum or anal sphincters. Although incontinence briefs may be helpful, unless they are changed frequently, they are likely to increase the risk for skin breakdown. Rectal tubes are avoided because of possible damage to the anal sphincter and ulceration of the rectal mucosa. A critically ill patient will not be able to tolerate getting up frequently to use the commode or bathroom.

16. A 24-year-old woman with Crohn's disease develops a fever and symptoms of a urinary tract infection (UTI) with tan, fecal-smelling urine. What information will the nurse add to a general teaching plan about UTIs in order to individualize the teaching for this patient? a. Bacteria in the perianal area can enter the urethra. b. Fistulas can form between the bowel and bladder. c. Drink adequate fluids to maintain normal hydration. d. Empty the bladder before and after sexual intercourse.

ANS: B Fistulas between the bowel and bladder occur in Crohn's disease and can lead to UTI. Teaching for UTI prevention in general includes good hygiene, adequate fluid intake, and voiding before and after intercourse.

33. The nurse is titrating the IV fluid infusion rate immediately after a patient has had kidney transplantation. Which parameter will be most important for the nurse to consider? a. Heart rate b. Urine output c. Creatinine clearance d. Blood urea nitrogen (BUN) level

ANS: B Fluid volume is replaced based on urine output after transplant because the urine output can be as high as a liter an hour. The other data will be monitored but are not the most important determinants of fluid infusion rate. DIF: Cognitive Level: Analyze (analysis) REF: 1095 OBJ: Special Questions: Prioritization TOP: Nursing Process: Implementation MSC: NCLEX: Physiological Integrity

A patient in the clinic reports a recent episode of dysphasia and left-sided weakness at home that resolved after 2 hours. The nurse will anticipate teaching the patient about a. alteplase (tPA). b. aspirin (Ecotrin). c. warfarin (Coumadin). d. nimodipine (Nimotop)

ANS: B Following a transient ischemic attack (TIA), patients typically are started on medications such as aspirin to inhibit platelet function and decrease stroke risk. tPA is used for acute ischemic stroke. Coumadin is usually used for patients with atrial fibrillation. Nimodipine is used to prevent cerebral vasospasm after a subarachnoid hemorrhage.

The intensive care unit (ICU) nurse educator will determine that teaching about arterial pressure monitoring for a new staff nurse has been effective when the nurse a. balances and calibrates the monitoring equipment every 2 hours. b. positions the zero-reference stopcock line level with the phlebostatic axis. c. ensures that the patient is supine with the head of the bed flat for all readings. d. rechecks the location of the phlebostatic axis when changing the patient's position.

ANS: B For accurate measurement of pressures, the zero-reference level should be at the phlebostatic axis. There is no need to rebalance and recalibrate monitoring equipment hourly. Accurate hemodynamic readings are possible with the patient's head raised to 45 degrees or in the prone position. The anatomic position of the phlebostatic axis does not change when patients are repositioned.

23. Which information will the nurse include in teaching a patient who had a proctocolectomy and ileostomy for ulcerative colitis? a. Restrict fluid intake to prevent constant liquid drainage from the stoma. b. Use care when eating high-fiber foods to avoid obstruction of the ileum. c. Irrigate the ileostomy daily to avoid having to wear a drainage appliance. d. Change the pouch every day to prevent leakage of contents onto the skin.

ANS: B High-fiber foods are introduced gradually and should be well chewed to avoid obstruction of the ileostomy. Patients with ileostomies lose the absorption of water in the colon and need to take in increased amounts of fluid. The pouch should be drained frequently but is changed every 5 to 7 days. The drainage from an ileostomy is liquid and continuous, so control by irrigation is not possible.

54. Which information will the nurse teach a 23-year-old patient with lactose intolerance? a. Ice cream is relatively low in lactose. b. Live-culture yogurt is usually tolerated. c. Heating milk will break down the lactose. d. Nonfat milk is a better choice than whole milk.

ANS: B Lactose-intolerant individuals can usually eat yogurt without experiencing discomfort. Ice cream, nonfat milk, and milk that has been heated are all high in lactose.

25. The nurse is preparing to test a patient for postvoid residual with a bladder scan. Which action will the nurse take? a. Measure bladder before the patient voids. b. Measure bladder within 10 minutes after the patient voids. c. Measure bladder with head of bed raised to 60 degrees. d. Measure bladder with head of bed raised to 90 degrees.

ANS: B Measurement should be within 10 minutes of voiding. It is a postvoid so the measurement is after the patient voids and the urine volume is recorded. Patient is supine with head slightly elevated.

A patient with massive trauma and possible spinal cord injury is admitted to the emergency department (ED). Which assessment finding by the nurse will help confirm a diagnosis of neurogenic shock? a. Inspiratory crackles c. Cool, clammy extremities b. Heart rate 45 beats/min d. Temperature 101.2°F (38.4°C)

ANS: B Neurogenic shock is characterized by hypotension and bradycardia. The other findings would be more consistent with other types of shock.

The nurse is admitting a patient who has a neck fracture at the C6 level to the intensive care unit. Which assessment findings indicate neurogenic shock? a. Involuntary and spastic movement b. Hypotension and warm extremities c. Hyperactive reflexes below the injury d. Lack of sensation or movement below the injury

ANS: B Neurogenic shock is characterized by hypotension, bradycardia, and vasodilation leading to warm skin temperature. Spasticity and hyperactive reflexes do not occur at this stage of spinal cord injury. Lack of movement and sensation indicate spinal cord injury but not neurogenic shock.

52. After change-of-shift report, which patient should the nurse assess first? a. 40-year-old male with celiac disease who has frequent frothy diarrhea b. 30-year-old female with a femoral hernia who has abdominal pain and vomiting c. 30-year-old male with ulcerative colitis who has severe perianal skin breakdown d. 40-year-old female with a colostomy bag that is pulling away from the adhesive wafer

ANS: B Pain and vomiting with a femoral hernia suggest possible strangulation, which will necessitate emergency surgery. The other patients have less urgent problems.

24. Which statement by the patient about an upcoming contrast computed tomography (CT) scan indicates a need for further teaching? a. "I will follow the food and drink restrictions as directed before the test is scheduled." b. "I will be anesthetized so that I lie perfectly still during the procedure." c. "I will complete my bowel prep program the night before the scan." d. "I will be drinking a lot of fluid after the test is over."

ANS: B Patients are not put under anesthesia for a CT scan; instead, the nurse should educate patients about the need to lie perfectly still and about possible methods of overcoming feelings of claustrophobia. The other options are correct and require no further teaching. Patients need to be assessed for an allergy to shellfish if receiving contrast for the CT scan. Bowel cleansing is often performed before CT scan. Another area to address is food and fluid restriction up to 4 hours prior to the test. After the procedure, encourage fluids to promote dye excretion.

50. A new 19-year-old male patient has familial adenomatous polyposis (FAP). Which action will the nurse in the gastrointestinal clinic include in the plan of care? a. Obtain blood samples for DNA analysis. b. Schedule the patient for yearly colonoscopy. c. Provide preoperative teaching about total colectomy. d. Discuss lifestyle modifications to decrease cancer risk.

ANS: B Patients with FAP should have annual colonoscopy starting at age 16 and usually have total colectomy by age 25 to avoid developing colorectal cancer. DNA analysis is used to make the diagnosis, but is not needed now for this patient. Lifestyle modifications will not decrease cancer risk for this patient.

2. When a patient with acute kidney injury (AKI) has an arterial blood pH of 7.30, the nurse will expect an assessment finding of a. persistent skin tenting b. rapid, deep respirations. c. hot, flushed face and neck. d. bounding peripheral pulses.

ANS: B Patients with metabolic acidosis caused by AKI may have Kussmaul respirations as the lungs try to regulate carbon dioxide. Bounding pulses and vasodilation are not associated with metabolic acidosis. Because the patient is likely to have fluid retention, poor skin turgor would not be a finding in AKI. DIF: Cognitive Level: Apply (application) REF: 1072 TOP: Nursing Process: Assessment MSC: NCLEX: Physiological Integrity

55. Which prescribed intervention for a 61-year-old female patient with chronic short bowel syndrome will the nurse question? a. Ferrous sulfate (Feosol) 325 mg daily b. Senna (Senokot) 1 tablet every day c. Psyllium (Metamucil) 2.1 grams 3 times daily d. Diphenoxylate with atropine (Lomotil) prn loose stools

ANS: B Patients with short bowel syndrome have diarrhea because of decreased nutrient and fluid absorption and would not need stimulant laxatives. Iron supplements are used to prevent iron-deficiency anemia, bulk-forming laxatives help make stools less watery, and opioid antidiarrheal drugs are helpful in slowing intestinal transit time.

When caring for a patient who has an arterial catheter in the left radial artery for arterial pressure-based cardiac output (APCO) monitoring, which information obtained by the nurse is most important to report to the health care provider? a. The patient has a positive Allen test. b. There is redness at the catheter insertion site. c. The mean arterial pressure (MAP) is 86 mm Hg. d. The dicrotic notch is visible in the arterial waveform.

ANS: B Redness at the catheter insertion site indicates possible infection. The Allen test is performed before arterial line insertion, and a positive test indicates normal ulnar artery perfusion. A MAP of 86 is normal and the dicrotic notch is normally present on the arterial waveform.

While family members are visiting, a patient has a respiratory arrest and is being resuscitated. Which action by the nurse is best? a. Tell the family members that watching the resuscitation will be very stressful. b. Ask family members if they wish to remain in the room during the resuscitation. c. Take the family members quickly out of the patient room and remain with them. d. Assign a staff member to wait with family members just outside the patient room.

ANS: B Research indicates that family members want the option of remaining in the room during procedures such as cardiopulmonary resuscitation (CPR) and that this decreases anxiety and facilitates grieving. The other options may be appropriate if the family decides not to remain with the patient.

6. A 58-year-old man with blunt abdominal trauma from a motor vehicle crash undergoes peritoneal lavage. If the lavage returns brown fecal drainage, which action will the nurse plan to take next? a. Auscultate the bowel sounds. b. Prepare the patient for surgery. c. Check the patient's oral temperature. d. Obtain information about the accident.

ANS: B Return of brown drainage and fecal material suggests perforation of the bowel and the need for immediate surgery. Auscultation of bowel sounds, checking the temperature, and obtaining information about the accident are appropriate actions, but the priority is to prepare to send the patient for emergency surgery.

The nurse is caring for a patient who has just returned after having left carotid artery angioplasty and stenting. Which assessment information is of most concern to the nurse? a. The pulse rate is 102 beats/min. b. The patient has difficulty speaking. c. The blood pressure is 144/86 mm Hg. d. There are fine crackles at the lung bases

ANS: B Small emboli can occur during carotid artery angioplasty and stenting, and the aphasia indicates a possible stroke during the procedure. Slightly elevated pulse rate and blood pressure are not unusual because of anxiety associated with the procedure. Fine crackles at the lung bases may indicate atelectasis caused by immobility during the procedure. The nurse should have the patient take some deep breaths.

20. The patient is taking phenazopyridine. When assessing the urine, what will the nurse expect? a. Red color b. Orange color c. Dark amber color d. Intense yellow color

ANS: B Some drugs change the color of urine (e.g., phenazopyridine—orange, riboflavin—intense yellow). Eating beets, rhubarb, and blackberries causes red urine. Dark amber urine is the result of high concentrations of bilirubin in patients with liver disease.

11. Which patient statement indicates that the nurse's teaching about sulfasalazine (Azulfidine) for ulcerative colitis has been effective? a. "The medication will be tapered if I need surgery." b. "I will need to use a sunscreen when I am outdoors." c. "I will need to avoid contact with people who are sick." d. "The medication will prevent infections that cause the diarrhea."

ANS: B Sulfasalazine may cause photosensitivity in some patients. It is not used to treat infections. Sulfasalazine does not reduce immune function. Unlike corticosteroids, tapering of sulfasalazine is not needed.

Which hemodynamic parameter is most appropriate for the nurse to monitor to determine the effectiveness of medications given to a patient to reduce left ventricular afterload? a. Mean arterial pressure (MAP) b. Systemic vascular resistance (SVR) c. Pulmonary vascular resistance (PVR) d. Pulmonary artery wedge pressure (PAWP)

ANS: B Systemic vascular resistance reflects the resistance to ventricular ejection, or afterload. The other parameters will be monitored, but do not reflect afterload as directly.

A patient with cardiogenic shock has the following vital signs: BP 102/50, pulse 128, respirations 28. The pulmonary artery wedge pressure (PAWP) is increased, and cardiac output is low. The nurse will anticipate an order for which medication? a. 5% albumin infusion c. epinephrine (Adrenalin) drip b. furosemide (Lasix) IV d. hydrocortisone (Solu-Cortef)

ANS: B The PAWP indicates that the patient's preload is elevated, and furosemide is indicated to reduce the preload and improve cardiac output. Epinephrine would further increase the heart rate and myocardial oxygen demand. 5% albumin would also increase the PAWP. Hydrocortisone might be considered for septic or anaphylactic shock.

The nurse educator is evaluating the performance of a new registered nurse (RN) who is providing care to a patient who is receiving mechanical ventilation with 15 cm H2O of peak end-expiratory pressure (PEEP). Which action indicates that the new RN is safe? a. The RN plans to suction the patient every 1 to 2 hours. b. The RN uses a closed-suction technique to suction the patient. c. The RN tapes connection between the ventilator tubing and the ET. d. The RN changes the ventilator circuit tubing routinely every 48 hours.

ANS: B The closed-suction technique is used when patients require high levels of PEEP (>10 cm H2O) to prevent the loss of PEEP that occurs when disconnecting the patient from the ventilator. Suctioning should not be scheduled routinely, but it should be done only when patient assessment data indicate the need for suctioning. Taping connections between the ET and the ventilator tubing would restrict the ability of the tubing to swivel in response to patient repositioning. Ventilator tubing changes increase the risk for ventilator-associated pneumonia (VAP) and are not indicated routinely.

31. The nurse is assessing a patient 4 hours after a kidney transplant. Which information is most important to communicate to the health care provider? a. The urine output is 900 to 1100 mL/hr. b. The patient's central venous pressure (CVP) is decreased. c. The patient has a level 7 (0- to 10-point scale) incisional pain. d. The blood urea nitrogen (BUN) and creatinine levels are elevated.

ANS: B The decrease in CVP suggests hypovolemia, which must be rapidly corrected to prevent renal hypoperfusion and acute tubular necrosis. The other information is not unusual in a patient after a transplant. DIF: Cognitive Level: Analyze (analysis) REF: 1095 OBJ: Special Questions: Prioritization TOP: Nursing Process: Assessment MSC: NCLEX: Physiological Integrity

A patient who has been involved in a motor vehicle crash arrives in the emergency department (ED) with cool, clammy skin; tachycardia; and hypotension. Which intervention ordered by the health care provider should the nurse implement first? a. Insert two large-bore IV catheters. b. Provide O2 at 100% per non-rebreather mask. c. Draw blood to type and crossmatch for transfusions. d. Initiate continuous electrocardiogram (ECG) monitoring.

ANS: B The first priority in the initial management of shock is maintenance of the airway and ventilation. ECG monitoring, insertion of IV catheters, and obtaining blood for transfusions should also be rapidly accomplished but only after actions to maximize O2 delivery have been implemented.

1. A nurse is teaching a patient about the urinary system. In which order will the nurse present the structures, following the flow of urine? a. Kidney, urethra, bladder, ureters b. Kidney, ureters, bladder, urethra c. Bladder, kidney, ureters, urethra d. Bladder, kidney, urethra, ureters

ANS: B The flow of urine follows these structures: kidney, ureters, bladder, and urethra.

25. A 62-yr-old female patient has been hospitalized for 4 days with acute kidney injury (AKI) caused by dehydration. Which information will be most important for the nurse to report to the health care provider? a. The creatinine level is 3.0 mg/dL. b. Urine output over an 8-hour period is 2500 mL. c. The blood urea nitrogen (BUN) level is 67 mg/dL. d. The glomerular filtration rate is less than 30 mL/min/1.73 m2.

ANS: B The high urine output indicates a need to increase fluid intake to prevent hypovolemia. The other information is typical of AKI and will not require a change in therapy. DIF: Cognitive Level: Analyze (analysis) REF: 1072 OBJ: Special Questions: Prioritization TOP: Nursing Process: Assessment MSC: NCLEX: Physiological Integrity

23. A 25-yr-old male patient has been admitted with a severe crushing injury after an industrial accident. Which laboratory result will be most important to report to the health care provider? a. Serum creatinine level of 2.1 mg/dL b. Serum potassium level of 6.5 mEq/L c. White blood cell count of 11,500/µL d. Blood urea nitrogen (BUN) of 56 mg/dL

ANS: B The hyperkalemia associated with crushing injuries may cause cardiac arrest and should be treated immediately. The nurse also will report the other laboratory values, but abnormalities in these are not immediately life threatening. DIF: Cognitive Level: Analyze (analysis) REF: 1072 OBJ: Special Questions: Prioritization TOP: Nursing Process: Assessment MSC: NCLEX: Physiological Integrity

When evaluating a patient with a central venous catheter, the nurse observes that the insertion site is red and tender to touch and the patient's temperature is 101.8° F. What should the nurse plan to do next? a. Give analgesics and antibiotics as ordered. b. Discontinue the catheter and culture the tip. c. Change the flush system and monitor the site. d. Check the site more frequently for any swelling.

ANS: B The information indicates that the patient has a local and systemic infection caused by the catheter, and the catheter should be discontinued. Changing the flush system, giving analgesics, and continued monitoring will not help prevent or treat the infection. Administration of antibiotics is appropriate, but the line should still be discontinued to avoid further complications such as endocarditis.

36. A 25-year-old male patient calls the clinic complaining of diarrhea for 24 hours. Which action should the nurse take first? a. Inform the patient that laboratory testing of blood and stools will be necessary. b. Ask the patient to describe the character of the stools and any associated symptoms. c. Suggest that the patient drink clear liquid fluids with electrolytes, such as Gatorade or Pedialyte. d. Advise the patient to use over-the-counter loperamide (Imodium) to slow gastrointestinal (GI) motility.

ANS: B The initial response by the nurse should be further assessment of the patient. The other responses may be appropriate, depending on what is learned in the assessment.

24. The nurse will determine that teaching a 67-year-old man to irrigate his new colostomy has been effective if the patient a. inserts the irrigation tubing 4 to 6 inches into the stoma. b. hangs the irrigating container 18 inches above the stoma. c. stops the irrigation and removes the irrigating cone if cramping occurs. d. fills the irrigating container with 1000 to 2000 mL of lukewarm tap water.

ANS: B The irrigating container should be hung 18 to 24 inches above the stoma. If cramping occurs, the irrigation should be temporarily stopped and the cone left in place. Five hundred to 1000 mL of water should be used for irrigation. An irrigation cone, rather than tubing, should be inserted into the stoma; 4 to 6 inches would be too far for safe insertion.

35. A 74-yr-old patient who is progressing to stage 5 chronic kidney disease asks the nurse, "Do you think I should go on dialysis? Which initial response by the nurse is best? a. "It depends on which type of dialysis you are considering." b. "Tell me more about what you are thinking regarding dialysis." c. "You are the only one who can make the decision about dialysis." d. "Many people your age use dialysis and have a good quality of life."

ANS: B The nurse should initially clarify the patient's concerns and questions about dialysis. The patient is the one responsible for the decision, and many people using dialysis do have good quality of life, but these responses block further assessment of the patient's concerns. Referring to which type of dialysis the patient might use only indirectly responds to the patient's question. DIF: Cognitive Level: Analyze (analysis) REF: 1091 OBJ: Special Questions: Prioritization TOP: Nursing Process: Assessment MSC: NCLEX: Psychosocial Integrity

36. A nurse is providing care to a group of patients. Which patient will the nurse see first? a. A patient who is dribbling small amounts on the way to the bathroom and has a diagnosis of urge incontinence b. A patient with reflex incontinence with elevated blood pressure and pulse rate c. A patient with an indwelling catheter that has stool on the catheter tubing d. A patient who has just voided and needs a postvoid residual test

ANS: B The nurse should see the patient with reflex incontinence first. Patients with reflex incontinence are at risk for developing autonomic dysreflexia, a life-threatening condition that causes severe elevation of blood pressure and pulse rate and diaphoresis. This is a medical emergency requiring immediate intervention; notify the health care provider immediately. A patient with urge incontinence will dribble, and this is expected. While a patient with a catheter and stool on the tubing does need to be cleaned, it is not life threatening. The nurse has 10 minutes before checking on the patient who has a postvoid residual test.

2. A 71-year-old male patient tells the nurse that growing old causes constipation so he has been using a suppository for constipation every morning. Which action should the nurse take first? a. Encourage the patient to increase oral fluid intake. b. Assess the patient about risk factors for constipation. c. Suggest that the patient increase intake of high-fiber foods. d. Teach the patient that a daily bowel movement is unnecessary.

ANS: B The nurse's initial action should be further assessment of the patient for risk factors for constipation and for his usual bowel pattern. The other actions may be appropriate but will be based on the assessment.

22. Which intervention will be included in the plan of care for a patient with acute kidney injury (AKI) who has a temporary vascular access catheter in the left femoral vein? a. Start continuous pulse oximetry. b. Restrict physical activity to bed rest. c. Restrict the patient's oral protein intake. d. Discontinue the urethral retention catheter.

ANS: B The patient with a femoral vein catheter must be on bed rest to prevent trauma to the vein. Protein intake is likely to be increased when the patient is receiving dialysis. The retention catheter is likely to remain in place because accurate measurement of output will be needed. There is no indication that the patient needs continuous pulse oximetry. DIF: Cognitive Level: Apply (application) REF: 1088 TOP: Nursing Process: Planning MSC: NCLEX: Physiological Integrity

32. During routine hemodialysis, a patient complains of nausea and dizziness. Which action should the nurse take first? a. Slow down the rate of dialysis. b. Check the blood pressure (BP). c. Review the hematocrit (Hct) level. d. Give prescribed PRN antiemetic drugs.

ANS: B The patient's complaints of nausea and dizziness suggest hypotension, so the initial action should be to check the BP. The other actions may also be appropriate based on the blood pressure obtained. DIF: Cognitive Level: Analyze (analysis) REF: 1090 OBJ: Special Questions: Prioritization TOP: Nursing Process: Implementation MSC: NCLEX: Physiological Integrity

A nurse is caring for a patient whose hemodynamic monitoring indicates a blood pressure of 92/54 mm Hg, a pulse of 64 beats/min, and an elevated pulmonary artery wedge pressure (PAWP). Which intervention ordered by the health care provider should the nurse question? a. Elevate head of bed to 30 degrees. b. Infuse normal saline at 250 mL/hr. c. Hold nitroprusside if systolic BP is less than 90 mm Hg. d. Titrate dobutamine to keep systolic BP is greater than 90 mm Hg.

ANS: B The patient's elevated PAWP indicates volume excess in relation to cardiac pumping ability, consistent with cardiogenic shock. A saline infusion at 250 mL/hr will exacerbate the volume excess. The other actions will help to improve cardiac output, which should lower the PAWP and may raise the BP.

A patient is admitted to the burn unit with burns to the head, face, and hands. Initially, wheezes are heard, but an hour later, the lung sounds are decreased and no wheezes are audible. What is the best action for the nurse to take? a. Encourage the patient to cough and auscultate the lungs again. b. Notify the health care provider and prepare for endotracheal intubation. c. Document the results and continue to monitor the patient's respiratory rate. d. Reposition the patient in high-Fowler's position and reassess breath sounds.

ANS: B The patient's history and clinical manifestations suggest airway edema and the health care provider should be notified immediately, so that intubation can be done rapidly. Placing the patient in a more upright position or having the patient cough will not address the problem of airway edema. Continuing to monitor is inappropriate because immediate action should occur

An 81-year-old patient who has been in the intensive care unit (ICU) for a week is now stable and transfer to the progressive care unit is planned. On rounds, the nurse notices that the patient has new onset confusion. The nurse will plan to a. give PRN lorazepam (Ativan) and cancel the transfer. b. inform the receiving nurse and then transfer the patient. c. notify the health care provider and postpone the transfer. d. obtain an order for restraints as needed and transfer the patient.

ANS: B The patient's history and symptoms most likely indicate delirium associated with the sleep deprivation and sensory overload in the ICU environment. Informing the receiving nurse and transferring the patient is appropriate. Postponing the transfer is likely to prolong the delirium. Benzodiazepines and restraints contribute to delirium and agitation.

3. The nurse is planning care for a patient with severe heart failure who has developed elevated blood urea nitrogen (BUN) and creatinine levels. The primary treatment goal in the plan will be a. augmenting fluid volume. b. maintaining cardiac output. c. diluting nephrotoxic substances. d. preventing systemic hypertension.

ANS: B The primary goal of treatment for acute kidney injury (AKI) is to eliminate the cause and provide supportive care while the kidneys recover. Because this patient's heart failure is causing AKI, the care will be directed toward treatment of the heart failure. For renal failure caused by hypertension, hypovolemia, or nephrotoxins, the other responses would be correct. DIF: Cognitive Level: Apply (application) REF: 1073 TOP: Nursing Process: Planning MSC: NCLEX: Physiological Integrity

A nurse is caring for a patient who has burns of the ears, head, neck, and right arm and hand. The nurse should place the patient in which position? a. Place the right arm and hand flexed in a position of comfort. b. Elevate the right arm and hand on pillows and extend the fingers. c. Assist the patient to a supine position with a small pillow under the head. d. Position the patient in a side-lying position with rolled towel under the neck.

ANS: B The right hand and arm should be elevated to reduce swelling and the fingers extended to avoid flexion contractures (even though this position may not be comfortable for the patient). The patient with burns of the ears should not use a pillow for the head because this will put pressure on the ears, and the pillow may stick to the ears. Patients with neck burns should not use a pillow because the head should be maintained in an extended position in order to avoid contractures

A young adult patient who is in the rehabilitation phase 6 months after a severe face and neck burn tells the nurse, "I'm sorry that I'm still alive. My life will never be normal again." Which response by the nurse is best? a. "Most people recover after a burn and feel satisfied with their lives." b. "It's true that your life may be different. What concerns you the most?" c. "It is really too early to know how much your life will be changed by the burn." d. "Why do you feel that way? You will be able to adapt as your recovery progresses."

ANS: B This response acknowledges the patient's feelings and asks for more assessment data that will help in developing an appropriate plan of care to assist the patient with the emotional response to the burn injury. The other statements are accurate, but do not acknowledge the anxiety and depression that the patient is expressing

Which information about the patient who has had a subarachnoid hemorrhage is most important to communicate to the health care provider? a. The patient complains of having a stiff neck. b. The patient's blood pressure (BP) is 90/50 mm Hg. c. The patient reports a severe and unrelenting headache. d. The cerebrospinal fluid (CSF) report shows red blood cells (RBCs).

ANS: B To prevent cerebral vasospasm and maintain cerebral perfusion, blood pressure needs to be maintained at a level higher than 90 mm Hg systolic after a subarachnoid hemorrhage. A low BP or drop in BP indicates a need to administer fluids and/or vasopressors to increase the BP. An ongoing headache, RBCs in the CSF, and a stiff neck are all typical clinical manifestations of a subarachnoid hemorrhage and do not need to be rapidly communicated to the health care provider.

27. A female patient is having difficulty voiding in a bedpan but states that her bladder feels full. To stimulate micturition, which nursing intervention should the nurse try first? a. Exiting the room and informing the patient that the nurse will return in 30 minutes to check on the patient's progress. b. Utilizing the power of suggestion by turning on the faucet and letting the water run. c. Obtaining an order for a Foley catheter. d. Administering diuretic medication.

ANS: B To stimulate micturition, the nurse should attempt noninvasive procedures first. Running warm water or stroking the inner aspect of the upper thigh promotes sensory perception that leads to urination. A patient should not be left alone on a bedpan for 30 minutes because this could cause skin breakdown. Catheterization places the patient at increased risk of infection and should not be the first intervention attempted. Diuretics are useful if the patient is not producing urine, but they do not stimulate micturition.

7. An 86-year-old patient is experiencing uncontrollable leakage of urine with a strong desire to void and even leaks on the way to the toilet. Which priority nursing diagnosis will the nurse include in the patient's plan of care? a. Functional urinary incontinence b. Urge urinary incontinence c. Impaired skin integrity d. Urinary retention

ANS: B Urge urinary incontinence is the leakage of urine associated with a strong urge to void. Patients leak urine on the way to or at the toilet and rush or hurry to the toilet. Urinary retention is the inability to empty the bladder. Functional urinary incontinence is incontinence due to causes outside the urinary tract, such as mobility or cognitive impairments. While Impaired skin integritycan occur, it is not the priority at this time, and there is no data to support this diagnosis.

12. While receiving a shift report on a patient, the nurse is informed that the patient has urinary incontinence. Upon assessment, which finding will the nurse expect? a. An indwelling Foley catheter b. Reddened irritated skin on buttocks c. Tiny blood clots in the patient's urine d. Foul-smelling discharge indicative of infection

ANS: B Urinary incontinence is uncontrolled urinary elimination; if the urine has prolonged contact with the skin, skin breakdown can occur. An indwelling Foley catheter is a solution for urine retention. Blood clots and foul-smelling discharge are often signs of infection.

While the patient's full-thickness burn wounds to the face are exposed, what is the best nursing action to prevent cross contamination? a. Use sterile gloves when removing old dressings. b. Wear gowns, caps, masks, and gloves during all care of the patient. c. Administer IV antibiotics to prevent bacterial colonization of wounds. d. Turn the room temperature up to at least 70° F (20° C) during dressing changes.

ANS: B Use of gowns, caps, masks, and gloves during all patient care will decrease the possibility of wound contamination for a patient whose burns are not covered. When removing contaminated dressings and washing the dirty wound, use nonsterile, disposable gloves. The room temperature should be kept at approximately 85° F for patients with open burn wounds to prevent shivering. Systemic antibiotics are not well absorbed into deep burns because of the lack of circulation

13. A patient who has had progressive chronic kidney disease (CKD) for several years has just begun regular hemodialysis. Which information about diet will the nurse include in patient teaching? a. Increased calories are needed because glucose is lost during hemodialysis. b. More protein is allowed because urea and creatinine are removed by dialysis. c. Dietary potassium is not restricted because the level is normalized by dialysis. d. Unlimited fluids are allowed because retained fluid is removed during dialysis.

ANS: B When the patient is started on dialysis and nitrogenous wastes are removed, more protein in the diet is encouraged. Fluids are still restricted to avoid excessive weight gain and complications such as shortness of breath. Glucose is not lost during hemodialysis. Sodium and potassium intake continues to be restricted to avoid the complications associated with high levels of these electrolytes. DIF: Cognitive Level: Apply (application) REF: 1087 TOP: Nursing Process: Implementation MSC: NCLEX: Physiological Integrity

When assessing a patient who spilled hot oil on the right leg and foot, the nurse notes that the skin is dry, pale, hard skin. The patient states that the burn is not painful. What term would the nurse use to document the burn depth? a. First-degree skin destruction b. Full-thickness skin destruction c. Deep partial-thickness skin destruction d. Superficial partial-thickness skin destruction

ANS: B With full-thickness skin destruction, the appearance is pale and dry or leathery and the area is painless because of the associated nerve destruction. Erythema, swelling, and blisters point to a deep partial-thickness burn. With superficial partial-thickness burns, the area is red, but no blisters are present. First-degree burns exhibit erythema, blanching, and pain.

Which is the initial treatment for the client with vWD who experiences a PPH? a.Cryoprecipitate b.Factor VIII and von Willebrand factor (vWf) c.Desmopressin d.Hemabate

ANS: C Desmopressin is the primary treatment of choice for vWD and can be administered orally, nasally, and intravenously. This medication promotes the release of factor VIII and vWf from storage. Cryoprecipitate may be used; however, because of the risk of possible donor viruses, other modalities are considered safer. Treatment with plasma products such as factor VIII and vWf is an acceptable option for this client. Because of the repeated exposure to donor blood products and possible viruses, this modality is not the initial treatment of choice. Although the administration of the prostaglandin, Hemabate, is known to promote contraction of the uterus during PPH, it is not effective for the client who has a bleeding disorder.

A woman who has recently given birth complains of pain and tenderness in her leg. On physical examination, the nurse notices warmth and redness over an enlarged, hardened area. Which condition should the nurse suspect, and how will it be confirmed? a.Disseminated intravascular coagulation (DIC); asking for laboratory tests b.von Willebrand disease (vWD); noting whether bleeding times have been extended c.Thrombophlebitis; using real-time and color Doppler ultrasound d.Idiopathic or immune thrombocytopenic purpura (ITP); drawing blood for laboratory analysis

ANS: C Pain and tenderness in the extremities, which show warmth, redness, and hardness, is likely thrombophlebitis. A Doppler ultrasound examination is a common noninvasive way to confirm the diagnosis. A diagnosis of DIC is made according to clinical findings and laboratory markers. With DIC, a physical examination will reveal symptoms that may include unusual bleeding, petechiae around a blood pressure cuff on the woman's arm, and/or excessive bleeding from the site of a slight trauma such as a venipuncture site. Symptoms of vWD, a type of hemophilia, include recurrent bleeding episodes, prolonged bleeding time, and factor VIII deficiency. A risk for PPH exists with vWD but does not exhibit a warm or reddened area in an extremity. ITP is an autoimmune disorder in which the life span of antiplatelet antibodies is decreased. Increased bleeding time is a diagnostic finding, and the risk of postpartum uterine bleeding is increased.

The most effective and least expensive treatment of puerperal infection is prevention. What is the most important strategy for the nurse to adopt? a.Large doses of vitamin C during pregnancy b.Prophylactic antibiotics c.Strict aseptic technique, including hand washing, by all health care personnel d.Limited protein and fat intake

ANS: C Strict adherence by all health care personnel to aseptic techniques during childbirth and the postpartum period is extremely important and the least expensive measure to prevent infection. Good nutrition to control anemia is a preventive measure. Increased iron intake assists in preventing anemia. Antibiotics may be administered to manage infections; they are not a cost-effective measure to prevent postpartum infection. Limiting protein and fat intake does not help prevent anemia or prevent infection.

An intraaortic balloon pump (IABP) is being used for a patient who is in cardiogenic shock. Which assessment data indicate to the nurse that the goals of treatment with the IABP are being met? a. Urine output of 25 mL/hr b. Heart rate of 110 beats/minute c. Cardiac output (CO) of 5 L/min d. Stroke volume (SV) of 40 mL/beat

ANS: C A CO of 5 L/min is normal and indicates that the IABP has been successful in treating the shock. The low SV signifies continued cardiogenic shock. The tachycardia and low urine output also suggest continued cardiogenic shock.

30. The nurse will anticipate inserting a Coudé catheter for which patient? a. An 8-year-old male undergoing anesthesia for a tonsillectomy b. A 24-year-old female who is going into labor c. A 56-year-old male admitted for bladder irrigation d. An 86-year-old female admitted for a urinary tract infection

ANS: C A Coudé catheter has a curved tip that is used for patients with enlarged prostates. This would be indicated for a middle-aged male who needs bladder irrigation. Coudé catheters are not indicated for children or women.

19. The nurse is providing preoperative teaching for a 61-year-old man scheduled for an abdominal-perineal resection. Which information will the nurse include? a. Another surgery in 8 to 12 weeks will be used to create an ileal-anal reservoir. b. The patient will begin sitting in a chair at the bedside on the first postoperative day. c. The patient will drink polyethylene glycol lavage solution (GoLYTELY) preoperatively. d. IV antibiotics will be started at least 24 hours before surgery to reduce the bowel bacteria.

ANS: C A bowel-cleansing agent is used to empty the bowel before surgery to reduce the risk for infection. A permanent colostomy is created with this surgery. Sitting is contraindicated after an abdominal-perineal resection. Oral antibiotics (rather than IV antibiotics) are given to reduce colonic and rectal bacteria.

4. A 26-year-old woman is being evaluated for vomiting and abdominal pain. Which question from the nurse will be most useful in determining the cause of the patient's symptoms? a. "What type of foods do you eat?" b. "Is it possible that you are pregnant?" c. "Can you tell me more about the pain?" d. "What is your usual elimination pattern?"

ANS: C A complete description of the pain provides clues about the cause of the problem. Although the nurse should ask whether the patient is pregnant to determine whether the patient might have an ectopic pregnancy and before any radiology studies are done, this information is not the most useful in determining the cause of the pain. The usual diet and elimination patterns are less helpful in determining the reason for the patient's symptoms.

17. A 38-yr-old patient who had a kidney transplant 8 years ago is receiving the immunosuppressants tacrolimus (Prograf), cyclosporine (Sandimmune), and prednisone . Which assessment data will be of most concern to the nurse? a. Skin is thin and fragile. b. Blood pressure is 150/92. c. A nontender axillary lump. d. Blood glucose is 144 mg/dL.

ANS: C A nontender lump suggests a malignancy such as a lymphoma, which could occur as a result of chronic immunosuppressive therapy. The elevated glucose, skin change, and hypertension are possible side effects of the prednisone and should be addressed, but they are not as great a concern as the possibility of a malignancy. DIF: Cognitive Level: Analyze (analysis) REF: 1096 OBJ: Special Questions: Prioritization TOP: Nursing Process: Assessment

A patient with burns covering 40% total body surface area (TBSA) is in the acute phase of burn treatment. Which snack would be best for the nurse to offer to this patient? a. Bananas b. Orange gelatin c. Vanilla milkshake d. Whole grain bagel

ANS: C A patient with a burn injury needs high protein and calorie food intake, and the milkshake is the highest in these nutrients. The other choices are not as nutrient-dense as the milkshake. Gelatin is likely high in sugar. The bagel is a good carbohydrate choice, but low in protein. Bananas are a good source of potassium, but are not high in protein and calories

A 68-year-old patient is being admitted with a possible stroke. Which information from the assessment indicates that the nurse should consult with the health care provider before giving the prescribed aspirin? a. The patient has dysphasia. b. The patient has atrial fibrillation. c. The patient reports that symptoms began with a severe headache. d. The patient has a history of brief episodes of right-sided hemiplegia.

ANS: C A sudden onset headache is typical of a subarachnoid hemorrhage, and aspirin is contraindicated. Atrial fibrillation, dysphasia, and transient ischemic attack (TIA) are not contraindications to aspirin use, so the nurse can administer the aspirin

The nurse is caring for a patient who has been experiencing stroke symptoms for 60 minutes. Which action can the nurse delegate to a licensed practical/vocational nurse (LPN/LVN)? a. Assess the patient's gag and cough reflexes. b. Determine when the stroke symptoms began. c. Administer the prescribed short-acting insulin. d. Infuse the prescribed IV metoprolol (Lopressor).

ANS: C Administration of subcutaneous medications is included in LPN/LVN education and scope of practice. The other actions require more education and scope of practice and should be done by the registered nurse (RN).

A patient with extensive electrical burn injuries is admitted to the emergency department. Which prescribed intervention should the nurse implement first? a. Assess oral temperature. b. Check a potassium level. c. Place on cardiac monitor. d. Assess for pain at contact points.

ANS: C After an electrical burn, the patient is at risk for fatal dysrhythmias and should be placed on a cardiac monitor. Assessing the oral temperature is not as important as assessing for cardiac dysrhythmias. Checking the potassium level is important. However, it will take time before the laboratory results are back. The first intervention is to place the patient on a cardiac monitor and assess for dysrhythmias, so that they can be treated if occurring. A decreased or increased potassium level will alert the nurse to the possibility of dysrhythmias. The cardiac monitor will alert the nurse immediately of any dysrhythmias. Assessing for pain is important, but the patient can endure pain until the cardiac monitor is attached. Cardiac dysrhythmias can be lethal

A patient has just arrived in the emergency department after an electrical burn from exposure to a high-voltage current. What is the priority nursing assessment? a. Oral temperature b. Peripheral pulses c. Extremity movement d. Pupil reaction to light

ANS: C All patients with electrical burns should be considered at risk for cervical spine injury, and assessments of extremity movement will provide baseline data. The other assessment data are also necessary but not as essential as determining the cervical spine status

5. The patient is having lower abdominal surgery and the nurse inserts an indwelling catheter. What is the rationale for the nurse's action? a. The patient may void uncontrollably during the procedure. b. Local trauma sometimes promotes excessive urine incontinence. c. Anesthetics can decrease bladder contractility and cause urinary retention. d. The patient will not interrupt the procedure by asking to go to the bathroom.

ANS: C Anesthetic agents and other agents given during surgery can decrease bladder contractility and/or sensation of bladder fullness, causing urinary retention. Local trauma during lower abdominal and pelvic surgery sometimes obstructs urine flow, requiring temporary use of an indwelling urinary catheter. The patient is more likely to retain urine rather than experience uncontrollable voiding.

16. Which assessment finding may indicate that a patient is experiencing adverse effects to a corticosteroid prescribed after kidney transplantation? a. Postural hypotension b. Recurrent tachycardia c. Knee and hip joint pain d. Increased serum creatinine

ANS: C Aseptic necrosis of the weight-bearing joints can occur when patients take corticosteroids over a prolonged period. Increased creatinine level, orthostatic dizziness, and tachycardia are not caused by corticosteroid use. DIF: Cognitive Level: Apply (application) REF: 1096 TOP: Nursing Process: Evaluation MSC: NCLEX: Physiological Integrity

A 72-year-old patient who has a history of a transient ischemic attack (TIA) has an order for aspirin 160 mg daily. When the nurse is administering medications, the patient says, "I don't need the aspirin today. I don't have a fever." Which action should the nurse take? a. Document that the aspirin was refused by the patient. b. Tell the patient that the aspirin is used to prevent a fever. c. Explain that the aspirin is ordered to decrease stroke risk. d. Call the health care provider to clarify the medication order.

ANS: C Aspirin is ordered to prevent stroke in patients who have experienced TIAs. Documentation of the patient's refusal to take the medication is an inadequate response by the nurse. There is no need to clarify the order with the health care provider. The aspirin is not ordered to prevent aches and pains.

23. A nurse is caring for a patient who just underwent an intravenous pyelography that revealed a renal calculus obstructing the left ureter. What is the nurse's first priority in caring for this patient? a. Turn the patient on the right side to alleviate pressure on the left kidney. b. Encourage the patient to increase fluid intake to flush the obstruction. c. Monitor the patient for fever, rash, and difficulty breathing. d. Administer narcotic medications to the patient for pain.

ANS: C Assess for delayed hypersensitivity to the contrast media. Intravenous pyelography is performed by administering iodine-based dye to view functionality of the urinary system. Therefore, the first nursing priority is to assess the patient for an allergic reaction that could be life threatening. The nurse should then encourage the patient to drink fluids to flush dye resulting from the procedure. Narcotics can be administered but are not the first priority. Turning the patient on the side will not affect patient safety.

1. Which action will the nurse include in the plan of care for a 42-year-old patient who is being admitted with Clostridium difficile? a. Educate the patient about proper food storage. b. Order a diet with no dairy products for the patient. c. Place the patient in a private room on contact isolation. d. Teach the patient about why antibiotics will not be used.

ANS: C Because C. difficile is highly contagious, the patient should be placed in a private room and contact precautions should be used. There is no need to restrict dairy products for this type of diarrhea. Metronidazole (Flagyl) is frequently used to treat C. difficile. Improper food handling and storage do not cause C. difficile.

The nurse identifies the nursing diagnosis of imbalanced nutrition: less than body requirements related to impaired self-feeding ability for a left-handed patient with left-sided hemiplegia. Which intervention should be included in the plan of care? a. Provide a wide variety of food choices. b. Provide oral care before and after meals. c. Assist the patient to eat with the right hand. d. Teach the patient the "chin-tuck" technique.

ANS: C Because the nursing diagnosis indicates that the patient's imbalanced nutrition is related to the left-sided hemiplegia, the appropriate interventions will focus on teaching the patient to use the right hand for self-feeding. The other interventions are appropriate for patients with other etiologies for the imbalanced nutrition.

The nurse notes thick, white secretions in the endotracheal tube (ET) of a patient who is receiving mechanical ventilation. Which intervention will be most effective in addressing this problem? a. Increase suctioning to every hour. b. Reposition the patient every 1 to 2 hours. c. Add additional water to the patient's enteral feedings. d. Instill 5 mL of sterile saline into the ET before suctioning.

ANS: C Because the patient's secretions are thick, better hydration is indicated. Suctioning every hour without any specific evidence for the need will increase the incidence of mucosal trauma and would not address the etiology of the ineffective airway clearance. Instillation of saline does not liquefy secretions and may decrease the SpO2. Repositioning the patient is appropriate but will not decrease the thickness of secretions.

21. A 71-year-old patient had an abdominal-perineal resection for colon cancer. Which nursing action is most important to include in the plan of care for the day after surgery? a. Teach about a low-residue diet. b. Monitor output from the stoma. c. Assess the perineal drainage and incision. d. Encourage acceptance of the colostomy stoma.

ANS: C Because the perineal wound is at high risk for infection, the initial care is focused on assessment and care of this wound. Teaching about diet is best done closer to discharge from the hospital. There will be very little drainage into the colostomy until peristalsis returns. The patient will be encouraged to assist with the colostomy, but this is not the highest priority in the immediate postoperative period.

29. A 62-year-old patient has had a hemorrhoidectomy at an outpatient surgical center. Which instructions will the nurse include in discharge teaching? a. Maintain a low-residue diet until the surgical area is healed. b. Use ice packs on the perianal area to relieve pain and swelling. c. Take prescribed pain medications before a bowel movement is expected. d. Delay having a bowel movement for several days until healing has occurred.

ANS: C Bowel movements may be very painful, and patients may avoid defecation unless pain medication is taken before the bowel movement. A high-residue diet will increase stool bulk and prevent constipation. Delay of bowel movements is likely to lead to constipation. Warm sitz baths rather than ice packs are used to relieve pain and keep the surgical area clean.

15. The nurse suspects cystitis related to a lower urinary tract infection. Which clinical manifestation does the nurse expect the patient to report? a. Dysuria b. Flank pain c. Frequency d. Fever

ANS: C Cystitis is inflammation of the bladder; associated symptoms include hematuria, foul-smelling cloudy urine, and urgency/frequency. Dysuria is a common symptom of a lower urinary tract infection (bladder). Flank pain, fever, and chills are all signs of pyelonephritis (upper urinary tract).

2. A nurse is reviewing urinary laboratory results. Which finding will cause the nurse to follow up? a. Protein level of 2 mg/100 mL b. Urine output of 80 mL/hr c. Specific gravity of 1.036 d. pH of 6.4

ANS: C Dehydration, reduced renal blood flow, and increase in antidiuretic hormone secretion elevate specific gravity. Normal specific gravity is 1.0053 to 1.030. An output of 30 mL/hr or less for 2 or more hours would be cause for concern; 80 mL/hr is normal. The normal pH of urine is between 4.6 and 8.0. Protein up to 8 mg/100 mL is acceptable; however, values in excess of this could indicate renal disease.

40. Which activity in the care of a 48-year-old female patient with a new colostomy could the nurse delegate to unlicensed assistive personnel (UAP)? a. Document the appearance of the stoma. b. Place a pouching system over the ostomy. c. Drain and measure the output from the ostomy. d. Check the skin around the stoma for breakdown.

ANS: C Draining and measuring the output from the ostomy is included in UAP education and scope of practice. The other actions should be implemented by LPNs or RNs.

13. Which diet choice by the patient with an acute exacerbation of inflammatory bowel disease (IBD) indicates a need for more teaching? a. Scrambled eggs b. White toast and jam c. Oatmeal with cream d. Pancakes with syrup

ANS: C During acute exacerbations of IBD, the patient should avoid high-fiber foods such as whole grains. High-fat foods also may cause diarrhea in some patients. The other choices are low residue and would be appropriate for this patient.

When caring for a patient with a new right-sided homonymous hemianopsia resulting from a stroke, which intervention should the nurse include in the plan of care? a. Apply an eye patch to the right eye. b. Approach the patient from the right side. c. Place objects needed on the patient's left side. d. Teach the patient that the left visual deficit will resolve.

ANS: C During the acute period, the nurse should place objects on the patient's unaffected side. Because there is a visual defect in the right half of each eye, an eye patch is not appropriate. The patient should be approached from the left side. The visual deficit may not resolve, although the patient can learn to compensate for the defect.

The nurse notes premature ventricular contractions (PVCs) while suctioning a patient's endotracheal tube. Which action by the nurse is a priority? a. Decrease the suction pressure to 80 mm Hg. b. Document the dysrhythmia in the patient's chart. c. Stop and ventilate the patient with 100% oxygen. d. Give antidysrhythmic medications per protocol.

ANS: C Dysrhythmias during suctioning may indicate hypoxemia or sympathetic nervous system stimulation. The nurse should stop suctioning and ventilate the patient with 100% oxygen. Lowering the suction pressure will decrease the effectiveness of suctioning without improving the hypoxemia. Because the PVCs occurred during suctioning, there is no need for antidysrhythmic medications (which may have adverse effects) unless they recur when the suctioning is stopped and patient is well oxygenated.

10. A 37-yr-old female patient is hospitalized with acute kidney injury (AKI). Which information will be most useful to the nurse in evaluating improvement in kidney function? a. Urine volume b. Creatinine level c. Glomerular filtration rate (GFR) d. Blood urea nitrogen (BUN) level

ANS: C GFR is the preferred method for evaluating kidney function. BUN levels can fluctuate based on factors such as fluid volume status and protein intake. Urine output can be normal or high in patients with AKI and does not accurately reflect kidney function. Creatinine alone is not an accurate reflection of renal function. DIF: Cognitive Level: Analyze (analysis) REF: 1079 TOP: Nursing Process: Evaluation MSC: NCLEX: Physiological Integrity

A patient with severe burns has crystalloid fluid replacement ordered using the Parkland formula. The initial volume of fluid to be administered in the first 24 hours is 30,000 mL. The initial rate of administration is 1875 mL/hr. After the first 8 hours, what rate should the nurse infuse the IV fluids? a. 350 mL/hour b. 523 mL/hour c. 938 mL/hour d. 1250 mL/hour

ANS: C Half of the fluid replacement using the Parkland formula is administered in the first 8 hours and the other half over the next 16 hours. In this case, the patient should receive half of the initial rate, or 938 mL/hr

21. A 55-yr-old patient with end-stage kidney disease (ESKD) is scheduled to receive a prescribed dose of epoetin alfa (Procrit). Which information should the nurse report to the health care provider before giving the medication? a. Creatinine 1.6 mg/dL b. Oxygen saturation 89% c. Hemoglobin level 13 g/dL d. Blood pressure 98/56 mm Hg

ANS: C High hemoglobin levels are associated with a higher rate of thromboembolic events and increased risk of death from serious cardiovascular events (heart attack, heart failure, stroke) when erythropoietin (EPO) is administered to a target hemoglobin of greater than 12 g/dL. Hemoglobin levels higher than 12 g/dL indicate a need for a decrease in epoetin alfa dose. The other information also will be reported to the health care provider but will not affect whether the medication is administered. DIF: Cognitive Level: Apply (application) REF: 1081 TOP: Nursing Process: Assessment MSC: NCLEX: Physiological Integrity

The nurse is reviewing laboratory results on a patient who had a large burn 48 hours ago. Which result requires priority action by the nurse? a. Hematocrit 53% b. Serum sodium 147 mEq/L c. Serum potassium 6.1 mEq/L d. Blood urea nitrogen 37 mg/dL

ANS: C Hyperkalemia can lead to fatal dysrhythmias and indicates that the patient requires cardiac monitoring and immediate treatment to lower the potassium level. The other laboratory values are also abnormal and require changes in treatment, but they are not as immediately life threatening as the elevated potassium level

Which stroke risk factor for a 48-year-old male patient in the clinic is most important for the nurse to address? a. The patient is 25 pounds above the ideal weight. b. The patient drinks a glass of red wine with dinner daily. c. The patient's usual blood pressure (BP) is 170/94 mm Hg. d. The patient works at a desk and relaxes by watching television.

ANS: C Hypertension is the single most important modifiable risk factor. People who drink more than 1 (for women) or 2 (for men) alcoholic beverages a day may increase risk for hypertension. Physical inactivity and obesity contribute to stroke risk but not as much as hypertension.

9. Before administration of calcium carbonate to a patient with chronic kidney disease (CKD), the nurse should check laboratory results for a. potassium level. b. total cholesterol. c. serum phosphate. d. serum creatinine.

ANS: C If serum phosphate is elevated, the calcium and phosphate can cause soft tissue calcification. Calcium carbonate should not be given until the phosphate level is lowered. Total cholesterol, creatinine, and potassium values do not affect whether calcium carbonate should be administered. DIF: Cognitive Level: Apply (application) REF: 1081 TOP: Nursing Process: Implementation MSC: NCLEX: Physiological Integrity

38. Which observation by the nurse best indicates that a continuous bladder irrigation for a patient following genitourinary surgery is effective? a. Output that is smaller than the amount instilled b. Blood clots or sediment in the drainage bag c. Bright red urine turns pink in the tubing d. Bladder distention with tenderness

ANS: C If urine is bright red or has clots, increase irrigation rate until drainage appears pink, indicating successful irrigation. Expect more output than fluid instilled because of urine production. If output is smaller than the amount instilled, suspect that the tube may be clogged. The presence of blood clots indicates the patient is still bleeding, while sediment could mean an infection or bleeding. The bladder should not be distended or tender; the irrigant may not be flowing freely if these occur, or the tube may be kinked or blocked.

9. A nurse is caring for a male patient with urinary retention. Which action should the nurse take first? a. Limit fluid intake. b. Insert a urinary catheter. c. Assist to a standing position. d. Ask for a diuretic medication.

ANS: C In some patients just helping them to a normal position to void prompts voiding. A urinary catheter would relieve urinary retention, but it is not the first measure; other nursing interventions should be tried before catheterization. Reducing fluids would reduce the amount of urine produced but would not alleviate the urine retention and is usually not recommended unless the retention is severe. Diuretic medication would increase urine production and may worsen the discomfort caused by urine retention.

The family members of a patient who has just been admitted to the intensive care unit (ICU) with multiple traumatic injuries have just arrived in the ICU waiting room. Which action should the nurse take next? a. Explain ICU visitation policies and encourage family visits. b. Immediately take the family members to the patient's bedside. c. Describe the patient's injuries and the care that is being provided. d. Invite the family to participate in a multidisciplinary care conference.

ANS: C Lack of information is a major source of anxiety for family members and should be addressed first. Family members should be prepared for the patient's appearance and the ICU environment before visiting the patient for the first time. ICU visiting should be individualized to each patient and family rather than being dictated by rigid visitation policies. Inviting the family to participate in a multidisciplinary conference is appropriate but should not be the initial action by the nurse.

18. The nurse in the dialysis clinic is reviewing the home medications of a patient with chronic kidney disease (CKD). Which medication reported by the patient indicates that patient teaching is required? a. Acetaminophen b. Calcium phosphate c. Magnesium hydroxide d. Multivitamin with iron

ANS: C Magnesium is excreted by the kidneys, and patients with CKD should not use over-the-counter products containing magnesium. The other medications are appropriate for a patient with CKD. DIF: Cognitive Level: Apply (application) REF: 1081 TOP: Nursing Process: Assessment MSC: NCLEX: Physiological Integrity

The nurse is caring for a patient who has an intraaortic balloon pump in place. Which action should be included in the plan of care? a. Position the patient supine at all times. b. Avoid the use of anticoagulant medications. c. Measure the patient's urinary output every hour. d. Provide passive range of motion for all extremities.

ANS: C Monitoring urine output will help determine whether the patient's cardiac output has improved and also help monitor for balloon displacement. The head of the bed can be elevated up to 30 degrees. Heparin is used to prevent thrombus formation. Limited movement is allowed for the extremity with the balloon insertion site to prevent displacement of the balloon.

28. A nurse is caring for an 8-year-old patient who is embarrassed about urinating in bed at night. Which intervention should the nurse suggest to reduce the frequency of this occurrence? a. "Set your alarm clock to wake you every 2 hours, so you can get up to void." b. "Line your bedding with plastic sheets to protect your mattress." c. "Drink your nightly glass of milk earlier in the evening." d. "Empty your bladder completely before going to bed."

ANS: C Nightly incontinence and nocturia are often resolved by limiting fluid intake 2 hours before bedtime. Setting the alarm clock to wake does not correct the physiological problem, nor does lining the bedding with plastic sheets. Emptying the bladder may help with early nighttime urination but will not affect urine produced throughout the night from late-night fluid intake.

An older patient with cardiogenic shock is cool and clammy. Hemodynamic monitoring indicates a high systemic vascular resistance (SVR). Which intervention should the nurse anticipate? a. Increase the rate for the dopamine infusion. b. Decrease the rate for the nitroglycerin infusion. c. Increase the rate for the sodium nitroprusside infusion. d. Decrease the rate for the 5% dextrose in normal saline (D5/.9 NS) infusion.

ANS: C Nitroprusside is an arterial vasodilator and will decrease the SVR and afterload, which will improve cardiac output. Changes in the D5/.9 NS and nitroglycerin infusions will not directly decrease SVR. Increasing the dopamine will tend to increase SVR.

29. A licensed practical/vocational nurse (LPN/LVN) is caring for a patient with stage 2 chronic kidney disease. Which observation by the RN requires an intervention? a. The LPN/LVN administers the erythropoietin subcutaneously. b. The LPN/LVN assists the patient to ambulate out in the hallway. c. The LPN/LVN administers the iron supplement and phosphate binder with lunch. d. The LPN/LVN carries a tray containing low-protein foods into the patient's room.

ANS: C Oral phosphate binders should not be given at the same time as iron because they prevent the iron from being absorbed. The phosphate binder should be given with a meal and the iron given at a different time. The other actions by the LPN/LVN are appropriate for a patient with renal insufficiency. DIF: Cognitive Level: Apply (application) REF: 1082 OBJ: Special Questions: Delegation TOP: Nursing Process: Implementation MSC: NCLEX: Safe and Effective Care Environment

When caring for a patient with pulmonary hypertension, which parameter is most appropriate for the nurse to monitor to evaluate the effectiveness of the treatment? a. Central venous pressure (CVP) b. Systemic vascular resistance (SVR) c. Pulmonary vascular resistance (PVR) d. Pulmonary artery wedge pressure (PAWP)

ANS: C PVR is a major contributor to pulmonary hypertension, and a decrease would indicate that pulmonary hypertension was improving. The other parameters also may be monitored but do not directly assess for pulmonary hypertension.

14. Which action by a patient who is using peritoneal dialysis (PD) indicates that the nurse should provide more teaching about PD? a. The patient leaves the catheter exit site without a dressing. b. The patient plans 30 to 60 minutes for a dialysate exchange. c. The patient cleans the catheter while taking a bath each day. d. The patient slows the inflow rate when experiencing abdominal pain.

ANS: C Patients are encouraged to take showers rather than baths to avoid infections at the catheter insertion side. The other patient actions indicate good understanding of peritoneal dialysis. DIF: Cognitive Level: Apply (application) REF: 1086 TOP: Nursing Process: Evaluation MSC: NCLEX: Physiological Integrity

When the nurse educator is evaluating the skills of a new registered nurse (RN) caring for patients experiencing shock, which action by the new RN indicates a need for more education? a. Placing the pulse oximeter on the ear for a patient with septic shock b. Keeping the head of the bed flat for a patient with hypovolemic shock c. Maintaining a cool room temperature for a patient with neurogenic shock d. Increasing the nitroprusside infusion rate for a patient with a very high SVR

ANS: C Patients with neurogenic shock have poikilothermia. The room temperature should be kept warm to avoid hypothermia. The other actions by the new RN are appropriate.

31. A nurse is evaluating a nursing assistive personnel's (NAP) care for a patient with an indwelling catheter. Which action by the NAP will cause the nurse to intervene? a. Emptying the drainage bag when half full b. Kinking the catheter tubing to obtain a urine specimen c. Placing the drainage bag on the side rail of the patient's bed d. Securing the catheter tubing to the patient's thigh

ANS: C Placing the drainage bag on the side rail of the bed could allow the bag to be raised above the level of the bladder and urine to flow back into the bladder. The urine in the drainage bag is a medium for bacteria; allowing it to reenter the bladder can cause infection. A key intervention to prevent catheter-associated urinary tract infections is prevention of urine back flow from the tubing and bag into the bladder. All the rest are correct procedures and do not require follow-up. The drainage bag should be emptied when half full; an overfull drainage bag can create tension and pulling on the catheter, resulting in trauma to the urethra and/or urinary meatus and increasing risk for urinary tract infections. Urine specimens are obtained by temporarily kinking the tubing; a prolonged kink could lead to bladder distention. Failure to secure the catheter to the patient's thigh places the patient at risk for tissue injury from catheter dislodgment.

8. Which menu choice by the patient who is receiving hemodialysis indicates that the nurse's teaching has been successful? a. Split-pea soup, English muffin, and nonfat milk b. Oatmeal with cream, half a banana, and herbal tea c. Poached eggs, whole-wheat toast, and apple juice d. Cheese sandwich, tomato soup, and cranberry juice

ANS: C Poached eggs would provide high-quality protein, and apple juice is low in potassium. Cheese is high in salt and phosphate, and tomato soup is high in potassium. Split-pea soup is high in potassium, and dairy products are high in phosphate. Bananas are high in potassium, and cream is high in phosphate. DIF: Cognitive Level: Apply (application) REF: 1087 TOP: Nursing Process: Evaluation MSC: NCLEX: Physiological Integrity

To evaluate the effectiveness of the pantoprazole (Protonix) ordered for a patient with systemic inflammatory response syndrome (SIRS), which assessment will the nurse perform? a. Auscultate bowel sounds. c. Check stools for occult blood. b. Ask the patient about nausea. d. Palpate for abdominal tenderness.

ANS: C Proton pump inhibitors are given to decrease the risk for stress ulcers in critically ill patients. The other assessments will also be done, but these will not help in determining the effectiveness of the pantoprazole administration.

38. Four hours after a bowel resection, a 74-year-old male patient with a nasogastric tube to suction complains of nausea and abdominal distention. The first action by the nurse should be to a. auscultate for hypotonic bowel sounds. b. notify the patient's health care provider. c. reposition the tube and check for placement. d. remove the tube and replace it with a new one.

ANS: C Repositioning the tube will frequently facilitate drainage. Because this is a common occurrence, it is not appropriate to notify the health care provider unless other interventions do not resolve the problem. Information about the presence or absence of bowel sounds will not be helpful in improving drainage. Removing the tube and replacing it are unnecessarily traumatic to the patient, so that would only be done if the tube was completely occluded.

16. Which assessment question should the nurse ask if stress incontinence is suspected? a. "Do you think your bladder feels distended?" b. "Do you empty your bladder completely when you void?" c. "Do you experience urine leakage when you cough or sneeze?" d. "Do your symptoms increase with consumption of alcohol or caffeine?"

ANS: C Stress incontinence can be related to intraabdominal pressure causing urine leakage, as would happen during coughing or sneezing. Asking the patient about the fullness of the bladder would rule out retention and overflow. An inability to void completely can refer to urge incontinence. Physiological causes and medications can effect elimination, but this is not related to stress incontinence.

A nurse is weaning a 68-kg male patient who has chronic obstructive pulmonary disease (COPD) from mechanical ventilation. Which patient assessment finding indicates that the weaning protocol should be stopped? a. The patient's heart rate is 97 beats/min. b. The patient's oxygen saturation is 93%. c. The patient respiratory rate is 32 breaths/min. d. The patient's spontaneous tidal volume is 450 mL.

ANS: C Tachypnea is a sign that the patient's work of breathing is too high to allow weaning to proceed. The patient's heart rate is within normal limits, although the nurse should continue to monitor it. An oxygen saturation of 93% is acceptable for a patient with COPD. A spontaneous tidal volume of 450 mL is within the acceptable range.

4. A patient who has acute glomerulonephritis is hospitalized with hyperkalemia. Which information will the nurse monitor to evaluate the effectiveness of the prescribed calcium gluconate IV? a. Urine volume b. Calcium level c. Cardiac rhythm d. Neurologic status

ANS: C The calcium gluconate helps prevent dysrhythmias that might be caused by the hyperkalemia. The nurse will monitor the other data as well, but these will not be helpful in determining the effectiveness of the calcium gluconate. DIF: Cognitive Level: Apply (application) REF: 1073 TOP: Nursing Process: Evaluation MSC: NCLEX: Physiological Integrity

A patient who has neurogenic shock is receiving a phenylephrine infusion through a right forearm IV. Which assessment finding obtained by the nurse indicates a need for immediate action? a. The patient's heart rate is 58 beats/min. b. The patient's extremities are warm and dry. c. The patient's IV infusion site is cool and pale. d. The patient's urine output is 28 mL over the past hour.

ANS: C The coldness and pallor at the infusion site suggest extravasation of the phenylephrine. The nurse should discontinue the IV and, if possible, infuse the drug into a central line. An apical pulse of 58 beats/min is typical for neurogenic shock but does not indicate an immediate need for nursing intervention. A 28-mL urinary output over 1 hour would require the nurse to monitor the output over the next hour, but an immediate change in therapy is not indicated. Warm, dry skin is consistent with early neurogenic shock, but it does not indicate a need for a change in therapy or immediate action.

Which assessment finding obtained by the nurse when caring for a patient receiving mechanical ventilation indicates the need for suctioning? a. The patient's oxygen saturation is 93%. b. The patient was last suctioned 6 hours ago. c. The patient's respiratory rate is 32 breaths/minute. d. The patient has occasional audible expiratory wheezes.

ANS: C The increase in respiratory rate indicates that the patient may have decreased airway clearance and requires suctioning. Suctioning is done when patient assessment data indicate that it is needed, not on a scheduled basis. Occasional expiratory wheezes do not indicate poor airway clearance, and suctioning the patient may induce bronchospasm and increase wheezing. An oxygen saturation of 93% is acceptable and does not suggest that immediate suctioning is needed.

A male patient who has right-sided weakness after a stroke is making progress in learning to use the left hand for feeding and other activities. The nurse observes that when the patient's wife is visiting, she feeds and dresses him. Which nursing diagnosis is most appropriate for the patient? a. Interrupted family processes related to effects of illness of a family member b. Situational low self-esteem related to increasing dependence on spouse for care c. Disabled family coping related to inadequate understanding by patient's spouse d. Impaired nutrition: less than body requirements related to hemiplegia and aphasia

ANS: C The information supports the diagnosis of disabled family coping because the wife does not understand the rehabilitation program. There are no data supporting low self-esteem, and the patient is attempting independence. The data do not support an interruption in family processes because this may be a typical pattern for the couple. There is no indication that the patient has impaired nutrition.

47. A 76-year-old patient with obstipation has a fecal impaction and is incontinent of liquid stool. Which action should the nurse take first? a. Administer bulk-forming laxatives. b. Assist the patient to sit on the toilet. c. Manually remove the impacted stool. d. Increase the patient's oral fluid intake.

ANS: C The initial action with a fecal impaction is manual disimpaction. The other actions will be used to prevent future constipation and impactions.

A patient is admitted to the emergency department (ED) for shock of unknown etiology. The first action by the nurse should be to a. obtain the blood pressure. b. check the level of orientation. c. administer supplemental oxygen. d. obtain a 12-lead electrocardiogram.

ANS: C The initial actions of the nurse are focused on the ABCs—airway, breathing, and circulation—and administration of O2 should be done first. The other actions should be accomplished as rapidly as possible after providing O2.

39. A 19-year-old female is brought to the emergency department with a knife handle protruding from the abdomen. During the initial assessment of the patient, the nurse should a. remove the knife and assess the wound. b. determine the presence of Rovsing sign. c. check for circulation and tissue perfusion. d. insert a urinary catheter and assess for hematuria.

ANS: C The initial assessment is focused on determining whether the patient has hypovolemic shock. The knife should not be removed until the patient is in surgery, where bleeding can be controlled. Rovsing sign is assessed in the patient with suspected appendicitis. A patient with a knife in place will be taken to surgery and assessed for bladder trauma there.

A 70-year-old female patient with left-sided hemiparesis arrives by ambulance to the emergency department. Which action should the nurse take first? a. Monitor the blood pressure. b. Send the patient for a computed tomography (CT) scan. c. Check the respiratory rate and effort. d. Assess the Glasgow Coma Scale score.

ANS: C The initial nursing action should be to assess the airway and take any needed actions to ensure a patent airway. The other activities should take place quickly after the ABCs (airway, breathing, and circulation) are completed.

After reviewing the information shown in the accompanying figure for a patient with pneumonia and sepsis, which information is most important to report to the health care provider? Physical Assessment: Petechiae noted on chest & legs; Crackles heard bilaterally in lung bases; no redness or swelling at central line IV site Lab Data: BUN 34 mg, Hct 30, Plt 50,000 VS: Temp 100; Pulse 102; RR 26; BP 110/60, O2 93 via NC a. Temperature and IV site appearance b. Oxygen saturation and breath sounds c. Platelet count and presence of petechiae d. Blood pressure, pulse rate, respiratory rate.

ANS: C The low platelet count and presence of petechiae suggest that the patient may have disseminated intravascular coagulation and that multiple organ dysfunction syndrome is developing. The other information will also be discussed with the health care provider but does not indicate that the patient's condition is deteriorating or that a change in therapy is needed immediately.

Which action is a priority for the nurse to take when the low pressure alarm sounds for a patient who has an arterial line in the left radial artery? a. Fast flush the arterial line. b. Check the left hand for pallor. c. Assess for cardiac dysrhythmias. d. Rezero the monitoring equipment.

ANS: C The low pressure alarm indicates a drop in the patient's blood pressure, which may be caused by cardiac dysrhythmias. There is no indication to rezero the equipment. Pallor of the left hand would be caused by occlusion of the radial artery by the arterial catheter, not by low pressure. There is no indication of a need for flushing the line.

The nurse notes that a patient's endotracheal tube (ET), which was at the 22-cm mark, is now at the 25-cm mark and the patient is anxious and restless. Which action should the nurse take next? a. Offer reassurance to the patient. b. Bag the patient at an FIO2 of 100%. c. Listen to the patient's breath sounds. d. Notify the patient's health care provider.

ANS: C The nurse should first determine whether the ET tube has been displaced into the right mainstem bronchus by listening for unilateral breath sounds. If so, assistance will be needed to reposition the tube immediately. The other actions are also appropriate, but detection and correction of tube malposition are the most critical actions.

A 47-year-old patient will attempt oral feedings for the first time since having a stroke. The nurse should assess the gag reflex and then a. order a varied pureed diet. b. assess the patient's appetite. c. assist the patient into a chair. d. offer the patient a sip of juice

ANS: C The patient should be as upright as possible before attempting feeding to make swallowing easier and decrease aspiration risk. To assess swallowing ability, the nurse should initially offer water or ice to the patient. Pureed diets are not recommended because the texture is too smooth. The patient may have a poor appetite, but the oral feeding should be attempted regardless.

5. Which statement by a patient with stage 5 chronic kidney disease (CKD) indicates that the nurse's teaching about management of CKD has been effective? a. "I need to get most of my protein from low-fat dairy products." b. "I will increase my intake of fruits and vegetables to 5 per day." c. "I will measure my urinary output each day to help calculate the amount I can drink." d. "I need to take erythropoietin to boost my immune system and help prevent infection."

ANS: C The patient with end-stage renal disease is taught to measure urine output as a means of determining an appropriate oral fluid intake. Erythropoietin is given to increase the red blood cell count and will not offer any benefit for immune function. Dairy products are restricted because of the high phosphate level. Many fruits and vegetables are high in potassium and should be restricted in the patient with CKD. DIF: Cognitive Level: Apply (application) REF: 1082 TOP: Nursing Process: Evaluation MSC: NCLEX: Physiological Integrity

1. After the insertion of an arteriovenous graft (AVG) in the right forearm, a patient complains of pain and coldness of the right fingers. Which action should the nurse take? a. Teach the patient about normal AVG function. b. Remind the patient to take a daily low-dose aspirin tablet. c. Report the patient's symptoms to the health care provider. d. Elevate the patient's arm on pillows to above the heart level.

ANS: C The patient's complaints suggest the development of distal ischemia (steal syndrome) and may require revision of the AVG. Elevation of the arm above the heart will further decrease perfusion. Pain and coolness are not normal after AVG insertion. Aspirin therapy is not used to maintain grafts. DIF: Cognitive Level: Apply (application) REF: 1088 TOP: Nursing Process: Implementation MSC: NCLEX: Physiological Integrity

On admission to the burn unit, a patient with an approximate 25% total body surface area (TBSA) burn has the following initial laboratory results: Hct 58%, Hgb 18.2 mg/dL (172 g/L), serum K+ 4.9 mEq/L (4.8 mmol/L), and serum Na+ 135 mEq/L (135 mmol/L). Which action will the nurse anticipate taking now? a. Monitor urine output every 4 hours. b. Continue to monitor the laboratory results. c. Increase the rate of the ordered IV solution. d. Type and crossmatch for a blood transfusion.

ANS: C The patient's laboratory data show hemoconcentration, which may lead to a decrease in blood flow to the microcirculation unless fluid intake is increased. Because the hematocrit and hemoglobin are elevated, a transfusion is inappropriate, although transfusions may be needed after the emergent phase once the patient's fluid balance has been restored. On admission to a burn unit, the urine output would be monitored more often than every 4 hours; likely every1 hour

After a patient experienced a brief episode of tinnitus, diplopia, and dysarthria with no residual effects, the nurse anticipates teaching the patient about a. cerebral aneurysm clipping. b. heparin intravenous infusion. c. oral low-dose aspirin therapy. d. tissue plasminogen activator (tPA).

ANS: C The patient's symptoms are consistent with transient ischemic attack (TIA), and drugs that inhibit platelet aggregation are prescribed after a TIA to prevent stroke. Continuous heparin infusion is not routinely used after TIA or with acute ischemic stroke. The patient's symptoms are not consistent with a cerebral aneurysm. tPA is used only for acute ischemic stroke, not for TIA.

49. A 72-year-old male patient with dehydration caused by an exacerbation of ulcerative colitis is receiving 5% dextrose in normal saline at 125 mL/hour. Which assessment finding by the nurse is most important to report to the health care provider? a. Patient has not voided for the last 4 hours. b. Skin is dry with poor turgor on all extremities. c. Crackles are heard halfway up the posterior chest. d. Patient has had 5 loose stools over the last 6 hours.

ANS: C The presence of crackles in an older patient receiving IV fluids at a high rate suggests volume overload and a need to reduce the rate of the IV infusion. The other data will also be reported, but are consistent with the patient's age and diagnosis and do not require a change in the prescribed treatment.

10. Upon palpation, the nurse notices that the bladder is firm and distended; the patient expresses an urge to urinate. Which question is most appropriate? a. "Does your urinary problem interfere with any activities?" b. "Do you lose urine when you cough or sneeze?" c. "When was the last time you voided?" d. "Are you experiencing a fever or chills?"

ANS: C To obtain an accurate assessment, the nurse should first determine the source of the discomfort. Urinary retention causes the bladder to be firm and distended; time of last void is most appropriate. Further assessment to determine the pathology of the condition can be performed later. Questions concerning fever and chills, interference with any activities, and losing urine during coughing or sneezing focus on specific pathological conditions.

42. Which care activity for a patient with a paralytic ileus is appropriate for the registered nurse (RN) to delegate to unlicensed assistive personnel (UAP)? a. Auscultation for bowel sounds b. Nasogastric (NG) tube irrigation c. Applying petroleum jelly to the lips d. Assessment of the nares for irritation

ANS: C UAP education and scope of practice include patient hygiene such as oral care. The other actions require education and scope of practice appropriate to the RN.

The nurse is caring for a patient receiving a continuous norepinephrine (Levophed) IV infusion. Which patient assessment finding indicates that the infusion rate may need to be adjusted? a. Heart rate is 58 beats/minute. b. Mean arterial pressure (MAP) is 56 mm Hg. c. Systemic vascular resistance (SVR) is elevated. d. Pulmonary artery wedge pressure (PAWP) is low.

ANS: C Vasoconstrictors such as norepinephrine (Levophed) will increase SVR, and this will increase the work of the heart and decrease peripheral perfusion. The infusion rate may need to be decreased. Bradycardia, hypotension (MAP of 56 mm Hg), and low PAWP are not associated with norepinephrine infusion.

During the change of shift report a nurse is told that a patient has an occluded left posterior cerebral artery. The nurse will anticipate that the patient may have a. dysphasia. b. confusion. c. visual deficits. d. poor judgment

ANS: C Visual disturbances are expected with posterior cerebral artery occlusion. Aphasia occurs with middle cerebral artery involvement. Cognitive deficits and changes in judgment are more typical of anterior cerebral artery occlusion

A nurse is assessing a patient who is receiving a nitroprusside infusion to treat cardiogenic shock. Which finding indicates that the drug is effective? a. No new heart murmurs c. Warm, pink, and dry skin b. Decreased troponin level d. Blood pressure of 92/40 mm Hg

ANS: C Warm, pink, and dry skin indicates that perfusion to tissues is improved. Because nitroprusside is a vasodilator, the blood pressure may be low even if the drug is effective. Absence of a heart murmur and a decrease in troponin level are not indicators of improvement in shock.

20. A patient with diabetes who has bacterial pneumonia is being treated with IV gentamicin 60 mg IV BID. The nurse will monitor for adverse effects of the medication by evaluating the patient's a. blood glucose. b. urine osmolality. c. serum creatinine. d. serum potassium.

ANS: C When a patient at risk for chronic kidney disease (CKD) receives a potentially nephrotoxic medication, it is important to monitor renal function with BUN and creatinine levels. The other laboratory values would not be useful in assessing for the adverse effects of the gentamicin. DIF: Cognitive Level: Apply (application) REF: 1083 TOP: Nursing Process: Evaluation MSC: NCLEX: Physiological Integrity

After receiving 2 L of normal saline, the central venous pressure for a patient who has septic shock is 10 mm Hg, but the blood pressure is still 82/40 mm Hg. The nurse will anticipate an order for a. furosemide . c. norepinephrine . b. nitroglycerin . d. sodium nitroprusside .

ANS: C When fluid resuscitation is unsuccessful, vasopressor drugs are given to increase the systemic vascular resistance (SVR) and blood pressure and improve tissue perfusion. Furosemide would cause diuresis and further decrease the BP. Nitroglycerin would decrease the preload and further drop cardiac output and BP. Nitroprusside is an arterial vasodilator and would further decrease SVR.

12. A 22-year-old female patient with an exacerbation of ulcerative colitis is having 15 to 20 stools daily and has excoriated perianal skin. Which patient behavior indicates that teaching regarding maintenance of skin integrity has been effective? a. The patient uses incontinence briefs to contain loose stools. b. The patient asks for antidiarrheal medication after each stool. c. The patient uses witch hazel compresses to decrease irritation. d. The patient cleans the perianal area with soap after each stool.

ANS: C Witch hazel compresses are suggested to reduce anal irritation and discomfort. Incontinence briefs may trap diarrhea and increase the incidence of skin breakdown. Antidiarrheal medications are not given 15 to 20 times a day. The perianal area should be washed with plain water after each stool.

If nonsurgical treatment for late PPH is ineffective, which surgical procedure would be appropriate to correct the cause of this condition? a.Hysterectomy b.Laparoscopy c.Laparotomy d.Dilation and curettage (D&C)

ANS: D D&C allows the examination of the uterine contents and the removal of any retained placental fragments or blood clots. Hysterectomy is the removal of the uterus and is not the appropriate treatment for late PPH. A laparoscopy is the insertion of an endoscope through the abdominal wall to examine the peritoneal cavity, but it, too, is not the appropriate treatment for this condition. A laparotomy is the surgical incision into the peritoneal cavity to explore it but is also not the appropriate treatment for late PPH.

Nurses need to understand the basic definitions and incidence data regarding PPH. Which statement regarding this condition is most accurate? a.PPH is easy to recognize early; after all, the woman is bleeding. b.Traditionally, it takes more than 1000 ml of blood after vaginal birth and 2500 ml after cesarean birth to define the condition as PPH. c.If anything, nurses and physicians tend to overestimate the amount of blood loss. d.Traditionally, PPH has been classified as early PPH or late PPH with respect to birth.

ANS: D Early PPH is also known as primary, or acute, PPH; late PPH is known as secondary PPH. Unfortunately, PPH can occur with little warning and is often recognized only after the mother has profound symptoms. Traditionally, a 500-ml blood loss after a vaginal birth and a 1000-ml blood loss after a cesarean birth constitute PPH. Medical personnel tend to underestimate blood loss by as much as 50% in their subjective observations.

The nurse suspects that her postpartum client is experiencing hemorrhagic shock. Which observation indicates or would confirm this diagnosis? a.Absence of cyanosis in the buccal mucosa b.Cool, dry skin c.Calm mental status d.Urinary output of at least 30 ml/hr

ANS: D Hemorrhage may result in hemorrhagic shock. Shock is an emergency situation during which the perfusion of body organs may become severely compromised, and death may occur. The presence of adequate urinary output indicates adequate tissue perfusion. The assessment of the buccal mucosa for cyanosis can be subjective. The presence of cool, pale, clammy skin is associated with hemorrhagic shock. Hemorrhagic shock is associated with lethargy, not restlessness.

Which classification of placental separation is not recognized as an abnormal adherence pattern? a.Placenta accreta b.Placenta increta c.Placenta percreta d.Placenta abruptio

ANS: D Placenta abruptio is premature separation of the placenta as opposed to partial or complete adherence. This classification occurs between the 20th week of gestation and delivery in the area of the decidua basalis. Symptoms include localized pain and bleeding. Placenta accreta is a recognized degree of attachment. With placenta accreta, the trophoblast slightly penetrates into the myometrium. Placenta increta is a recognized degree of attachment that results in deep penetration of the myometrium. Placenta percreta is the most severe degree of placental penetration that results in deep penetration of the myometrium. Bleeding with complete placental attachment occurs only when separation of the placenta is attempted after delivery. Treatment includes blood component therapy and, in extreme cases, hysterectomy may be necessary.

What is one of the initial signs and symptoms of puerperal infection in the postpartum client? a.Fatigue continuing for longer than 1 week b.Pain with voiding c.Profuse vaginal lochia with ambulation d.Temperature of 38° C (100.4° F) or higher on 2 successive days

ANS: D Postpartum or puerperal infection is any clinical infection of the genital canal that occurs within 28 days after miscarriage, induced abortion, or childbirth. The definition used in the United States continues to be the presence of a fever of 38° C (100.4° F) or higher on 2 successive days of the first 10 postpartum days, starting 24 hours after birth. Fatigue is a late finding associated with infection. Pain with voiding may indicate a urinary tract infection (UTI), but it is not typically one of the earlier symptoms of infection. Profuse lochia may be associated with endometritis, but it is not the first symptom associated with infection.

3. A 64-year-old woman who has chronic constipation asks the nurse about the use of psyllium (Metamucil). Which information will the nurse include in the response? a. Absorption of fat-soluble vitamins may be reduced by fiber-containing laxatives. b. Dietary sources of fiber should be eliminated to prevent excessive gas formation. c. Use of this type of laxative to prevent constipation does not cause adverse effects. d. Large amounts of fluid should be taken to prevent impaction or bowel obstruction.

ANS: D A high fluid intake is needed when patients are using bulk-forming laxatives to avoid worsening constipation. Although bulk-forming laxatives are generally safe, the nurse should emphasize the possibility of constipation or obstipation if inadequate fluid intake occurs. Although increased gas formation is likely to occur with increased dietary fiber, the patient should gradually increase dietary fiber and eventually may not need the psyllium. Fat-soluble vitamin absorption is blocked by stool softeners and lubricants, not by bulk-forming laxatives.

The emergency department (ED) nurse receives report that a seriously injured patient involved in a motor vehicle crash is being transported to the facility with an estimated arrival in 5 minutes. In preparation for the patient's arrival, the nurse will obtain a. a dopamine infusion. c. lactated Ringer's solution. b. a hypothermia blanket. d. two 16-gauge IV catheters.

ANS: D A patient with multiple trauma may require fluid resuscitation to prevent or treat hypovolemic shock, so the nurse will anticipate the need for 2 large-bore IV lines to administer normal saline. Lactated Ringer's solution should be used cautiously and will not be ordered until the patient has been assessed for possible liver abnormalities. Vasopressor infusion is not used as the initial therapy for hypovolemic shock. Patients in shock need to be kept warm not cool.

27. A 42-year-old male patient has had a herniorrhaphy to repair an incarcerated inguinal hernia. Which patient teaching will the nurse provide before discharge? a. Soak in sitz baths several times each day. b. Cough 5 times each hour for the next 48 hours. c. Avoid use of acetaminophen (Tylenol) for pain. d. Apply a scrotal support and ice to reduce swelling.

ANS: D A scrotal support and ice are used to reduce edema and pain. Coughing will increase pressure on the incision. Sitz baths will not relieve pain and would not be of use after this surgery. Acetaminophen can be used for postoperative pain.

17. A 73-year-old patient with diverticulosis has a large bowel obstruction. The nurse will monitor for a. referred back pain. b. metabolic alkalosis. c. projectile vomiting. d. abdominal distention.

ANS: D Abdominal distention is seen in lower intestinal obstruction. Referred back pain is not a common clinical manifestation of intestinal obstruction. Metabolic alkalosis is common in high intestinal obstruction because of the loss of HCl acid from vomiting. Projectile vomiting is associated with higher intestinal obstruction.

A patient who has burns on the arms, legs, and chest from a house fire has become agitated and restless 8 hours after being admitted to the hospital. Which action should the nurse take first? a. Stay at the bedside and reassure the patient. b. Administer the ordered morphine sulfate IV. c. Assess orientation and level of consciousness. d. Use pulse oximetry to check the oxygen saturation.

ANS: D Agitation in a patient who may have suffered inhalation injury might indicate hypoxia, and this should be assessed by the nurse first. Administration of morphine may be indicated if the nurse determines that the agitation is caused by pain. Assessing level of consciousness and orientation is also appropriate but not as essential as determining whether the patient is hypoxemic. Reassurance is not helpful to reduce agitation in a hypoxemic patient

Which action will the nurse include in the plan of care for a patient in the rehabilitation phase after a burn injury to the right arm and chest? a. Keep the right arm in a position of comfort. b. Avoid the use of sustained-release narcotics. c. Teach about the purpose of tetanus immunization. d. Apply water-based cream to burned areas frequently.

ANS: D Application of water-based emollients will moisturize new skin and decrease flakiness and itching. To avoid contractures, the joints of the right arm should be positioned in an extended position, which is not the position of comfort. Patients may need to continue the use of opioids during rehabilitation. Tetanus immunization would have been given during the emergent phase of the burn injury

28. Which breakfast choice indicates a patient's good understanding of information about a diet for celiac disease? a. Oatmeal with nonfat milk b. Whole wheat toast with butter c. Bagel with low-fat cream cheese d. Corn tortilla with scrambled eggs

ANS: D Avoidance of gluten-containing foods is the only treatment for celiac disease. Corn does not contain gluten, while oatmeal and wheat do.

19. A nurse is reviewing results from a urine specimen. What will the nurse expect to see in a patient with a urinary tract infection? a. Casts b. Protein c. Crystals d. Bacteria

ANS: D Bacteria in the urine along with other symptoms support a diagnosis of urinary tract infection. Crystals would be seen with renal stone formation. Casts indicate renal disease. Protein indicates kidney function and damage to the glomerular membrane such as in glomerulonephritis.

A patient admitted with possible stroke has been aphasic for 3 hours and his current blood pressure (BP) is 174/94 mm Hg. Which order by the health care provider should the nurse question? a. Keep head of bed elevated at least 30 degrees. b. Infuse normal saline intravenously at 75 mL/hr. c. Administer tissue plasminogen activator (tPA) per protocol. d. Administer a labetalol (Normodyne) drip to keep BP less than 140/90 mm Hg.

ANS: D Because elevated BP may be a protective response to maintain cerebral perfusion, antihypertensive therapy is recommended only if mean arterial pressure (MAP) is >130 mm Hg or systolic pressure is >220 mm Hg. Fluid intake should be 1500 to 2000 mL daily to maintain cerebral blood flow. The head of the bed should be elevated to at least 30 degrees, unless the patient has symptoms of poor tissue perfusion. tPA may be administered if the patient meets the other criteria for tPA use.

Which assessment information is most important for the nurse to obtain when evaluating whether treatment of a patient with anaphylactic shock has been effective? a. Heart rate c. Blood pressure b. Orientation d. Oxygen saturation

ANS: D Because the airway edema that is associated with anaphylaxis can affect airway and breathing, the O2 saturation is the most critical assessment. Improvements in the other assessments will also be expected with effective treatment of anaphylactic shock.

When teaching about clopidogrel (Plavix), the nurse will tell the patient with cerebral atherosclerosis a. to monitor and record the blood pressure daily. b. that Plavix will dissolve clots in the cerebral arteries. c. that Plavix will reduce cerebral artery plaque formation. d. to call the health care provider if stools are bloody or tarry.

ANS: D Clopidogrel (Plavix) inhibits platelet function and increases the risk for gastrointestinal bleeding, so patients should be advised to notify the health care provider about any signs of bleeding. The medication does not lower blood pressure, decrease plaque formation, or dissolve clots

31. The nurse will plan to teach a patient with Crohn's disease who has megaloblastic anemia about the need for a. oral ferrous sulfate tablets. b. regular blood transfusions. c. iron dextran (Imferon) infusions. d. cobalamin (B12) spray or injections.

ANS: D Crohn's disease frequently affects the ileum, where absorption of cobalamin occurs. Cobalamin must be administered regularly by nasal spray or IM to correct the anemia. Iron deficiency does not cause megaloblastic anemia. The patient may need occasional transfusions but not regularly scheduled transfusions.

Which action will the nurse need to do when preparing to assist with the insertion of a pulmonary artery catheter? a. Determine if the cardiac troponin level is elevated. b. Auscultate heart and breath sounds during insertion. c. Place the patient on NPO status before the procedure. d. Attach cardiac monitoring leads before the procedure.

ANS: D Dysrhythmias can occur as the catheter is floated through the right atrium and ventricle, and it is important for the nurse to monitor for these during insertion. Pulmonary artery catheter insertion does not require anesthesia, and the patient will not need to be NPO. Changes in cardiac troponin or heart and breath sounds are not expected during pulmonary artery catheter insertion.

To verify the correct placement of an oral endotracheal tube (ET) after insertion, the best initial action by the nurse is to a. auscultate for the presence of bilateral breath sounds. b. obtain a portable chest x-ray to check tube placement. c. observe the chest for symmetric chest movement with ventilation. d. use an end-tidal CO2 monitor to check for placement in the trachea.

ANS: D End-tidal CO2 monitors are currently recommended for rapid verification of ET placement. Auscultation for bilateral breath sounds and checking chest expansion are also used, but they are not as accurate as end-tidal CO2 monitoring. A chest x-ray confirms the placement but is done after the tube is secured.

21. Which clinical manifestation will the nurse expect to observe in a patient with excessive white blood cells present in the urine? a. Reduced urine specific gravity b. Increased blood pressure c. Abnormal blood sugar d. Fever with chills

ANS: D Fever and chills may be observed. The presence of white blood cells in urine indicates a urinary tract infection or inflammation. Overhydration, early renal disease, and inadequate antidiuretic hormone secretion reduce specific gravity. Increased blood pressure is associated with renal disease or damage and some medications. Abnormal blood sugars would be seen in someone with ketones in the urine or a patient with diabetes.

Esomeprazole (Nexium) is prescribed for a patient who incurred extensive burn injuries 5 days ago. Which nursing assessment would best evaluate the effectiveness of the medication? a. Bowel sounds b. Stool frequency c. Abdominal distention d. Stools for occult blood

ANS: D H2 blockers and proton pump inhibitors are given to prevent Curling's ulcer in the patient who has suffered burn injuries. Proton pump inhibitors usually do not affect bowel sounds, stool frequency, or appetite

13. A nurse is inserting a catheter into a female patient. When the nurse inserts the catheter, no urine is obtained. The nurse suspects the catheter is not in the urethra. What should the nurse do? a. Throw the catheter way and begin again. b. Fill the balloon with the recommended sterile water. c. Remove the catheter, wipe with alcohol, and reinsert after lubrication. d. Leave the catheter in the vagina as a landmark for insertion of a new, sterile catheter.

ANS: D If no urine appears, the catheter may be in the vagina. If misplaced, leave the catheter in the vagina as a landmark to indicate where not to insert, and insert another sterile catheter. The catheter should be left in place until the new, sterile catheter is inserted. The balloon should not be filled since the catheter is in the vagina. The catheter must be sterile; using alcohol will not make the catheter sterile.

The nurse is caring for a patient with a subarachnoid hemorrhage who is intubated and placed on a mechanical ventilator with 10 cm H2O of peak end-expiratory pressure (PEEP). When monitoring the patient, the nurse will need to notify the health care provider immediately if the patient develops a. oxygen saturation of 93%. b. respirations of 20 breaths/minute. c. green nasogastric tube drainage. d. increased jugular venous distention.

ANS: D Increases in jugular venous distention in a patient with a subarachnoid hemorrhage may indicate an increase in intracranial pressure (ICP) and that the PEEP setting is too high for this patient. A respiratory rate of 20, O2 saturation of 93%, and green nasogastric tube drainage are within normal limits.

34. A patient complains of leg cramps during hemodialysis. The nurse should a. massage the patient's legs. b. reposition the patient supine. c. give acetaminophen (Tylenol). d. infuse a bolus of normal saline.

ANS: D Muscle cramps during dialysis are caused by rapid removal of sodium and water. Treatment includes infusion of normal saline. The other actions do not address the reason for the cramps. DIF: Cognitive Level: Apply (application) REF: 1091 TOP: Nursing Process: Implementation MSC: NCLEX: Physiological Integrity

34. Which question from the nurse would help determine if a patient's abdominal pain might indicate irritable bowel syndrome? a. "Have you been passing a lot of gas?" b. "What foods affect your bowel patterns?" c. "Do you have any abdominal distention?" d. "How long have you had abdominal pain?"

ANS: D One criterion for the diagnosis of irritable bowel syndrome (IBS) is the presence of abdominal discomfort or pain for at least 3 months. Abdominal distention, flatulence, and food intolerance are also associated with IBS, but are not diagnostic criteria.

The nurse is reviewing the medication administration record (MAR) on a patient with partial-thickness burns. Which medication is best for the nurse to administer before scheduled wound debridement? a. Ketorolac (Toradol) b. Lorazepam (Ativan) c. Gabapentin (Neurontin) d. Hydromorphone (Dilaudid)

ANS: D Opioid pain medications are the best choice for pain control. The other medications are used as adjuvants to enhance the effects of opioids

When monitoring for the effectiveness of treatment for a patient with a large anterior wall myocardial infarction, the most important information for the nurse to obtain is a. central venous pressure (CVP). b. systemic vascular resistance (SVR). c. pulmonary vascular resistance (PVR). d. pulmonary artery wedge pressure (PAWP).

ANS: D PAWP reflects left ventricular end diastolic pressure (or left ventricular preload) and is a sensitive indicator of cardiac function. Because the patient is high risk for left ventricular failure, the PAWP must be monitored. An increase will indicate left ventricular failure. The other values would also provide useful information, but the most definitive measurement of changes in cardiac function is the PAWP.

A 58-year-old patient with a left-brain stroke suddenly bursts into tears when family members visit. The nurse should a. use a calm voice to ask the patient to stop the crying behavior. b. explain to the family that depression is normal following a stroke. c. have the family members leave the patient alone for a few minutes. d. teach the family that emotional outbursts are common after strokes.

ANS: D Patients who have left-sided brain stroke are prone to emotional outbursts that are not necessarily related to the emotional state of the patient. Depression after a stroke is common, but the suddenness of the patient's outburst suggests that depression is not the major cause of the behavior. The family should stay with the patient. The crying is not within the patient's control and asking the patient to stop will lead to embarrassment.

Nurses in change-of-shift report are discussing the care of a patient with a stroke who has progressively increasing weakness and decreasing level of consciousness (LOC). Which nursing diagnosis do they determine has the highest priority for the patient? a. Impaired physical mobility related to weakness b. Disturbed sensory perception related to brain injury c. Risk for impaired skin integrity related to immobility d. Risk for aspiration related to inability to protect airway

ANS: D Protection of the airway is the priority of nursing care for a patient having an acute stroke. The other diagnoses are also appropriate, but interventions to prevent aspiration are the priority at this time.

A patient with left-sided weakness that started 60 minutes earlier is admitted to the emergency department and diagnostic tests are ordered. Which test should be done first? a. Complete blood count (CBC) b. Chest radiograph (Chest x-ray) c. 12-Lead electrocardiogram (ECG) d. Noncontrast computed tomography (CT) scan

ANS: D Rapid screening with a noncontrast CT scan is needed before administration of tissue plasminogen activator (tPA), which must be given within 4.5 hours of the onset of clinical manifestations of the stroke. The sooner the tPA is given, the less brain injury. The other diagnostic tests give information about possible causes of the stroke and do not need to be completed as urgently as the CT scan.

A 73-year-old patient with a stroke experiences facial drooping on the right side and right-sided arm and leg paralysis. When admitting the patient, which clinical manifestation will the nurse expect to find? a. Impulsive behavior b. Right-sided neglect c. Hyperactive left-sided tendon reflexes d. Difficulty comprehending instructions

ANS: D Right-sided paralysis indicates a left-brain stroke, which will lead to difficulty with comprehension and use of language. The left-side reflexes are likely to be intact. Impulsive behavior and neglect are more likely with a right-side stroke.

After change-of-shift report on a ventilator weaning unit, which patient should the nurse assess first? a. Patient who failed a spontaneous breathing trial and has been placed in a rest mode on the ventilator b. Patient who is intubated and has continuous partial pressure end-tidal CO2 (PETCO2) monitoring c. Patient with a central venous oxygen saturation (ScvO2) of 69% while on bilevel positive airway pressure (BiPAP) d. Patient who was successfully weaned and extubated 4 hours ago and now has no urine output for the last 6 hours

ANS: D The decreased urine output may indicate acute kidney injury or that the patient's cardiac output and perfusion of vital organs have decreased. Any of these causes would require rapid action. The data about the other patients indicate that their conditions are stable and do not require immediate assessment or changes in their care. Continuous PETCO2 monitoring is frequently used when patients are intubated. The rest mode should be used to allow patient recovery after a failed SBT, and an ScvO2 of 69% is within normal limits.

After change-of-shift report, which patient should the progressive care nurse assess first? a. Patient who was extubated in the morning and has a temperature of 101.4° F (38.6° C) b. Patient with bilevel positive airway pressure (BiPAP) for sleep apnea whose respiratory rate is 16 c. Patient with arterial pressure monitoring who is 2 hours post-percutaneous coronary intervention who needs to void d. Patient who is receiving IV heparin for a venous thromboembolism and has a partial thromboplastin time (PTT) of 98 sec

ANS: D The findings for this patient indicate high risk for bleeding from an elevated (nontherapeutic) PTT. The nurse needs to adjust the rate of the infusion (dose) per the health care provider's parameters. The patient with BiPAP for sleep apnea has a normal respiratory rate. The patient recovering from the percutaneous coronary intervention will need to be assisted with voiding and this task could be delegated to unlicensed assistive personnel. The patient with a fever may be developing ventilator-associated pneumonia, but addressing the bleeding risk is a higher priority.

The nurse educator is evaluating the care that a new registered nurse (RN) provides to a patient receiving mechanical ventilation. Which action by the new RN indicates the need for more education? a. The RN increases the FIO2 to 100% before suctioning. b. The RN secures a bite block in place using adhesive tape. c. The RN asks for assistance to reposition the endotracheal tube. d. The RN positions the patient with the head of bed at 10 degrees.

ANS: D The head of the patient's bed should be positioned at 30 to 45 degrees to prevent ventilator-associated pneumonia. The other actions by the new RN are appropriate.

26. A patient with acute kidney injury (AKI) has longer QRS intervals on the electrocardiogram (ECG) than were noted on the previous shift. Which action should the nurse take first? a. Notify the patient's health care provider. b. Document the QRS interval measurement. c. Review the chart for the patient's current creatinine level. d. Check the medical record for the most recent potassium level.

ANS: D The increasing QRS interval is suggestive of hyperkalemia, so the nurse should check the most recent potassium and then notify the patient's health care provider. The BUN and creatinine will be elevated in a patient with AKI, but they would not directly affect the electrocardiogram (ECG). Documentation of the QRS interval is also appropriate, but interventions to decrease the potassium level are needed to prevent life-threatening dysrhythmias. DIF: Cognitive Level: Analyze (analysis) REF: 1072 OBJ: Special Questions: Prioritization TOP: Nursing Process: Implementation MSC: NCLEX: Physiological Integrity

While waiting for cardiac transplantation, a patient with severe cardiomyopathy has a ventricular assist device (VAD) implanted. When planning care for this patient, the nurse should anticipate a. giving immunosuppressive medications. b. preparing the patient for a permanent VAD. c. teaching the patient the reason for complete bed rest. d. monitoring the surgical incision for signs of infection.

ANS: D The insertion site for the VAD provides a source for transmission of infection to the circulatory system and requires frequent monitoring. Patient's with VADs are able to have some mobility and may not be on bed rest. The VAD is a bridge to transplantation, not a permanent device. Immunosuppression is not necessary for nonbiologic devices like the VAD.

To maintain proper cuff pressure of an endotracheal tube (ET) when the patient is on mechanical ventilation, the nurse should a. inflate the cuff with a minimum of 10 mL of air. b. inflate the cuff until the pilot balloon is firm on palpation. c. inject air into the cuff until a manometer shows 15 mm Hg pressure. d. inject air into the cuff until a slight leak is heard only at peak inflation.

ANS: D The minimal occluding volume technique involves injecting air into the cuff until an air leak is present only at peak inflation. The volume to inflate the cuff varies with the ET and the patient's size. Cuff pressure should be maintained at 20 to 25 mm Hg. An accurate assessment of cuff pressure cannot be obtained by palpating the pilot balloon.

The nurse responds to a ventilator alarm and finds the patient lying in bed holding the endotracheal tube (ET). Which action should the nurse take next? a. Activate the rapid response team. b. Provide reassurance to the patient. c. Call the health care provider to reinsert the tube. d. Manually ventilate the patient with 100% oxygen.

ANS: D The nurse should ensure maximal patient oxygenation by manually ventilating with a bag-valve-mask system. Offering reassurance to the patient, notifying the health care provider about the need to reinsert the tube, and activating the rapid response team are also appropriate after the nurse has stabilized the patient's oxygenation.

26. A nurse is watching a nursing assistive personnel (NAP) perform a postvoid bladder scan on a female with a previous hysterectomy. Which action will require the nurse to follow up? a. Palpates the patient's symphysis pubis b. Wipes scanner head with alcohol pad c. Applies a generous amount of gel d. Sets the scanner to female

ANS: D The nurse will follow up if the NAP sets the scanner to female. Women who have had a hysterectomy should be designated as male. All the rest are correct and require no follow-up. The NAP should palpate the symphysis pubis, the scanner head should be cleaned with an alcohol pad, and a generous amount of gel should be applied.

18. To obtain a clean-voided urine specimen from a female patient, what should the nurse teach the patient to do? a. Cleanse the urethral meatus from the area of most contamination to least. b. Initiate the first part of the urine stream directly into the collection cup. c. Drink fluids 5 minutes before collecting the urine specimen. d. Hold the labia apart while voiding into the specimen cup.

ANS: D The patient should hold the labia apart to reduce bacterial levels in the specimen. The urethral meatus should be cleansed from the area of least contamination to greatest contamination (or front to back). The initial stream flushes out microorganisms in the urethra and prevents bacterial transmission in the specimen. Drink fluids 30 minutes before giving a specimen.

36. After receiving change-of-shift report, which patient should the nurse assess first? a. Patient who is scheduled for the drain phase of a peritoneal dialysis exchange b. Patient with stage 4 chronic kidney disease who has an elevated phosphate level c. Patient with stage 5 chronic kidney disease who has a potassium level of 3.4 mEq/L d. Patient who has just returned from having hemodialysis and has a heart rate of 124/min

ANS: D The patient who has tachycardia after hemodialysis may be bleeding or excessively hypovolemic and should be assessed immediately for these complications. The other patients also need assessments or interventions but are not at risk for life-threatening complications. DIF: Cognitive Level: Analyze (analysis) REF: 1091 OBJ: Special Questions: Prioritization | Special Questions: Multiple Patients TOP: Nursing Process: Assessment MSC: NCLEX: Safe and Effective Care Environment

Four hours after mechanical ventilation is initiated for a patient with chronic obstructive pulmonary disease (COPD), the patient's arterial blood gas (ABG) results include a pH of 7.51, PaO2 of 82 mm Hg, PaCO2 of 26 mm Hg, and HCO3- of 23 mEq/L (23 mmol/L). The nurse will anticipate the need to a. increase the FIO2. b. increase the tidal volume. c. increase the respiratory rate. d. decrease the respiratory rate.

ANS: D The patient's PaCO2 and pH indicate respiratory alkalosis caused by too high a respiratory rate. The PaO2 is appropriate for a patient with COPD and increasing the respiratory rate and tidal volume would further lower the PaCO2.

44. Which patient should the nurse assess first after receiving change-of-shift report? a. 60-year-old patient whose new ileostomy has drained 800 mL over the previous 8 hours b. 50-year-old patient with familial adenomatous polyposis who has occult blood in the stool c. 40-year-old patient with ulcerative colitis who has had six liquid stools in the previous 4 hours d. 30-year-old patient who has abdominal distention and an apical heart rate of 136 beats/minute

ANS: D The patient's abdominal distention and tachycardia suggest hypovolemic shock caused by problems such as peritonitis or intestinal obstruction, which will require rapid intervention. The other patients should also be assessed as quickly as possible, but the data do not indicate any life-threatening complications associated with their diagnoses.

7. A 27-year-old female patient is admitted to the hospital for evaluation of right lower quadrant abdominal pain with nausea and vomiting. Which action should the nurse take? a. Encourage the patient to sip clear liquids. b. Assess the abdomen for rebound tenderness. c. Assist the patient to cough and deep breathe. d. Apply an ice pack to the right lower quadrant.

ANS: D The patient's clinical manifestations are consistent with appendicitis, and application of an ice pack will decrease inflammation at the area. Checking for rebound tenderness frequently is unnecessary and uncomfortable for the patient. The patient should be NPO in case immediate surgery is needed. The patient will need to know how to cough and deep breathe postoperatively, but coughing will increase pain at this time.

A 56-year-old patient arrives in the emergency department with hemiparesis and dysarthria that started 2 hours previously, and health records show a history of several transient ischemic attacks (TIAs). The nurse anticipates preparing the patient for a. surgical endarterectomy. b. transluminal angioplasty. c. intravenous heparin administration. d. tissue plasminogen activator (tPA) infusion

ANS: D The patient's history and clinical manifestations suggest an acute ischemic stroke and a patient who is seen within 4.5 hours of stroke onset is likely to receive tPA (after screening with a CT scan). Heparin administration in the emergency phase is not indicated. Emergent carotid transluminal angioplasty or endarterectomy is not indicated for the patient who is having an acute ischemic stroke

43. After several days of antibiotic therapy, an older hospitalized patient develops watery diarrhea. Which action should the nurse take first? a. Notify the health care provider. b. Obtain a stool specimen for analysis. c. Teach the patient about handwashing. d. Place the patient on contact precautions.

ANS: D The patient's history and new onset diarrhea suggest a C. difficile infection, which requires implementation of contact precautions to prevent spread of the infection to other patients. The other actions are also appropriate but can be accomplished after contact precautions are implemented.

A patient who was found unconscious in a burning house is brought to the emergency department by ambulance. The nurse notes that the patient's skin color is bright red. Which action should the nurse take first? a. Insert two large-bore IV lines. b. Check the patient's orientation. c. Assess for singed nasal hair and dark oral mucous membranes. d. Place the patient on 100% oxygen using a non-rebreather mask.

ANS: D The patient's history and skin color suggest carbon monoxide poisoning, which should be treated by rapidly starting oxygen at 100%. The other actions can be taken after the action to correct gas exchange

37. A 45-year-old patient is admitted to the emergency department with severe abdominal pain and rebound tenderness. Vital signs include temperature 102° F (38.3° C), pulse 120, respirations 32, and blood pressure (BP) 82/54. Which prescribed intervention should the nurse implement first? a. Administer IV ketorolac (Toradol) 15 mg. b. Draw blood for a complete blood count (CBC). c. Obtain a computed tomography (CT) scan of the abdomen. d. Infuse 1 liter of lactated Ringer's solution over 30 minutes.

ANS: D The priority for this patient is to treat the patient's hypovolemic shock with fluid infusion. The other actions should be implemented after starting the fluid infusion.

When assisting with the placement of a pulmonary artery (PA) catheter, the nurse notes that the catheter is correctly placed when the monitor shows a a. typical PA pressure waveform. b. tracing of the systemic arterial pressure. c. tracing of the systemic vascular resistance. d. typical PA wedge pressure (PAWP) tracing.

ANS: D The purpose of a PA line is to measure PAWP, so the catheter is floated through the pulmonary artery until the dilated balloon wedges in a distal branch of the pulmonary artery, and the PAWP readings are available. After insertion, the balloon is deflated and the PA waveform will be observed. Systemic arterial pressures are obtained using an arterial line and the systemic vascular resistance is a calculated value, not a waveform.

17. The patient has a catheter that must be irrigated. The nurse is using a needleless closed irrigation technique. In which order will the nurse perform the steps, starting with the first one? 1. Clean injection port. 2. Inject prescribed solution. 3. Twist needleless syringe into port. 4. Remove clamp and allow to drain. 5. Clamp catheter just below specimen port. 6. Draw up prescribed amount of sterile solution ordered. a. 3, 2, 6, 1, 5, 4 b. 5, 6, 1, 2, 3, 4 c. 1, 5, 6, 3, 2, 4 d. 6, 5, 1, 3, 2, 4

ANS: D The steps for irrigating with a needleless closed irrigation technique is as follows: Draw up in a syringe the prescribed amount of medication or sterile solution; clamp indwelling retention catheter just below specimen port; using circular motion, clean injection port with antiseptic swab; insert tip of needleless syringe using twisting motion into irrigation port; slowly and evenly inject fluid into catheter and bladder; and withdraw syringe, remove clamp, and allow solution to drain into drainage bag.

22. A 47-year-old female patient is transferred from the recovery room to a surgical unit after a transverse colostomy. The nurse observes the stoma to be deep pink with edema and a small amount of sanguineous drainage. The nurse should a. place ice packs around the stoma. b. notify the surgeon about the stoma. c. monitor the stoma every 30 minutes. d. document stoma assessment findings.

ANS: D The stoma appearance indicates good circulation to the stoma. There is no indication that surgical intervention is needed or that frequent stoma monitoring is required. Swelling of the stoma is normal for 2 to 3 weeks after surgery, and an ice pack is not needed.

Eight hours after a thermal burn covering 50% of a patient's total body surface area (TBSA) the nurse assesses the patient. Which information would be a priority to communicate to the health care provider? a. Blood pressure is 95/48 per arterial line. b. Serous exudate is leaking from the burns. c. Cardiac monitor shows a pulse rate of 108. d. Urine output is 20 mL per hour for the past 2 hours.

ANS: D The urine output should be at least 0.5 to 1.0 mL/kg/hr during the emergent phase, when the patient is at great risk for hypovolemic shock. The nurse should notify the health care provider because a higher IV fluid rate is needed. BP during the emergent phase should be greater than 90 systolic, and the pulse rate should be less than 120. Serous exudate from the burns is expected during the emergent phase

A patient in the emergency department with sudden-onset right-sided weakness is diagnosed with an intracerebral hemorrhage. Which information about the patient is most important to communicate to the health care provider? a. The patient's speech is difficult to understand. b. The patient's blood pressure is 144/90 mm Hg. c. The patient takes a diuretic because of a history of hypertension. d. The patient has atrial fibrillation and takes warfarin (Coumadin)

ANS: D The use of warfarin probably contributed to the intracerebral bleeding and remains a risk factor for further bleeding. Administration of vitamin K is needed to reverse the effects of the warfarin, especially if the patient is to have surgery to correct the bleeding. The history of hypertension is a risk factor for the patient but has no immediate effect on the patient's care. The BP of 144/90 indicates the need for ongoing monitoring but not for any immediate change in therapy. Slurred speech is consistent with a left-sided stroke, and no change in therapy is indicated

A young adult patient who is in the rehabilitation phase after having deep partial-thickness face and neck burns has a nursing diagnosis of disturbed body image. Which statement by the patient indicates that the problem is resolving? a. "I'm glad the scars are only temporary." b. "I will avoid using a pillow, so my neck will be OK." c. "I bet my boyfriend won't even want to look at me anymore." d. "Do you think dark beige makeup foundation would cover this scar on my cheek?"

ANS: D The willingness to use strategies to enhance appearance is an indication that the disturbed body image is resolving. Expressing feelings about the scars indicates a willingness to discuss appearance, but not resolution of the problem. Because deep partial-thickness burns leave permanent scars, a statement that the scars are temporary indicates denial rather than resolution of the problem. Avoiding using a pillow will help prevent contractures, but it does not address the problem of disturbed body image

29. A nurse is inserting an indwelling urinary catheter for a male patient. Which action will the nurse take? a. Hold the shaft of the penis at a 60-degree angle. b. Hold the shaft of the penis with the dominant hand. c. Cleanse the meatus 3 times with the same cotton ball from clean to dirty. d. Cleanse the meatus with circular strokes beginning at the meatus and working outward.

ANS: D Using the uncontaminated dominant hand, cleanse the meatus with cotton balls/swab sticks, using circular strokes, beginning at the meatus and working outward in a spiral motion. Repeat 3 times using a clean cotton ball/swabstick each time. With the nondominant hand (now contaminated), retract the foreskin (if uncircumcised) and gently grasp the penis at the shaft just below the glans. Hold the shaft of the penis at a right angle to the body.

During the emergent phase of burn care, which assessment will be most useful in determining whether the patient is receiving adequate fluid infusion? a. Check skin turgor. b. Monitor daily weight. c. Assess mucous membranes. d. Measure hourly urine output.

ANS: D When fluid intake is adequate, the urine output will be at least 0.5 to 1 mL/kg/hour. The patient's weight is not useful in this situation because of the effects of third spacing and evaporative fluid loss. Mucous membrane assessment and skin turgor also may be used, but they are not as adequate in determining that fluid infusions are maintaining adequate perfusion

When caring for the patient with a pulmonary artery (PA) pressure catheter, the nurse observes that the PA waveform indicates that the catheter is in the wedged position. Which action should the nurse take next? a. Zero balance the transducer. b. Activate the fast flush system. c. Notify the health care provider. d. Deflate and reinflate the PA balloon.

ANS: D When the catheter is in the wedge position, blood flow past the catheter is obstructed, placing the patient at risk for pulmonary infarction. A health care provider or advanced practice nurse should be called to reposition the catheter. The other actions will not correct the wedging of the PA catheter.

A pregnant woman arrives at the birth unit in labor at term, having had no prenatal care. After birth, her infant is noted to be small for gestational age with small eyes and a thin upper lip. The infant also is microcephalic. Based on her infant's physical findings, this woman should be questioned about her use of which substance during pregnancy?

Alcohol

Which substance, when abused during pregnancy, is the most significant cause of cognitive impairment and dysfunction in the infant?

Alcohol

a

An infant at 26 weeks of gestation arrives intubated from the delivery room. The nurse weighs the infant, places him under the radiant warmer, and attaches him to the ventilator at the prescribed settings. A pulse oximeter and cardiorespiratory monitor are placed. The pulse oximeter is recording oxygen saturations of 80%. The prescribed saturations are 92%. What is the nurse's most appropriate action at this time? a. Listening to breath sounds, and ensuring the patency of the endotracheal tube, increasing oxygen, and notifying a physician b. Continuing to observe and making no changes until the saturations are 75% c. Continuing with the admission process to ensure that a thorough assessment is completed d. Notifying the parents that their infant is not doing well

c

An infant at 36 weeks of gestation has increasing respirations (80 to 100 breaths per minute with significant substernal retractions). The infant is given oxygen by continuous nasal positive airway pressure (CPAP). What level of partial pressure of arterial oxygen (PaO2) indicates hypoxia? a. 67 mm Hg b. 89 mm Hg c. 45 mm Hg d. 73 mm Hg

c

An infant is being discharged from the NICU after 70 days of hospitalization. The infant was born at 30 weeks of gestation with several conditions associated with prematurity, including RDS, mild bronchopulmonary dysplasia (BPD), and retinopathy of prematurity (ROP), requiring surgical treatment. During discharge teaching, the infant's mother asks the nurse if her baby will meet developmental milestones on time, as did her son who was born at term. What is the nurse's most appropriate response? a. "Your baby will develop exactly like your first child." b. "Your baby does not appear to have any problems at this time." c. "Your baby will need to be corrected for prematurity." d. "Your baby will need to be followed very closely."

c

An infant is to receive gastrostomy feedings. Which intervention should the nurse institute to prevent bloating, gastrointestinal reflux into the esophagus, vomiting, and respiratory compromise? a. Rapid bolusing of the entire amount in 15 minutes b. Warm cloths to the abdomen for the first 10 minutes c. Slow, small, warm bolus feedings over 30 minutes d. Cold, medium bolus feedings over 20 minutes

A 3.8-kg infant was vaginally delivered at 39 weeks after a 30-minute second stage. A nuchal cord was found at delivery. After birth, the infant is noted to have petechiae over the face and upper back. Which information regarding petechiae is most accurate and should be provided to the parents?

Are benign if they disappear within 48 hours of birth.

When admitting an acutely confused 20-year-old patient with a head injury, which action should the nurse take?

Ask family members about the patient's health history.

A pregnant woman at 29 weeks of gestation has been diagnosed with preterm labor. Her labor is being controlled with tocolytic medications. She asks when she might be able to go home. What response by the nurse is most accurate? A. "It depends on what kind of insurance coverage you have." B. "When we can stabilize your preterm labor and arrange home health visits." C. "Whenever the doctor says that it is okay." D. "After the baby is born."

B. "When we can stabilize your preterm labor and arrange home health visits."

While assessing a newborn, a nurse should be aware that the average expected apical pulse range of a full-term, alert newborn is: A. 150 to 180 beats/min B. 120 to 160 beats/min C. 100 to 120 beats/min D. 80 to 100 beats/min

B. 120 to 160 beats/min

While assessing the integument of a 24-hour-old newborn, the nurse notes a pink papular rash with vesicles superimposed on the thorax, back, and abdomen. The nurse should: A. Move the newborn to an isolation nursery B. Document the finding as erythema toxicum C. Take the newborn's temperature and obtain a culture of one of the vesicles D. Notify the physician immediately

B. Document the finding as erythema toxicum

Several changes in the integumentary system that appear during pregnancy disappear after birth, although not always completely. Patients often ask the nurse about these changes. What change is almost certain to be completely reversed? A. Spider nevi B. Nail brittleness and weakness C. Darker pigmentation of the areolae and linea nigra D. Striae gravidarum on the breasts, abdomen, and thighs

B. Nail brittleness and weakness

A newborn is jaundiced and is receiving phototherapy via ultraviolet bank lights. An appropriate nursing intervention when caring for an infant with hyperbilirubinemia and receiving phototherapy by this method is to: A. Limit the newborn's intake of milk to prevent nausea, vomiting, and diarrhea B. Place eye shields over the newborn's closed eyes C. Change the newborn's position every 4 hours D. Apply an oil-based lotion to the newborn's skin to prevent dying and cracking

B. Place eye shields over the newborn's closed eyes

A nurse can assist a father in his transition to parenthood by: A. Telling him to tape the infant's diaper a different way B. Pointing out that the infant turned at the sound of his voice C. Encouraging him to go home to get some sleep D. Suggesting that he let the infant sleep in the bassinet

B. Pointing out that the infant turned at the sound of his voice

A woman gave birth vaginally to a 9-lb, 12-oz girl yesterday. Her primary health care provider has written orders for perineal ice packs, use of a sitz bath tid, and a stool softener. What information is most closely correlated with these orders? A. The woman had a vacuum-assisted birth B. The woman has an episiotomy C. The woman is a gravida 2, para 2 D. The woman received epidural anesthesia

B. The woman has an episiotomy

Obese women are at risk for several complications during pregnancy and birth. These include: SELECT ALL THAT APPLY ( partial credit will not be given) A. Breech presentation B. Thromboembolism C. Cesarean birth D. Preterm labor

B. Thromboembolism C. Cesarean birth

A woman who has recently given birth complains of pain and tenderness in her leg. On physical examination, the nurse notices warmth and redness over an enlarged, hardened area. The nurse should suspect the patient may be diagnosed with A. Thrombocytopenic purpura B. Thrombophlebitis; C. Disseminated intravascular coagulation (DIC); D. von Willebrand disease (vWD)

B. Thrombophlebitis;

d

Because of the premature infant's decreased immune functioning, what nursing diagnosis should the nurse include in a plan of care for a premature infant? a. Delayed growth and development b. Ineffective thermoregulation c. Ineffective infant feeding pattern d. Risk for infection

c

By understanding the four mechanisms of heat transfer (convection, conduction, radiation, and evaporation), the nurse can create an environment for the infant that prevents temperature instability. Which significant symptoms will the infant display when experiencing cold stress? a. Decreased respiratory rate b. Bradycardia, followed by an increased heart rate c. Mottled skin with acrocyanosis d. Increased physical activity

Which woman is most likely to experience strong afterpains and cramping post delivery? A. A woman who is G3 P 2 and is bottle feeding her infant B. A woman whose infant weighed 5 lb, 3 oz C. A woman who is a gravida 4, para 4 and is breastfeeding D. A woman who experienced oligohydramnios

C. A woman who is a gravida 4, para 4 and is breastfeeding

A woman gave birth 48 hours ago to a healthy infant girl. She has decided to bottle-feed. During your assessment you notice that both her breasts are swollen, warm, and tender on palpation. The woman should be advised that this condition can best be treated by: A. Wearing a loose-fitting bra to prevent nipple irritation B. Running warm water on her breasts during a shower C. Applying ice to the breasts and/or cabbage leaves for comfort D. Expressing small amounts of milk from the breasts to relieve pressure

C. Applying ice to the breasts and/or cabbage leaves for comfort

A nurse providing care for a woman with preterm labor on terbutaline includes which intervention to identify side effects of the drug? A. Assess for hypoglycemia. B. Assess for bradycardia C. Assess for tachycardia and tremors D. Assess deep tendon reflexes (DTRs).

C. Assess for tachycardia and tremors

The nurse administers vitamin K to the newborn for what reason? A. The supply of vitamin K is inadequate for at least 3 to 4 months, and the newborn must be supplemented B. Most mothers have a diet deficient in vitamin K, which results in the infant being deficient. C. Bacteria that synthesize vitamin K are not present in the newborn's intestinal tract. D. Vitamin K prevents the synthesis of prothrombin in the liver and must be given by injection.

C. Bacteria that synthesize vitamin K are not present in the newborn's intestinal tract.

In evaluating the effectiveness of an oxytocin induction, the nurse expects: A. At least 30 milliunits/min of oxytocin are needed to achieve cervical dilation B. Labor to progress at least 2 cm/hr dilation C. Contractions lasting 40 to 90 seconds, 2 to 3 minutes apart D. The intensity of contractions to be at least 110 to 130 mm Hg

C. Contractions lasting 40 to 90 seconds, 2 to 3 minutes apart

Two days ago a woman gave birth to a full-term infant. Last night she awakened several times to urinate and noted that her gown and bedding were wet from profuse diaphoresis. One mechanism for the diaphoresis and diuresis that this woman is experiencing during the early postpartum period is: A. Elevated temperature caused by postpartum infection B. Increased basal metabolic rate after giving birth C. Hormonal changes and the loss of increased blood volume associated with pregnancy D. Increased venous pressure in the lower extremities

C. Hormonal changes and the loss of increased blood volume associated with pregnancy

The nurse is assessing a client who is 6 hours postpartum after delivering a full term healthy newborn with no delivery complications. The client complains of feeling faint and dizzy from time to time. What is the MOST appropriate nursing action at this time? A. Obtain hemoglobin and hematocrit levels B. Raise the head of the bed C. Instruct the client to request help when getting out of bed D. Inform the nursery nurse not to bring the baby out to the mother

C. Instruct the client to request help when getting out of bed

An dark skinned woman noticed some bruises on her newborn girl's buttocks. She asks the nurse who spanked her daughter. The nurse explains that these marks are called: A. Nevus flammeus B. Vascular nevi C. Mongolian spots D. Lanugo

C. Mongolian spots

Nurses should know some basic definitions concerning preterm birth, preterm labor, and low birth weight. The maternity nurse knows when assessing her patient that: A. In the United States early in this century, preterm birth accounted for 18% to 20% of all births B. The terms preterm birth and low birth weight can be used interchangeably C. Preterm labor is defined as cervical changes and uterine contractions occurring between 20 and 37 weeks of pregnancy D. Low birth weight is anything below 3.7 lb

C. Preterm labor is defined as cervical changes and uterine contractions occurring between 20 and 37 weeks of pregnancy

A new father wants to know what medication was put into his infant's eyes and why it is needed. The nurse explains to the father that the purpose of the erythromycin (Ilotycin) ophthalmic ointment is to: A. Destroy an infectious exudate caused by Staphylococcus that could make the infant blind B. Prevent the infant's eyelids from sticking together and help the infant see C. Prevent gonorrheal and chlamydial infection of the infant's eyes potentially acquired from the birth canal D. Prevent potentially harmful exudate from invading the tear ducts of the infant's eyes, leading to dry eyes

C. Prevent gonorrheal and chlamydial infection of the infant's eyes potentially acquired from the birth canal

Which hormone remains elevated in the immediate postpartum period of the breastfeeding woman? A. Human placental lactogen B. Progesterone C. Prolactin D. Estrogen

C. Prolactin

After giving birth to a healthy infant boy, a primiparous woman, age 16, is admitted to the postpartum unit. An appropriate nursing diagnosis for her at this time is "Risk for Impaired Parenting" related to deficient knowledge of newborn care. In planning for the woman's discharge, what should the nurse should be certain to include in the plan of care? A. Give the woman written information on bathing her infant. B. Advise the woman that all mothers instinctively know how to care for their infants C. Provide time for the woman to bathe her infant after she views an infant bath demonstration D. Tell the woman how to feed and bathe her infant.

C. Provide time for the woman to bathe her infant after she views an infant bath demonstration

A woman in preterm labor at 30 weeks of gestation receives two 12-mg doses of betamethasone intramuscularly. The purpose of this pharmacologic treatment is to: A. Suppress uterine contractions B. Maintain adequate maternal respiratory effort and ventilation during magnesium sulfate therapy C. Stimulate fetal surfactant production D. Reduce maternal and fetal tachycardia associated with ritodrine administration

C. Stimulate fetal surfactant production

A number of common drugs of abuse may cross into the breast milk of a mother who is currently using these substances, which may result in behavioral effects in the newborn. Which substances are contraindicated if the mother elects to breastfeed her infant? (Select all that apply.)

Cocaine Marijuana Nicotine

Near the end of the first week of life, an infant who has not been treated for any infection develops a copper-colored maculopapular rash on the palms and around the mouth and anus. The newborn is displaying signs and symptoms of which condition?

Congenital syphilis

Many first-time parents do not plan on their parents' help immediately after the newborn arrives. What statement by the nurse is the most appropriate when counseling new parents about the involvement of grandparents? A. "You should tell your parents to leave you alone." B. "They are getting old. You should let them be involved while they can." C. "Grandparent involvement can be very disruptive to the family." D. "Grandparents can help you with parenting skills and also help preserve family traditions."

D. "Grandparents can help you with parenting skills and also help preserve family traditions."

A client is to have an amniotomy to induce labor. The nurse recognizes that the priority intervention after the amniotomy is to: A. Apply clean linens under the woman B. Take the client's vital signs C. Perform a vaginal examination D. Assess the fetal heart rate (FHR)

D. Assess the fetal heart rate (FHR)

A woman gave birth to an infant boy 10 hours ago. Where does the nurse expect to locate this woman's fundus? A. Nonpalpable abdominally B. Midway between the umbilicus and the symphysis pubis C. Two centimeters below the umbilicus D. At the level of the umbilicus

D. At the level of the umbilicus

A 25 week gestation pregnant patient arrives at the Labor and Delivery unit complaining of contractions and spotting. As the nurses work quickly to get the patient into bed the doctor orders a biochemical marker test to determine the likelihood that this patient is in preterm labor. The nurse knows that this biochemical marker is known as A. Rh sensitization test B. Gestational age predictor test C. Version predictor test D. Fetal fibronectin test

D. Fetal fibronectin test

While examining a newborn, the nurse notes uneven skin folds on the buttocks and a clunk/popping noise when performing the Ortolani maneuver. The nurse recognizes these findings as a sign that the newborn will most likely be diagnosed with: A. Clubfoot B. Webbing C. Polydactyly D. Hip dysplasia

D. Hip dysplasia

Which finding could prevent early discharge of a newborn who is now 12 hours old? A. Voided, clear, pale urine three times since birth B. Birth weight of 6 pounds C. One meconium stool since birth D. Infant breastfed once with some difficulty with latch and sucking and once with some success for about 5 minutes on each breast

D. Infant breastfed once with some difficulty with latch and sucking and once with some success for about 5 minutes on each breast

During a phone follow-up conversation with a woman who is 4 days postpartum, the woman tells the nurse, "I don't know what's wrong. I love my son, but I feel so let down. I seem to cry for no reason!" The nurse recognizes that the woman is experiencing: A. Attachment difficulty B. Extreme Postpartum depression (PPD) C. Letting-go D. Postpartum blues

D. Postpartum blues

A 30-year-old multiparous woman has a boy who is 2½ years old and now an infant girl. She tells the nurse, "I don't know how I'll ever manage both children when I get home." Which suggestion would best help this woman alleviate sibling rivalry? A. Tell the older child that he is a big boy now and should love his new sister. B. Let the older child stay with his grandparents for the first 6 weeks to allow him to adjust to the newborn. C. Ask friends and relatives not to bring gifts to the older sibling because you do not want to spoil him. D. Realize that the regression in habits and behaviors in the older child is a typical reaction and that he needs extra love and attention at this time.

D. Realize that the regression in habits and behaviors in the older child is a typical reaction and that he needs extra love and attention at this time.

With regard to the care management of preterm labor, nurses should be aware that: A. Because preterm labor is likely to be the start of an extended labor, a woman with symptoms can wait several hours before contacting the primary caregiver B. Braxton Hicks contractions often signal the onset of preterm labor C. Because all women must be considered at risk for preterm labor and prediction is so hit and miss, teaching pregnant women the symptoms probably causes more harm through false alarms D. The diagnosis of preterm labor is based on gestational age, uterine activity, and progressive cervical change

D. The diagnosis of preterm labor is based on gestational age, uterine activity, and progressive cervical change

With regard to the functioning of the renal system in newborns, nurses should be aware that: A. Weight loss from fluid loss and other normal factors should be made up in 4 to 7 days B. Breastfed infants likely will void more often during the first days after birth C. "Brick dust" or blood on a diaper is always cause to notify the physician D. The pediatrician should be notified if the newborn has not voided in 24 hours

D. The pediatrician should be notified if the newborn has not voided in 24 hours

A primiparous woman is to be discharged from the hospital tomorrow with her infant girl. Which behavior indicates a need for further intervention by the nurse before the woman can be discharged? A. The woman changes her infant's diaper and then shows the nurse the contents of the diaper. B. The woman continues to hold and cuddle her infant after she has fed her. C. The woman reads a magazine while her infant sleeps. D. The woman is disinterested in learning about infant care.

D. The woman is disinterested in learning about infant care.

a

During the assessment of a preterm infant, the nurse notices continued respiratory distress even though oxygen and ventilation have been provided. In this situation, which condition should the nurse suspect? a. Hypovolemia and/or shock b. Excessively cool environment c. Central nervous system (CNS) injury d. Pending renal failure

The nurse will anticipate teaching a patient with a possible seizure disorder about which test?

Electroencephalography (EEG)

Which finding would the nurse expect when assessing the legs of a patient who has a lower motor neuron lesion?

Flaccidity

c

For clinical purposes, the most accurate definition of preterm and postterm infants is defined as what? a. Preterm: Before 34 weeks of gestation if the infant is appropriate for gestational age (AGA); before 37 weeks if the infant is small for gestational age (SGA) b. Postterm: After 40 weeks of gestation if the infant is large for gestational age (LGA); beyond 42 weeks if the infant is AGA c. Preterm: Before 37 weeks of gestation and postterm beyond 42 weeks of gestation; no matter the size for gestational age at birth d. Preterm: Before 38 to 40 weeks of gestation if the infant is SGA; postterm, beyond 40 to 42 weeks gestation if the infant is LGA

A macrosomic infant is born after a difficult forceps-assisted delivery. After stabilization, the infant is weighed, and the birth weight is 4550 g (9 lb, 6 oz). What is the nurse's first priority?

Frequently monitor blood glucose levels, and closely observe the infant for signs of hypoglycemia

What is the most important nursing action in preventing neonatal infection?

Good handwashing

What bacterial infection is definitely decreasing because of effective drug treatment?

Group B Stretococci (GBS) infection

An infant was born 2 hours ago at 37 weeks of gestation and weighs 4.1 kg. The infant appears chubby with a flushed complexion and is very tremulous. The tremors are most likely the result of what condition?

Hypoglycemia

Which information regarding to injuries to the infant's plexus during labor and birth is most accurate?

If the nerves are stretched with no avulsion, then they should completely recover in 3 to 6 months.

b

In appraising the growth and development potential of a preterm infant, the nurse should be cognizant of the information that is best described in which statement? a. Tell the parents that their child will not catch up until approximately age 10 years (for girls) to age 12 years (for boys). b. Correct for milestones, such as motor competencies and vocalizations, until the child is approximately 2 years of age. c. Know that the greatest catch-up period is between 9 and 15 months postconceptual age. d. Know that the length and breadth of the trunk is the first part of the infant to experience catch-up growth.

b

In caring for the preterm infant, what complication is thought to be a result of high arterial blood oxygen level? a. NEC b. ROP c. BPD d. Intraventricular hemorrhage (IVH)

The nurse is evaluating a neonate who was delivered 3 hours ago by vacuum-assisted delivery. The infant has developed a cephalhematoma. Which statement is most applicable to the care of this neonate?

In many infants, signs of hemorrhage in a full-term infant are absent and diagnosed only through laboratory tests.

acd

Infants born between 34 0/7 and 36 6/7 weeks of gestation are called late-preterm infants because they have many needs similar to those of preterm infants. Because they are more stable than early-preterm infants, they may receive care that is similar to that of a full-term baby. These infants are at increased risk for which conditions? (Select all that apply.) a. Problems with thermoregulation b. Cardiac distress c. Hyperbilirubinemia d. Sepsis e. Hyperglycemia

b

NEC is an acute inflammatory disease of the gastrointestinal mucosa that can progress to perforation of the bowel. Approximately 2% to 5% of premature infants succumb to this fatal disease. Care is supportive; however, known interventions may decrease the risk of NEC. Which intervention has the greatest effect on lowering the risk of NEC? a. Early enteral feedings b. Breastfeeding c. Exchange transfusion d. Prophylactic probiotics

b

Necrotizing enterocolitis (NEC) is an inflammatory disease of the gastrointestinal mucosa. The signs of NEC are nonspecific. What are generalized signs and symptoms of this condition? a. Hypertonia, tachycardia, and metabolic alkalosis b. Abdominal distention, temperature instability, and grossly bloody stools c. Hypertension, absence of apnea, and ruddy skin color d. Scaphoid abdomen, no residual with feedings, and increased urinary output

Providing care for the neonate born to a mother who abuses substances can present a challenge for the health care team. Nursing care for this infant requires a multisystem approach. What is the first step in the provision of care for the infant?

Neonatal abstinence Syndrome (NAS) Scoring

What information regarding a fractured clavicle is most important for the nurse to take into consideration when planning the infant's care?

No special treatment is necessary.

d

On day 3 of life, a newborn continues to require 100% oxygen by nasal cannula. The parents ask if they may hold their infant during his next gavage feeding. Considering that this newborn is physiologically stable, what response should the nurse provide? a. "Parents are not allowed to hold their infants who are dependent on oxygen." b. "You may only hold your baby's hand during the feeding." c. "Feedings cause more physiologic stress; therefore, the baby must be closely monitored. I don't think you should hold the baby." d. "You may hold your baby during the feeding."

Several patients have been hospitalized for diagnosis of neurologic problems. Which patient will the nurse assess first?

Patient with a brain tumor who has just arrived on the unit after a cerebral angiogram

An unconscious male patient has just arrived in the emergency department after a head injury caused by a motorcycle crash. Which order should the nurse question?

Prepare the patient for lumbar puncture.

Which cerebrospinal fluid analysis result will be most important for the nurse to communicate to the health care provider?

Protein 65 mg/dL (0.65 g/L)

Which conditions are infants of diabetic mothers (IDMs) at a higher risk for developing?

Respiratory Distress Syndrome

Which nursing diagnosis is expected to be appropriate for a patient who has a positive Romberg test?

Risk for falls

A pregnant woman at 37 weeks of gestation has had ruptured membranes for 26 hours. A cesarean section is performed for failure to progress. The fetal heart rate (FHR) before birth is 180 beats per minute with limited variability. At birth the newborn has Apgar scores of 6 and 7 at 1 and 5 minutes and is noted to be pale and tachypneic. Based on the maternal history, what is the most likely cause of this newborn's distress?

Sepsis

For an infant experiencing symptoms of drug withdrawal, which intervention should be included in the plan of care?

Snugly swaddling the infant and tightly holding the baby.

The charge nurse is observing a new staff nurse who is assessing a patient with a traumatic spinal cord injury for sensation. Which action indicates a need for further teaching of the new nurse about neurologic assessment?

The new nurse asks the patient if the instrument feels sharp.

A 39-year-old patient with a suspected herniated intervertebral disc is scheduled for a myelogram. Which information is most important for the nurse to communicate to the health care provider before the procedure?

The patient has an allergy to shellfish.

Human immunodeficiency virus (HIV) may be transmitted perinatally or during the postpartum period. Which statement regarding the method of transmission is most accurate?

Through the ingestion of breast milk from an infected mother.

Which equipment will the nurse obtain to assess vibration sense in a diabetic patient who has peripheral nerve dysfunction?

Tuning fork

Which information about a 76-year-old patient is most important for the admitting nurse to report to the patient's health care provider?

Unintended weight loss of 20 pounds

The nurse should be cognizant of which condition related to skeletal injuries sustained by a neonate during labor or childbirth?

Unless a blood vessel is involved, linear skull fractures heal without special treatment.

a

What is the most important nursing action in preventing neonatal infection? a. Good handwashing b. Isolation of infected infants c. Separate gown technique d. Standard Precautions

d

When evaluating the preterm infant, the nurse understands that compared with the term infant, what information is important for the nurse to understand? a. Few blood vessels visible through the skin b. More subcutaneous fat c. Well-developed flexor muscles d. Greater surface area in proportion to weight

d

When providing an infant with a gavage feeding, which infant assessment should be documented each time? a. Abdominal circumference after the feeding b. Heart rate and respirations before feeding c. Suck and swallow coordination d. Response to the feeding

d

Which clinical findings would alert the nurse that the neonate is expressing pain? a. Low-pitched crying; tachycardia; eyelids open wide b. Cry face; flaccid limbs; closed mouth c. High-pitched, shrill cry; withdrawal; change in heart rate d. Cry face; eyes squeezed; increase in blood pressure

d

Which condition might premature infants who exhibit 5 to 10 seconds of respiratory pauses, followed by 10 to 15 seconds of compensatory rapid respiration, be experiencing? a. Suffering from sleep or wakeful apnea b. Experiencing severe swings in blood pressure c. Trying to maintain a neutral thermal environment d. Breathing in a respiratory pattern common to premature infants

abc

Which risk factors are associated with NEC? (Select all that apply.) a. Polycythemia b. Anemia c. Congenital heart disease d. Bronchopulmonary dysphasia e. Retinopathy

c

With regard to an eventual discharge of the high-risk newborn or the transfer of the newborn to a different facility, which information is essential to provide to the parents? a. Infants stay in the NICU until they are ready to go home. b. Once discharged to go home, the high-risk infant should be treated like any healthy term newborn. c. Parents of high-risk infants need special support and detailed contact information. d. If a high-risk infant and mother need to be transferred to a specialized regional center, then waiting until after the birth and until the infant is stabilized is best.

b

With regard to infants who are SGA and intrauterine growth restriction (IUGR), the nurse should be aware of which information? a. In the first trimester, diseases or abnormalities result in asymmetric IUGR. b. Infants with asymmetric IUGR have the potential for normal growth and development. c. In asymmetric IUGR, weight is slightly larger than SGA, whereas length and head circumference are somewhat less than SGA. d. Symmetric IUGR occurs in the later stages of pregnancy.

Which action will the nurse include in the plan of care for a patient with impaired functioning of the left glossopharyngeal nerve (CN IX) and the vagus nerve (CN X)?

Withhold oral fluid or foods.

26. Which patient statement indicates that the nurse's teaching following a gastroduodenostomy has been effective? a. "Vitamin supplements may prevent anemia." b. "Persistent heartburn is common after surgery." c. "I will try to drink more liquids with my meals." d. "I will need to choose high carbohydrate foods."

a. "Vitamin supplements may prevent anemia." Cobalamin deficiency may occur after partial gastrectomy, and the patient may need to receive cobalamin via injections or nasal spray. Although peptic ulcer disease may recur, persistent heartburn is not expected after surgery and the patient should call the health care provider if this occurs. Ingestion of liquids with meals is avoided to prevent dumping syndrome. Foods that have moderate fat and low carbohydrate should be chosen to prevent dumping syndrome.

Which patient is most appropriate for the intensive care unit (ICU) charge nurse to assign to a registered nurse (RN) who has floated from the medical unit? a. A 45-yr-old patient receiving IV antibiotics for meningococcal meningitis b. A 35-yr-old patient with intracranial pressure (ICP) monitoring after a head injury c. A 25-yr-old patient admitted with a skull fracture and craniotomy the previous day d. A 55-yr-old patient who has increased intracranial pressure (ICP) and is receiving hyperventilation therapy

a. A 45-yr-old patient receiving IV antibiotics for meningococcal meningitis

A male patient who has possible cerebral edema has a serum sodium level of 116 mEq/L (116 mmol/L) and a decreasing level of consciousness (LOC). He is now complaining of a headache. Which prescribed interventions should the nurse implement first? a. Administer IV 5% hypertonic saline. b. Draw blood for arterial blood gases (ABGs). c. Send patient for computed tomography (CT). d. Administer acetaminophen (Tylenol) 650 mg orally.

a. Administer IV 5% hypertonic saline

5. Which information will the nurse include when teaching adults to decrease the risk for cancers of the tongue and buccal mucosa? a. Avoid use of cigarettes and smokeless tobacco. b. Use sunscreen when outside even on cloudy days. c. Complete antibiotic courses used to treat throat infections. d. Use antivirals to treat herpes simplex virus (HSV) infections.

a. Avoid use of cigarettes and smokeless tobacco. Tobacco use greatly increases the risk for oral cancer. Acute throat infections do not increase the risk for oral cancer, although chronic irritation of the oral mucosa does increase risk. Sun exposure does not increase the risk for cancers of the buccal mucosa. Human papillomavirus (HPV) infection is associated with an increased risk, but HSV infection is not a risk factor for oral cancer.

Admission vital signs for a brain-injured patient are blood pressure of 128/68 mm Hg, pulse of 110 beats/min, and of respirations 26 breaths/min. Which set of vital signs, if taken 1 hour later, will be of most concern to the nurse? a. Blood pressure of 154/68 mm Hg, pulse of 56 beats/min, respirations of 12 breaths/min b. Blood pressure of 134/72 mm Hg, pulse of 90 beats/min, respirations of 32 breaths/min c. Blood pressure of 148/78 mm Hg, pulse of 112 beats/min, respirations of 28 breaths/min d. Blood pressure of 110/70 mm Hg, pulse of 120 beats/min, respirations of 30 breaths/min

a. Blood pressure of 154/68 mm Hg, pulse of 56 beats/min, respirations of 12 breaths/min

A 68-yr-old male patient is brought to the emergency department (ED) by ambulance after being found unconscious on the bathroom floor by his spouse. Which action will the nurse take first? a. Check oxygen saturation. b. Assess pupil reaction to light. c. Palpate the head for injuries d. Verify Glasgow Coma Scale (GCS) score.

a. Check oxygen saturation.

A patient who has bacterial meningitis is disoriented and anxious. Which nursing action will be included in the plan of care? a. Encourage family members to remain at the bedside. b. Apply soft restraints to protect the patient from injury. c. Keep the room well-lighted to improve patient orientation. d. Minimize contact with the patient to decrease sensory input.

a. Encourage family members to remain at the bedside.

8. Which action should the nurse in the emergency department anticipate for a 23-year-old patient who has had several episodes of bloody diarrhea? a. Obtain a stool specimen for culture. b. Administer antidiarrheal medication. c. Provide teaching about antibiotic therapy. d. Teach about adverse effects of acetaminophen (Tylenol).

a. Obtain a stool specimen for culture. Patients with bloody diarrhea should have a stool culture for E. coli O157:H7. Antidiarrheal medications are usually avoided for possible infectious diarrhea to avoid prolonging the infection. Antibiotic therapy in the treatment of infectious diarrhea is controversial because it may precipitate kidney complications. Acetaminophen does not cause bloody diarrhea.

The nurse is caring for a patient who has a head injury. Which finding, when reported to the health care provider, should the nurse expect will result in new prescribed interventions? a. Pale yellow urine output of 1200 mL over the past 2 hours. b. Ventriculostomy drained 40 mL of fluid in the past 2 hours. c. Intracranial pressure spikes to 16 mm Hg when patient is turned. d. LICOX brain tissue oxygenation catheter shows PbtO2 of 38 mm Hg.

a. Pale yellow urine output of 1200 mL over the past 2 hours.

A patient with possible viral meningitis is admitted to the nursing unit after lumbar puncture was performed in the emergency department. Which action prescribed by the health care provider should the nurse question? a. Restrict oral fluids to 1000 mL/day. b. Elevate the head of the bed 20 degrees. c. Administer ceftriaxone (Rocephin) 1 g IV every 12 hours. d. Give ibuprofen (Motrin) 400 mg every 6 hours as needed for headache.

a. Restrict oral fluids to 1000 mL/day.

A college athlete is seen in the clinic 6 weeks after a concussion. Which assessment information will the nurse collect to determine whether the patient is developing postconcussion syndrome? a. Short-term memory c. Glasgow Coma Scale b. Muscle coordination d. Pupil reaction to light

a. Short-term memory

While admitting a 42-yr-old patient with a possible brain injury after a car accident to the emergency department (ED), the nurse obtains the following information. Which finding is most important to report to the health care provider? a. The patient takes warfarin (Coumadin) daily. b. The patient's blood pressure is 162/94 mm Hg. c. The patient is unable to remember the accident. d. The patient complains of a severe dull headache.

a. The patient takes warfarin (Coumadin) daily.

5. A 50-year-old patient who underwent a gastroduodenostomy (Billroth I) earlier today complains of increasing abdominal pain. The patient has no bowel sounds and 200 mL of bright red nasogastric (NG) drainage in the last hour. The highest priority action by the nurse is to a. contact the surgeon. b. irrigate the NG tube. c. monitor the NG drainage. d. administer the prescribed morphine.

a. contact the surgeon. Increased pain and 200 mL of bright red NG drainage 12 hours after surgery indicate possible postoperative hemorrhage, and immediate actions such as blood transfusion and/or return to surgery are needed. Because the NG is draining, there is no indication that irrigation is needed. Continuing to monitor the NG drainage is not an adequate response. The patient may need morphine, but this is not the highest priority action.

19. The nurse will anticipate preparing a 71-year-old female patient who is vomiting "coffee-ground" emesis for a. endoscopy. b. angiography. c. barium studies. d. gastric analysis.

a. endoscopy. Endoscopy is the primary tool for visualization and diagnosis of upper gastrointestinal (GI) bleeding. Angiography is used only when endoscopy cannot be done because it is more invasive and has more possible complications. Barium studies are helpful in determining the presence of gastric lesions, but not whether the lesions are actively bleeding. Gastric analysis testing may help with determining the cause of gastric irritation, but it is not used for acute GI bleeding.

Which question will the nurse ask a patient who has been admitted with a benign occipital lobe tumor to assess for functional deficits? a. "Do you have difficulty in hearing?" b. "Are you experiencing visual problems?" c. "Are you having any trouble with your balance?" d. "Have you developed any weakness on one side?"

b. "Are you experiencing visual problems?"

30. Which information about dietary management should the nurse include when teaching a patient with peptic ulcer disease (PUD)? a. "You will need to remain on a bland diet." b. "Avoid foods that cause pain after you eat them." c. "High-protein foods are least likely to cause you pain." d. "You should avoid eating any raw fruits and vegetables."

b. "Avoid foods that cause pain after you eat them." The best information is that each individual should choose foods that are not associated with postprandial discomfort. Raw fruits and vegetables may irritate the gastric mucosa, but chewing well seems to decrease this problem and some patients may tolerate these foods well. High-protein foods help neutralize acid, but they also stimulate hydrochloric (HCl) acid secretion and may increase discomfort for some patients. Bland diets may be recommended during an acute exacerbation of PUD, but there is little scientific evidence to support their use.

Which statement by patient who is being discharged from the emergency department (ED) after a concussion indicates a need for intervention by the nurse? a. "I will return if I feel dizzy or nauseated." b. "I am going to drive home and go to bed." c. "I do not even remember being in an accident." d. "I can take acetaminophen (Tylenol) for my headache."

b. "I am going to drive home and go to bed."

12. Which information will the nurse include for a patient with newly diagnosed gastroesophageal reflux disease (GERD)? a. "Peppermint tea may reduce your symptoms." b. "Keep the head of your bed elevated on blocks." c. "You should avoid eating between meals to reduce acid secretion." d. "Vigorous physical activities may increase the incidence of reflux."

b. "Keep the head of your bed elevated on blocks." Elevating the head of the bed will reduce the incidence of reflux while the patient is sleeping. Peppermint will decrease lower esophageal sphincter (LES) pressure and increase the chance for reflux. Small, frequent meals are recommended to avoid abdominal distention. There is no need to make changes in physical activities because of GERD.

1. Which information will the nurse include when teaching a patient with peptic ulcer disease about the effect of ranitidine (Zantac)? a. "Ranitidine absorbs the gastric acid." b. "Ranitidine decreases gastric acid secretion." c. "Ranitidine constricts the blood vessels near the ulcer." d. "Ranitidine covers the ulcer with a protective material."

b. "Ranitidine decreases gastric acid secretion." Ranitidine is a histamine-2 (H2) receptor blocker, which decreases the secretion of gastric acid. The response beginning, "Ranitidine constricts the blood vessels" describes the effect of vasopressin. The response "Ranitidine absorbs the gastric acid" describes the effect of antacids. The response beginning "Ranitidine covers the ulcer" describes the action of sucralfate (Carafate).

16. The nurse determines that teaching regarding cobalamin injections has been effective when the patient with chronic atrophic gastritis states which of the following? a. "The cobalamin injections will prevent gastric inflammation." b. "The cobalamin injections will prevent me from becoming anemic." c. "These injections will increase the hydrochloric acid in my stomach." d. "These injections will decrease my risk for developing stomach cancer."

b. "The cobalamin injections will prevent me from becoming anemic." Cobalamin supplementation prevents the development of pernicious anemia. Chronic gastritis may cause achlorhydria, but cobalamin does not correct this. The loss of intrinsic factor secretion with chronic gastritis is permanent, and the patient will need lifelong supplementation with cobalamin. The incidence of stomach cancer is higher in patients with chronic gastritis, but cobalamin does not reduce the risk for stomach cancer.

Family members of a patient who has a traumatic brain injury ask the nurse about the purpose of the ventriculostomy system being used for intracranial pressure monitoring. Which response by the nurse is best for this situation? a. "This type of monitoring system is complex and it is managed by skilled staff." b. "The monitoring system helps show whether blood flow to the brain is adequate." c. "The ventriculostomy monitoring system helps check for alterations in cerebral perfusion pressure." d. "This monitoring system has multiple benefits including facilitation of cerebrospinal fluid drainage."

b. "The monitoring system helps show whether blood flow to the brain is adequate."

17. Which medications will the nurse teach the patient about whose peptic ulcer disease is associated with Helicobacter pylori? a. Sucralfate (Carafate), nystatin (Mycostatin), and bismuth (Pepto-Bismol) b. Amoxicillin (Amoxil), clarithromycin (Biaxin), and omeprazole (Prilosec) c. Famotidine (Pepcid), magnesium hydroxide (Mylanta), and pantoprazole (Protonix) d. Metoclopramide (Reglan), bethanechol (Urecholine), and promethazine (Phenergan)

b. Amoxicillin (Amoxil), clarithromycin (Biaxin), and omeprazole (Prilosec) The drugs used in triple drug therapy include a proton pump inhibitor such as omeprazole and the antibiotics amoxicillin and clarithromycin. The other combinations listed are not included in the protocol for H. pylori

A 20-yr-old male patient is admitted with a head injury after a collision while playing football. After noting that the patient has developed clear nasal drainage, which action should the nurse take? a. Have the patient gently blow the nose. b. Check the drainage for glucose content. c. Teach the patient that rhinorrhea is expected after a head injury. d. Obtain a specimen of the fluid to send for culture and sensitivity.

b. Check the drainage for glucose content

24. A 44-year-old man admitted with a peptic ulcer has a nasogastric (NG) tube in place. When the patient develops sudden, severe upper abdominal pain, diaphoresis, and a firm abdomen, which action should the nurse take? a. Irrigate the NG tube. b. Check the vital signs. c. Give the ordered antacid. d. Elevate the foot of the bed.

b. Check the vital signs. The patient's symptoms suggest acute perforation, and the nurse should assess for signs of hypovolemic shock. Irrigation of the NG tube, administration of antacids, or both would be contraindicated because any material in the stomach will increase the spillage into the peritoneal cavity. Elevating the foot of the bed may increase abdominal pressure and discomfort, as well as making it more difficult for the patient to breathe.

2. Which item should the nurse offer to the patient who is to restart oral intake after being NPO due to nausea and vomiting? a. Glass of orange juice b. Dish of lemon gelatin c. Cup of coffee with cream d. Bowl of hot chicken broth

b. Dish of lemon gelatin Clear cool liquids are usually the first foods started after a patient has been nauseated. Acidic foods such as orange juice, very hot foods, and coffee are poorly tolerated when patients have been nauseated.

13. Which nursing action should be included in the postoperative plan of care for a patient after a laparoscopic esophagectomy? a. Notify the doctor about bloody nasogastric (NG) drainage. b. Elevate the head of the bed to at least 30 degrees. c. Reposition the NG tube if drainage stops. d. Start oral fluids when the patient has active bowel sounds.

b. Elevate the head of the bed to at least 30 degrees. Elevation of the head of the bed decreases the risk for reflux and aspiration of gastric secretions. The NG tube should not be repositioned without consulting with the health care provider. Bloody NG drainage is expected for the first 8 to 12 hours. A swallowing study is needed before oral fluids are started.

Which action will the public health nurse take to reduce the incidence of epidemic encephalitis in a community? a. Teach about prophylactic antibiotics after exposure to encephalitis. b. Encourage the use of effective insect repellent during mosquito season. c. Remind patients that most cases of viral encephalitis can be cared for at home. d. Arrange to screen school-age children for West Nile virus during the school year.

b. Encourage the use of effective insect repellent during mosquito season.

After endotracheal suctioning, the nurse notes that the intracranial pressure (ICP) for a patient with a traumatic head injury has increased from 14 to 17 mm Hg. Which action should the nurse take first? a. Document the increase in intracranial pressure. b. Ensure that the patient's neck is in neutral position. c. Notify the health care provider about the change in pressure. d. Increase the rate of the prescribed propofol (Diprivan) infusion.

b. Ensure that the patient's neck is in neutral position.

The public health nurse is planning a program to decrease the incidence of meningitis in teenagers and young adults. Which action is most likely to be effective? a. Emphasize the importance of hand washing. b. Immunize adolescents and college freshman. c. Support serving healthy nutritional options in the college cafeteria. d. Encourage adolescents and young adults to avoid crowds in the winter.

b. Immunize adolescents and college freshman.

36. A 49-year-old man has been admitted with hypotension and dehydration after 3 days of nausea and vomiting. Which order from the health care provider will the nurse implement first? a. Insert a nasogastric (NG) tube. b. Infuse normal saline at 250 mL/hr. c. Administer IV ondansetron (Zofran). d. Provide oral care with moistened swabs.

b. Infuse normal saline at 250 mL/hr. Because the patient has severe dehydration, rehydration with IV fluids is the priority. The other orders should be accomplished as quickly as possible after the IV fluids are initiated.

The nurse is admitting a patient with a basal skull fracture. The nurse notes ecchymoses around both eyes and clear drainage from the patient's nose. Which admission order should the nurse question? a. Keep the head of bed elevated. b. Insert nasogastric tube to low suction. c. Turn patient side to side every 2 hours. d. Apply cold packs intermittently to face.

b. Insert nasogastric tube to low suction.

A patient who is suspected of having an epidural hematoma is admitted to the emergency department. Which action will the nurse expect to take? a. Administer IV furosemide (Lasix). b. Prepare the patient for craniotomy. c. Initiate high-dose barbiturate therapy. d. Type and crossmatch for blood transfusion.

b. Prepare the patient for craniotomy.

Which action will the emergency department nurse anticipate for a patient diagnosed with a concussion who did not lose consciousness? a. Coordinate the transfer of the patient to the operating room. b. Provide discharge instructions about monitoring neurologic status. c. Transport the patient to radiology for magnetic resonance imaging (MRI). d. Arrange to admit the patient to the neurologic unit for 24 hours of observation.

b. Provide discharge instructions about monitoring neurologic status.

A patient admitted with a diffuse axonal injury has a systemic blood pressure (BP) of 106/52 mm Hg and an intracranial pressure (ICP) of 14 mm Hg. Which action should the nurse take first? a. Document the BP and ICP in the patient's record. b. Report the BP and ICP to the health care provider. c. Elevate the head of the patient's bed to 60 degrees. d. Continue to monitor the patient's vital signs and ICP.

b. Report the BP and ICP to the health care provider.

After evacuation of an epidural hematoma, a patient's intracranial pressure (ICP) is being monitored with an intraventricular catheter. Which information obtained by the nurse requires urgent communication with the health care provider? a. Pulse of 102 beats/min b. Temperature of 101.6° F c. Intracranial pressure of 15 mm Hg d. Mean arterial pressure of 90 mm Hg

b. Temperature of 101.6° F

44. The nurse and a licensed practical/vocational nurse (LPN/LVN) are working together to care for a patient who had an esophagectomy 2 days ago. Which action by the LPN/LVN requires that the nurse intervene? a. The LPN/LVN uses soft swabs to provide for oral care. b. The LPN/LVN positions the head of the bed in the flat position. c. The LPN/LVN encourages the patient to use pain medications before coughing. d. The LPN/LVN includes the enteral feeding volume when calculating intake and output.

b. The LPN/LVN positions the head of the bed in the flat position. The patient's bed should be in Fowler's position to prevent reflux and aspiration of gastric contents. The other actions by the LPN/LVN are appropriate.

41. The nurse is administering IV fluid boluses and nasogastric irrigation to a patient with acute gastrointestinal (GI) bleeding. Which assessment finding is most important for the nurse to communicate to the health care provider? a. The bowel sounds are hyperactive in all four quadrants. b. The patient's lungs have crackles audible to the midchest. c. The nasogastric (NG) suction is returning coffee-ground material. d. The patient's blood pressure (BP) has increased to 142/84 mm Hg.

b. The patient's lungs have crackles audible to the midchest. The patient's lung sounds indicate that pulmonary edema may be developing as a result of the rapid infusion of IV fluid and that the fluid infusion rate should be slowed. The return of coffee-ground material in an NG tube is expected for a patient with upper GI bleeding. The BP is slightly elevated but would not be an indication to contact the health care provider immediately. Hyperactive bowel sounds are common when a patient has GI bleeding.

The nurse admitting a patient who has a right frontal lobe tumor would expect the patient may have a. expressive aphasia. b. impaired judgment. c. right-sided weakness. d. difficulty swallowing.

b. impaired judgment.

22. A family member of a 28-year-old patient who has suffered massive abdominal trauma in an automobile accident asks the nurse why the patient is receiving famotidine (Pepcid). The nurse will explain that the medication will a. decrease nausea and vomiting. b. inhibit development of stress ulcers. c. lower the risk for H. pylori infection. d. prevent aspiration of gastric contents.

b. inhibit development of stress ulcers. Famotidine is administered to prevent the development of physiologic stress ulcers, which are associated with a major physiologic insult such as massive trauma. Famotidine does not decrease nausea or vomiting, prevent aspiration, or prevent H. pylori infection.

23. A 68-year-old patient with a bleeding duodenal ulcer has a nasogastric (NG) tube in place, and the health care provider orders 30 mL of aluminum hydroxide/magnesium hydroxide (Maalox) to be instilled through the tube every hour. To evaluate the effectiveness of this treatment, the nurse a. monitors arterial blood gas values daily. b. periodically aspirates and tests gastric pH. c. checks each stool for the presence of occult blood. d. measures the volume of residual stomach contents.

b. periodically aspirates and tests gastric pH. The purpose for antacids is to increase gastric pH. Checking gastric pH is the most direct way of evaluating the effectiveness of the medication. Arterial blood gases may change slightly, but this does not directly reflect the effect of antacids on gastric pH. Because the patient has upper gastrointestinal (GI) bleeding, occult blood in the stools will appear even after the acute bleeding has stopped. The amount of residual stomach contents is not a reflection of resolution of bleeding or of gastric pH.

32. A 26-year-old patient with a family history of stomach cancer asks the nurse about ways to decrease the risk for developing stomach cancer. The nurse will teach the patient to avoid a. emotionally stressful situations. b. smoked foods such as ham and bacon. c. foods that cause distention or bloating. d. chronic use of H2 blocking medications.

b. smoked foods such as ham and bacon Smoked foods such as bacon, ham, and smoked sausage increase the risk for stomach cancer. Stressful situations, abdominal distention, and use of H2 blockers are not associated with an increased incidence of stomach cancer.

Which nursing action will be included in the care for a patient who has had cerebral angiography?

bleeding

Propranolol (Inderal), a β-adrenergic blocker that inhibits sympathetic nervous system activity, is prescribed for a patient who has extreme anxiety about public speaking. The nurse monitors the patient for

bradycardia.

11. A 58-year-old woman who recently has been diagnosed with esophageal cancer tells the nurse, "I do not feel ready to die yet." Which response by the nurse is most appropriate? a. "You may have quite a few years still left to live." b. "Thinking about dying will only make you feel worse." c. "Having this new diagnosis must be very hard for you." d. "It is important that you be realistic about your prognosis."

c. "Having this new diagnosis must be very hard for you." This response is open-ended and will encourage the patient to further discuss feelings of anxiety or sadness about the diagnosis. Patients with esophageal cancer have only a low survival rate, so the response "You may have quite a few years still left to live" is misleading. The response beginning, "Thinking about dying" indicates that the nurse is not open to discussing the patient's fears of dying. The response beginning, "It is important that you be realistic," discourages the patient from feeling hopeful, which is important to patients with any life-threatening diagnosis.

6. A 46-year-old female with gastroesophageal reflux disease (GERD) is experiencing increasing discomfort. Which patient statement indicates that additional teaching about GERD is needed? a. "I take antacids between meals and at bedtime each night." b. "I sleep with the head of the bed elevated on 4-inch blocks." c. "I eat small meals during the day and have a bedtime snack." d. "I quit smoking several years ago, but I still chew a lot of gum."

c. "I eat small meals during the day and have a bedtime snack." GERD is exacerbated by eating late at night, and the nurse should plan to teach the patient to avoid eating at bedtime. The other patient actions are appropriate to control symptoms of GERD.

A patient with a head injury opens his eyes to verbal stimulation, curses when stimulated, and does not respond to a verbal command to move but attempts to push away a painful stimulus. The nurse records the patient's Glasgow Coma Scale score as a. 9. b. 11. c. 13. d. 15.

c. 13.

37. Which patient should the nurse assess first after receiving change-of-shift report? a. A patient with nausea who has a dose of metoclopramide (Reglan) due b. A patient who is crying after receiving a diagnosis of esophageal cancer c. A patient with esophageal varices who has a blood pressure of 92/58 mm Hg d. A patient admitted yesterday with gastrointestinal (GI) bleeding who has melena

c. A patient with esophageal varices who has a blood pressure of 92/58 mm Hg The patient's history and blood pressure indicate possible hemodynamic instability caused by GI bleeding. The data about the other patients do not indicate acutely life-threatening complications.

40. Which order from the health care provider will the nurse implement first for a patient who has vomited 1200 mL of blood? a. Give an IV H2 receptor antagonist. b. Draw blood for typing and crossmatching. c. Administer 1000 mL of lactated Ringer's solution. d. Insert a nasogastric (NG) tube and connect to suction.

c. Administer 1000 mL of lactated Ringer's solution. Because the patient has vomited a large amount of blood, correction of hypovolemia and prevention of hypovolemic shock are the priorities. The other actions also are important to implement quickly but are not the highest priorities.

20. A 57-year-old man with Escherichia coli O157:H7 food poisoning is admitted to the hospital with bloody diarrhea and dehydration. Which order will the nurse question? a. Infuse lactated Ringer's solution at 250 mL/hr. b. Monitor blood urea nitrogen and creatinine daily. c. Administer loperamide (Imodium) after each stool. d. Provide a clear liquid diet and progress diet as tolerated.

c. Administer loperamide (Imodium) after each stool. Use of antidiarrheal agents is avoided with this type of food poisoning. The other orders are appropriate.

1.A 53-year-old male patient with deep partial-thickness burns from a chemical spill in the workplace experiences severe pain followed by nausea during dressing changes. Which action will be most useful in decreasing the patient's nausea? a. Keep the patient NPO for 2 hours before and after dressing changes. b. Avoid performing dressing changes close to the patient's mealtimes. c. Administer the prescribed morphine sulfate before dressing changes. d. Give the ordered prochlorperazine (Compazine) before dressing changes.

c. Administer the prescribed morphine sulfate before dressing changes. Because the patient's nausea is associated with severe pain, it is likely that it is precipitated by stress and pain. The best treatment will be to provide adequate pain medication before dressing changes. The nurse should avoid doing painful procedures close to mealtimes, but nausea/vomiting that occur at other times also should be addressed. Keeping the patient NPO does not address the reason for the nausea and vomiting and will have an adverse effect on the patient's nutrition. Administration of antiemetics is not the best choice for a patient with nausea caused by pain.

An unconscious patient is admitted to the emergency department (ED) with a head injury. The patient's spouse and teenage children stay at the patient's side and ask many questions about the treatment being given. What action is best for the nurse to take? a. Call the family's pastor or spiritual advisor to take them to the chapel. b. Ask the family to stay in the waiting room until the assessment is completed. c. Allow the family to stay with the patient and briefly explain all procedures to them. d. Refer the family members to the hospital counseling service to deal with their anxiety.

c. Allow the family to stay with the patient and briefly explain all procedures to them.

35. A 26-year-old woman has been admitted to the emergency department with nausea and vomiting. Which action could the RN delegate to unlicensed assistive personnel (UAP)? a. Auscultate the bowel sounds. b. Assess for signs of dehydration. c. Assist the patient with oral care. d. Ask the patient about the nausea.

c. Assist the patient with oral care. Oral care is included in UAP education and scope of practice. The other actions are all assessments that require more education and a higher scope of nursing practice.

A patient with increased intracranial pressure after a head injury has a ventriculostomy in place. Which action can the nurse delegate to unlicensed assistive personnel (UAP) who regularly work in the intensive care unit? a. Document intracranial pressure every hour. b. Turn and reposition the patient every 2 hours. c. Check capillary blood glucose level every 6 hours. d. Monitor cerebrospinal fluid color and volume hourly.

c. Check capillary blood glucose level every 6 hours.

9. Which patient choice for a snack 2 hours before bedtime indicates that the nurse's teaching about gastroesophageal reflux disease (GERD) has been effective? a. Chocolate pudding b. Glass of low-fat milk c. Cherry gelatin with fruit d. Peanut butter and jelly sandwich

c. Cherry gelatin with fruit Gelatin and fruit are low fat and will not decrease lower esophageal sphincter (LES) pressure. Foods such as chocolate are avoided because they lower LES pressure. Milk products increase gastric acid secretion. High-fat foods such as peanut butter decrease both gastric emptying and LES pressure.

The nurse has administered prescribed IV mannitol (Osmitrol) to an unconscious patient. Which parameter should the nurse monitor to determine the medication's effectiveness? a. Blood pressure b. Oxygen saturation c. Intracranial pressure d. Hemoglobin and hematocrit

c. Intracranial pressure

A patient who is unconscious has ineffective cerebral tissue perfusion and cerebral tissue swelling. Which nursing intervention will be included in the plan of care? a. Encourage coughing and deep breathing. b. Position the patient with knees and hips flexed. c. Keep the head of the bed elevated to 30 degrees. d. Cluster nursing interventions to provide rest periods.

c. Keep the head of the bed elevated to 30 degrees.

43. An 80-year-old who is hospitalized with peptic ulcer disease develops new-onset auditory hallucinations. Which prescribed medication will the nurse discuss with the health care provider before administration? a. Sucralfate (Carafate) b. Omeprazole (Prilosec) c. Metoclopramide (Reglan) d. Aluminum hydroxide (Amphojel)

c. Metoclopramide (Reglan) Metoclopramide can cause central nervous system (CNS) side effects ranging from anxiety to hallucinations. Hallucinations are not a side effect of proton-pump inhibitors, mucosal protectants, or antacids.

42. After the nurse has completed teaching a patient with newly diagnosed eosinophilic esophagitis about the management of the disease, which patient action indicates that the teaching has been effective? a. Patient orders nonfat milk for each meal. b. Patient uses the prescribed corticosteroid inhaler. c. Patient schedules an appointment for allergy testing. d. Patient takes ibuprofen (Advil) to control throat pain.

c. Patient schedules an appointment for allergy testing. Eosinophilic esophagitis is frequently associated with environmental allergens, so allergy testing is used to determine possible triggers. Corticosteroid therapy may be prescribed, but the medication will be swallowed, not inhaled. Milk is a frequent trigger for attacks. NSAIDs are not used for eosinophilic esophagitis.

Which information about a 30-yr-old patient who is hospitalized after a traumatic brain injury requires the most rapid action by the nurse? a. Intracranial pressure of 15 mm Hg b. Cerebrospinal fluid (CSF) drainage of 25 mL/hr c. Pressure of oxygen in brain tissue (PbtO2) is 14 mm Hg d. Cardiac monitor shows sinus tachycardia at 120 beats/minute

c. Pressure of oxygen in brain tissue (PbtO2) is 14 mm Hg

4. Which finding in the mouth of a patient who uses smokeless tobacco is suggestive of oral cancer? a. Bleeding during tooth brushing b. Painful blisters at the lip border c. Red, velvety patches on the buccal mucosa d. White, curdlike plaques on the posterior tongue

c. Red, velvety patches on the buccal mucosa A red, velvety patch suggests erythroplasia, which has a high incidence (greater than 50%) of progression to squamous cell carcinoma. The other lesions are suggestive of acute processes (e.g., gingivitis, oral candidiasis, herpes simplex).

33. The nurse is assessing a patient who had a total gastrectomy 8 hours ago. What information is most important to report to the health care provider? a. Absent bowel sounds b. Complaints of incisional pain c. Temperature 102.1° F (38.9° C) d. Scant nasogastric (NG) tube drainage

c. Temperature 102.1° F (38.9° C) An elevation in temperature may indicate leakage at the anastomosis, which may require return to surgery or keeping the patient NPO. The other findings are expected in the immediate postoperative period for patients who have this surgery.

38. A patient returned from a laparoscopic Nissen fundoplication for hiatal hernia 4 hours ago. Which assessment finding is most important for the nurse to address immediately? a. The patient is experiencing intermittent waves of nausea. b. The patient complains of 7/10 (0 to 10 scale) abdominal pain. c. The patient has absent breath sounds in the left anterior chest. d. The patient has hypoactive bowel sounds in all four quadrants.

c. The patient has absent breath sounds in the left anterior chest. Decreased breath sounds on one side may indicate a pneumothorax, which requires rapid diagnosis and treatment. The nausea and abdominal pain should also be addressed but they are not as high priority as the patient's respiratory status. The patient's decreased bowel sounds are expected after surgery and require ongoing monitoring but no other action.

34. A 58-year-old patient has just been admitted to the emergency department with nausea and vomiting. Which information requires the most rapid intervention by the nurse? a. The patient has been vomiting for 4 days. b. The patient takes antacids 8 to 10 times a day. c. The patient is lethargic and difficult to arouse. d. The patient has undergone a small intestinal resection.

c. The patient is lethargic and difficult to arouse. A lethargic patient is at risk for aspiration, and the nurse will need to position the patient to decrease aspiration risk. The other information is also important to collect, but it does not require as quick action as the risk for aspiration.

The nurse is caring for a patient who has a head injury and fractured right arm after being assaulted. Which assessment information requires rapid action by the nurse? a. The apical pulse is slightly irregular. b. The patient complains of a headache. c. The patient is more difficult to arouse. d. The blood pressure (BP) increases to 140/62 mm Hg.

c. The patient is more difficult to arouse.

The charge nurse observes an inexperienced staff nurse caring for a patient who has had a craniotomy for resection of a brain tumor. Which action by the inexperienced nurse requires the charge nurse to intervene? a. The staff nurse assesses neurologic status every hour. b. The staff nurse elevates the head of the bed to 30 degrees. c. The staff nurse suctions the patient routinely every 2 hours. d. The staff nurse administers an analgesic before turning the patient.

c. The staff nurse suctions the patient routinely every 2 hours.

3. A 38-year old woman receiving chemotherapy for breast cancer develops a Candida albicans oral infection. The nurse will anticipate the need for a. hydrogen peroxide rinses. b. the use of antiviral agents. c. administration of nystatin (Mycostatin) tablets. d. referral to a dentist for professional tooth cleaning.

c. administration of nystatin (Mycostatin) tablets. Candida albicans is treated with an antifungal such as nystatin. Oral saltwater rinses may be used but will not cure the infection. Antiviral agents are used for viral infections such as herpes simplex. Referral to a dentist is indicated for gingivitis but not for Candida infection.

7. A 68-year-old male patient with a stroke is unconscious and unresponsive to stimuli. After learning that the patient has a history of gastroesophageal reflux disease (GERD), the nurse will plan to do frequent assessments of the patient's a. apical pulse. b. bowel sounds. c. breath sounds. d. abdominal girth.

c. breath sounds. Because GERD may cause aspiration, the unconscious patient is at risk for developing aspiration pneumonia. Bowel sounds, abdominal girth, and apical pulse will not be affected by the patient's stroke or GERD and do not require more frequent monitoring than the routine.

When a brain-injured patient responds to nail bed pressure with internal rotation, adduction, and flexion of the arms, the nurse reports the response as a. flexion withdrawal. b. localization of pain. c. decorticate posturing. d. decerebrate posturing.

c. decorticate posturing.

27. At his first postoperative checkup appointment after a gastrojejunostomy (Billroth II), a patient reports that dizziness, weakness, and palpitations occur about 20 minutes after each meal. The nurse will teach the patient to a. increase the amount of fluid with meals. b. eat foods that are higher in carbohydrates. c. lie down for about 30 minutes after eating. d. drink sugared fluids or eat candy after meals.

c. lie down for about 30 minutes after eating. The patient is experiencing symptoms of dumping syndrome, which may be reduced by lying down after eating. Increasing fluid intake and choosing high carbohydrate foods will increase the risk for dumping syndrome. Having a sweet drink or hard candy will correct the hypoglycemia that is associated with dumping syndrome but will not prevent dumping syndrome.

45. After change-of-shift report, which patient should the nurse assess first? a. 42-year-old who has acute gastritis and ongoing epigastric pain b. 70-year-old with a hiatal hernia who experiences frequent heartburn c. 53-year-old who has dumping syndrome after a recent partial gastrectomy d. 60-year-old with nausea and vomiting who has dry oral mucosa and lethargy

d. 60-year-old with nausea and vomiting who has dry oral mucosa and lethargy This older patient is at high risk for problems such as aspiration, dehydration, and fluid and electrolyte disturbances. The other patients will also need to be assessed, but the information about them indicates symptoms that are typical for their diagnoses and are not life threatening.

After the emergency department nurse has received a status report on the following patients who have been admitted with head injuries, which patient should the nurse assess first? a. A 20-yr-old patient whose cranial x-ray shows a linear skull fracture b. A 50-yr-old patient who has an initial Glasgow Coma Scale score of 13 c. A 30-yr-old patient who lost consciousness for a few seconds after a fall d. A 40-yr-old patient whose right pupil is 10 mm and unresponsive to light

d. A 40-yr-old patient whose right pupil is 10 mm and unresponsive to light

31. A 73-year-old patient is diagnosed with stomach cancer after an unintended 20-pound weight loss. Which nursing action will be included in the plan of care? a. Refer the patient for hospice services. b. Infuse IV fluids through a central line. c. Teach the patient about antiemetic therapy. d. Offer supplemental feedings between meals.

d. Offer supplemental feedings between meals. The patient data indicate a poor nutritional state and improvement in nutrition will be helpful in improving the response to therapies such as surgery, chemotherapy, or radiation. Nausea and vomiting are not common clinical manifestations of stomach cancer. There is no indication that the patient requires hospice or IV fluid infusions.

A patient has been admitted with meningococcal meningitis. Which observation by the nurse requires action? a. The patient receives a regular diet tray. b. The bedrails on both sides of the bed are elevated. c. Staff have turned off the lights in the patient's room. d. Staff have entered the patient's room without a mask.

d. Staff have entered the patient's room without a mask.

A patient being admitted with bacterial meningitis has a temperature of 102.5° F (39.2° C) and a severe headache. Which order should the nurse implement first? a. Administer ceftizoxime (Cefizox) 1 g IV. b. Give acetaminophen (Tylenol) 650 mg PO. c. Use a cooling blanket to lower temperature. d. Swab the nasopharyngeal mucosa for cultures.

d. Swab the nasopharyngeal mucosa for cultures.

39. Which assessment should the nurse perform first for a patient who just vomited bright red blood? a. Measuring the quantity of emesis b. Palpating the abdomen for distention c. Auscultating the chest for breath sounds d. Taking the blood pressure (BP) and pulse

d. Taking the blood pressure (BP) and pulse The nurse is concerned about blood loss and possible hypovolemic shock in a patient with acute gastrointestinal (GI) bleeding. BP and pulse are the best indicators of these complications. The other information is important to obtain, but BP and pulse rate are the best indicators for assessing intravascular volume.

The nurse is caring for a patient who was admitted the previous day with a basilar skull fracture after a motor vehicle crash. Which assessment finding indicates a possible complication that should be reported to the health care provider? a. Complaint of severe headache b. Large contusion behind left ear c. Bilateral periorbital ecchymosis d. Temperature of 101.4° F (38.6° C)

d. Temperature of 101.4° F (38.6° C)

When assessing a 53-yr-old patient with bacterial meningitis, the nurse obtains the following data. Which finding requires the most immediate intervention? a. The patient exhibits nuchal rigidity. b. The patient has a positive Kernig's sign. c. The patient's temperature is 101° F (38.3° C). d. The patient's blood pressure is 88/42 mm Hg.

d. The patient's blood pressure is 88/42 mm Hg.

29. The health care provider prescribes antacids and sucralfate (Carafate) for treatment of a patient's peptic ulcer. The nurse will teach the patient to take a. sucralfate at bedtime and antacids before each meal. b. sucralfate and antacids together 30 minutes before meals. c. antacids 30 minutes before each dose of sucralfate is taken. d. antacids after meals and sucralfate 30 minutes before meals.

d. antacids after meals and sucralfate 30 minutes before meals. Sucralfate is most effective when the pH is low and should not be given with or soon after antacids. Antacids are most effective when taken after eating. Administration of sucralfate 30 minutes before eating and antacids just after eating will ensure that both drugs can be most effective. The other regimens will decrease the effectiveness of the medications.

28. A 62-year-old man patient who requires daily use of a nonsteroidal antiinflammatory drug (NSAID) for the management of severe rheumatoid arthritis has recently developed melena. The nurse will anticipate teaching the patient about a. substitution of acetaminophen (Tylenol) for the NSAID. b. use of enteric-coated NSAIDs to reduce gastric irritation. c. reasons for using corticosteroids to treat the rheumatoid arthritis. d. misoprostol (Cytotec) to protect the gastrointestinal (GI) mucosa.

d. misoprostol (Cytotec) to protect the gastrointestinal (GI) mucosa. Misoprostol, a prostaglandin analog, reduces acid secretion and the incidence of upper GI bleeding associated with NSAID use. Enteric coating of NSAIDs does not reduce the risk for GI bleeding. Corticosteroids increase the risk for ulcer development, and will not be substituted for NSAIDs for this patient. Acetaminophen will not be effective in treating the patient's rheumatoid arthritis.

After having a craniectomy and left anterior fossae incision, a 64-yr-old patient has impaired physical mobility related to decreased level of consciousness and weakness. An appropriate nursing intervention is to a. cluster nursing activities to allow longer rest periods. b. turn and reposition the patient side to side every 2 hours. c. position the bed flat and log roll to reposition the patient. d. perform range-of-motion (ROM) exercises every 4 hours.

d. perform range-of-motion (ROM) exercises every 4 hours.

10. The nurse will anticipate teaching a patient experiencing frequent heartburn about a. a barium swallow. b. radionuclide tests. c. endoscopy procedures. d. proton pump inhibitors.

d. proton pump inhibitors. Because diagnostic testing for heartburn that is probably caused by gastroesophageal reflux disease (GERD) is expensive and uncomfortable, proton pump inhibitors are frequently used for a short period as the first step in the diagnosis of GERD. The other tests may be used but are not usually the first step in diagnosis.

4. When a 72-year-old patient is diagnosed with achalasia, the nurse will teach the patient that a. lying down after meals is recommended. b. a liquid or blenderized diet will be necessary. c. drinking fluids with meals should be avoided. d. treatment may include endoscopic procedures.

d. treatment may include endoscopic procedures. Endoscopic and laparoscopic procedures are the most effective therapy for improving symptoms caused by achalasia. Keeping the head elevated after eating will improve esophageal emptying. A semisoft diet is recommended to improve esophageal emptying. Patients are advised to drink fluid with meals.

8. The nurse explaining esomeprazole (Nexium) to a patient with recurring heartburn describes that the medication a. reduces gastroesophageal reflux by increasing the rate of gastric emptying. b. neutralizes stomach acid and provides relief of symptoms in a few minutes. c. coats and protects the lining of the stomach and esophagus from gastric acid. d. treats gastroesophageal reflux disease by decreasing stomach acid production.

d. treats gastroesophageal reflux disease by decreasing stomach acid production. The proton pump inhibitors decrease the rate of gastric acid secretion. Promotility drugs such as metoclopramide (Reglan) increase the rate of gastric emptying. Cryoprotective medications such as sucralfate (Carafate) protect the stomach. Antacids neutralize stomach acid and work rapidly.

15. A 50-year-old man vomiting blood-streaked fluid is admitted to the hospital with acute gastritis. To determine possible risk factors for gastritis, the nurse will ask the patient about a. the amount of saturated fat in the diet. b. any family history of gastric or colon cancer. c. a history of a large recent weight gain or loss. d. use of nonsteroidal antiinflammatory drugs (NSAIDs).

d. use of nonsteroidal antiinflammatory drugs (NSAIDs). Use of an NSAID is associated with damage to the gastric mucosa, which can result in acute gastritis. Family history, recent weight gain or loss, and fatty foods are not risk factors for acute gastritis.

During the neurologic assessment, the patient is unable to respond verbally to the nurse but cooperates with the nurse's directions to move his hands and feet. The nurse will suspect

frontal lobe damage.

A patient with suspected meningitis is scheduled for a lumbar puncture. Before the procedure, the nurse will plan to

help the patient to a lateral position.

A 45-year-old patient has a dysfunction of the cerebellum. The nurse will plan interventions to

prevent falls.

The priority nursing assessment for a 72-year-old patient being admitted with a brainstem infarction is

respiratory rate and rhythm.

To assess the functioning of the trigeminal and facial nerves (CNs V and VII), the nurse should

shine a light into the patient's pupil.

The nurse performing a focused assessment of left posterior temporal lobe functions will assess the patient for

understanding written and oral language.


Kaugnay na mga set ng pag-aaral

PT677- Ankle Sprains & Complications

View Set

Week 3 - Requirements Elicitation

View Set

OB/GYN Registry Review Chapter 32 Fetal Environment And Maternal Complications

View Set

Chapter 11 NCLEX-Style Review Questions

View Set

Integrated Business Policy & Strategy Midterm

View Set

Chapter 5 Cost-Volume-Profit-Relationships

View Set